Sunteți pe pagina 1din 216

DUMITRU BU NEAG ( COORDONATOR ) FLORENTINA BOBOC DANA PICIU

EDITURA UNIVERSITARIA CRAIOVA 1999

Referen i !tiin ifici : Prof. univ. dr. Alexandru Dinc $ Universitatea din Craiova. Prof. univ. dr. Franois Gramain Universit Jean Monnnet, Saint -tienne, France. Dumitru Busneag, Florentina Boboc, Dana Piciu: Arithmetic and number theory 1999 EUC CRAIOVA All rights reserved. No part of this publication may be reproduced, stored in a retrieval system, or transmitted, in any form or by any means, electronic, mechanical, photocopying, recording, or other wise, without the prior written permission of the publisher.

Tehnoredactare computerizat : Florentina Boboc, Dana Piciu Bun de tipar: 11.05.1999 Tipografia Universit #ii din Craiova Str. Al. I. Cuza Craiova, Romnia. Published in Romania by Editura Universitaria Craiova ISBN: 973 9271 73 - 1

CUVNT NAINTE Aceast lucrare este o edi#ie revizuit $i mbun t #it a lucr rii Elemente de aritmetic$ !i teoria numerelor, avnd aceia$i autori, $i care a fost publicat n anul 1998, la editura Radical din Craiova (I.S.B.N. 973-9253-52-0). Fa # de vechea edi#ie, pe lng ndreptarea unor mici erori (att de redactare ct $i de tehnoredactare ), am adus mbun t #iri paragrafelor 4 $i 7 de la Capitolul 7, ca $i paragrafului 3 de la Capitolul 11. n finalul Capitolului 12 am introdus un nou paragraf (paragraful 6) n care se prezint rezolvarea n numere ntregi a sistemelor de ecua #ii liniare cu coeficien#i ntregi. Pentru fiecare capitol s-au introdus exerci#ii suplimentare cu solu#ii complete. n finalul lucr rii s-au ata$at urm toarele anexe: Anexa 1: Tabelul cu numerele prime ( eviden iind numerele prime gemene) de la 1 la 10.000. Anexa 2: Func ia p ( x ) !i estim$rile sale. Anexa 3: Numerele lui Fermat, numerele lui Mersenne !i numere perfecte. Dac lucrarea ini#ial avea 254 pagini format A5 , prezenta edi #ie are 288 pagini (acela $i format ). Craiova, 20 aprilie 1999. Autorii.

L. Kronecker : Dumnezeu a creat numerele naturale restul este munca omului . CAPITOLUL 1 : MUL'IMEA NUMERELOR NATURALE 1 Triplete Peano DEFINI'IA 1.1. Numim triplet Peano un triplet ( N, 0, s ) unde .

N este o mul ime nevid$, 0!N iar s:N(N este o func ie astfel nct sunt verificate axiomele : P1 : 0"s( N ) P2 : s este o func ie injectiv$ P3 : dac$ P#N este o submul ime astfel nct 0 !P !i (n!P$s(n)!P ), atunci P=N . n cele ce urmeaz , accept m ca axiom existen#a unui triplet Peano (cititorului dornic de aprofundarea acestei chestiuni i recomand m lucr rile [7] $i [19] ) . LEMA 1.2. N={0}%s(N). Dac$ ( N, 0, s ) este un triplet Peano, atunci

Demonstra #ie Dac not m P={0}%s (N), atunci P #N $i cum P verific

P3, deducem c P=N .& TEOREMA 1.3. Fie ( N, 0, s ) un triplet Peano iar ( N ', 0', s ' ) un alt triplet format dintr-o mul ime nevid$ N', un element 0'!N' !i o func ie s':N' ( N'. Atunci : 1 ) Exist$ o unic$ func ie f:N(N' astfel nct f(0)= 0 ', iar diagrama N
s

f N'
s'

N' f
4

este comutativ$ (adic$ f ( s = s'(f ) . 2 ) Dac$ ( N', 0', s') este un triplet Peano, atunci f este bijec ie. Demonstra#ie 1) Pentru a proba existen#a lui f, vom considera toate rela#iile R#NN' a.. : r1 : (0, 0') ! R r2 : Dac (n, n')!R, atunci (s(n), s'(n'))!R iar prin R0 vom nota intersec#ia acestor rela#ii . Vom demonstra c R0 este o rela#ie func#ional $i astfel f va fi func#ia ce va avea drept grafic pe R0 (astfel, din (0, 0')!R0 vom deduce c f (0)=0' iar dac n!N $i f (n)=n'!N', (n , n')!R0, deci (s(n), s'(n'))!R0, adic , f(s(n))=s'(n')=s'(f (n)). Pentru a demonstra c R0 este o rela#ie func#ional , vom demonstra c pentru orice n!N, exist n'!N' a. . (n, n')!R
0

iar dac pentru n!N $i n',

n''!N' avem (n, n')!R0 $i (n, n'')!R0 , atunci n'= n'' . Pentru prima parte, fie P={n!N : exist n'!N' a. . (n, n')!R0 }#N. Cum (0, 0')!R0 deducem c 0!P. Fie acum n!P $i n'!N' a.. (n, n')!R0. Din defini#ia lui R0 deducem c (s(n), s'(n'))!R0 ; ob#inem c s(n)!P $i cum (N, 0, s) este triplet Peano, deducem c P=N. Pentru a doua parte, fie Q={n!N : dac n', n''!N ' $i (n, n'), (n, n'')!R0 $ n'= n''}#N $i s demonstr m la nceput c 0!Q. n acest sens, vom demonstra c dac (0, n')!R0 atunci n'=0'. Dac prin absurd, n''0', atunci vom considera rela#ia R1=R0 ){(0, n')}#NN'. Din n''0' deducem c (0, 0')!R1 iar dac pentru m!N' avem (n, m)!R1 , atunci (n, m)!R0 $i (n , m) ' (0, n'). Astfel (s(n), s'(m))!R0 $i cum (s(n), s'(m))'(0, n') (c ci s(n) ' 0 conform cu P1), deducem c (s(n), s'(m))!R1 . Cum R1 verific r1 $i r2 ar trebui ca R0#R1 absurd (c ci R1 este inclus strict n R0 ). Pentru a proba c 0!Q, fie n', n''!N' a. . (0, n'), (0 , n'')!R0. Atunci, #innd cont de cele stabilite mai sus, deducem c n'=n''=0', deci 0!Q. Fie acum n!Q $i n '!N ' a. . (n, n')!R0 ; vom demonstra c dac (s(n), n'')!R0, atunci n''=s'(n'). S presupunem prin absurd c n''' s'(n') $i s consider m rela#ia R2 =R0 ){(s (n), n'')} . Vom demonstra c R2 verific r1 $i r2 . 5

ntradev r, (0, 0')!R2 ( c ci 0 ' s(n) ) iar dac (p, p')!R2 , atunci (p, p') !R0 $i (p, p')'( s(n), n'') . Deducem c (s(p), s'(p'))!R0 $i dac presupunem (s(p), s'(p'))= =(s(n), n''), atunci s(p) =s(n), deci p=n. De asemenea, s '(p')=n''. Atunci (n, n')!R0 $i (n, p')!R0 iar cum n!Q $ n'=p', deci n''=s'(p')=s'(n'), ceea ce contrazice faptul c n'''s(n'). Prin urmare, (s(p), s'(p')) ' (s(n), n''), ceea ce ne arat c (s(p), s'(p'))!R2 , adic R2 satisface r1 $i r2 . Din nou ar trebui ca R0*R2 absurd !. Deci (s (n), n'')!R0 $ n''=s'(n') astfel c dac r, s !N ' $i (s(n), r), (s(n), s )!R0 , atunci r = s = s'(n), adic s(n)!Q, deci Q=N. Pentru a proba unicitatea lui f, s presupunem c mai exist f':N(N' a.. f'(0)=0' $i s'(f'(n))=f'(s(n)) pentru orice n!N. Considernd P={n!N : f(n)=f'(n)}#N, atunci 0!P iar dac (adic n!P f(n)=f'(n)), atunci s'(f(n))=s'(f'(n))$f(s(n))=f'(s(n))$s(n)!P $i atunci

P=N, adic f=f'. 2) S ar t m la nceput c f este injectiv . Pentru aceasta vom considera P={n!N : dac m!N $i f(m)=f(n)$m=n}#N $i s demonstr m la nceput c 0!P. Pentru aceasta fie m!N a. . f(0)=f(m) $i s demonstr m c m=0. Dac prin absurd m'0, atunci m=s(n) cu n!N iar egalitatea f(m)=f(0) devine f(s(n))=f(0)= =0', de unde s'(f(n))=0', ceea ce este absurd deoarece prin ipotez este un triplet Peano. Fie acum n!P; pentru a demonstra c f(m)=f(s(n)). (N', 0', s')

s(n)!P, fie m!N a..

Atunci m'0 (c ci n caz contrar ar rezulta c 0'=f(0)=f(s(n))=s'(f(n)), absurd !), deci conform Lemei 1.2., m=s(p) cu p !N iar egalitatea f(m)=f(s(n)) devine f(s(p))=f(s(n))+s'(f(p))=s'(f(n)), adic f(p)=f(n) $i cum n!P, atunci n=p $i astfel m=s(p)=s(n). Pentru a demonstra surjectivitatea lui f s consider m P'={n'!N':exist n!N a. . n'=f (n)}#N' . Cum f(0)=0' deducem c 0'!P'. Fie acum n'!P' ; atunci exist n!N a.. n'=f (n). Deoarece s'(n')=s'(f(n))=f(s(n)), deducem c s'(n')!P' $i cum

tripletul (N', 0', s') este un triplet Peano, deducem c P'=N', adic f este $i surjectiv , deci bijectiv . & Observa#ie Conform Teoremei 1.3. (cunoscut $i sub numele de teorema de recuren #$ ) un triplet Peano este unic pn la o bijec#ie. n cele ce urmeaz vom alege un triplet Peano oarecare ( , 0, s) $i pe care l vom fixa ; elementele lui le vom numi numere naturale . *= \ {0}. Elementul 0 va purta numele de zero . Not m

Vom nota 1=s(0), 2=s(1), 3=s(2), e.t.c., astfel c ={0, 1, 2, }. Func#ia s poart numele de func#ia succesor . Axiomele P1 P3 sunt cunoscute sub numele de axiomele lui Peano . Axioma P3 poart numele de axioma induc #iei matematice. 2 Adunarea numerelor naturale TEOREMA 2.1. Exist$ o unic$ opera ie algebric$ pe pe care o vom nota prin + !i o vom numi adunarea numerelor naturale astfel nct pentru orice m, n! s$ avem : A1 : 0+m=m A2 : s(n)+m=s(n+m) . Demonstra#ie S prob m la nceput unicitatea $i pentru aceasta s presupunem c mai exist o opera#ie algebric , pe A2. Fie P={n! a.. sunt verificate A1 $i

| n+m=n,m, pentru orice m! }# .

Din A1 deducem c 0!P iar din A2 deducem c dac n!P, atunci s(n)+m=s(n),m + s(n+m)=s(n,m), ceea ce este adev rat deoarece s este injectiv $i am presupus c n!P. Deci P= , adic cele dou opera#ii coincid. Consider m un element m! =fm(s(n)) pentru orice n! Pentru n! . (pe care l fix m) $i tripletul ( , m, s) ; a. . fm(0)=0 $i s(fm(n))= conform Teoremei 1.3. exist o unic func#ie fm: -

definim n+m=fm (n). Atunci 0+m=fm(0)=m iar s(n)+m=

=fm (s(n))=s (fm (n))=s( n+m ). & Axiomele A1A2 poart numele de axiomele adun $rii Observa#ie numerelor naturale .

PROPOZI'IA 2.2. Pentru orice m, n!


0 1 : 0 A2 :

avem

n+0=n n+s (m)= s(n+m) . Fie P={m! : m+0=m }# . Dac n A1 facem pe 0!P. Dac m!P, (adic m+0=m), atunci

Demonstra#ie m=0, deducem c rela#ie.&

0+0=0, adic

s(m)+0=s(m+0)=s(m), adic s(m)!P, deci P= . Analog se probeaz $i a doua

PROPOZI'IA 2.3. Dubletul ( , +) este monoid comutativ cu proprietatea de simplificare. Demonstra#ie Din cele stabilite anterior, deducem c 0 este element neutru pentru adunarea numerelor naturale. Pentru a proba comutativitatea adun rii s consider m P={n! : n+m=m+n pentru orice m! } # . Evident 0!P. Dac n!P, adic n+m=m+n pentru orice m! , atunci s(n)+m=m+s(n) + s(n+m)=s(m+n) + n+m=m+n, ceea ce este adev rat. Deducem c P= , adic adunarea numerelor naturale este comutativ . Pentru a demonstra asociativitatea adun rii numerelor naturale, s consider m P ={p! : (m+n)+p=m+(n+p) pentru orice m, n ! }# . Evident 0!P. Fie acum n!P. Atunci (s(n)+m)+p=s(n+m)+p= =s(n+(m+p)) iar s(n)+(m+p)=s(n+(m+p)) $i cum (n+m)+p=n+(m+p) deducem c s(n)!P, adic P= . Pentru partea final fie P={p! : dac m+p=n+p $ m=n}# . presupunem c p!P. Atunci m+s(p)=n+s(p) Evident 0!P $i s nou P= . & Observa#ie Dac n! , atunci s(n)=s(n+0)=n+s(0)=n+1. PROPOZI'IA 2.4. Dac$ m, n! !i m+n=0, atunci m=n=0.

+s(m+p)=s(n+p) + m+p=n+p + m=n (c ci p!P), adic s(p)!P $i astfel din

Demonstra#ie Dac m ' 0 sau n ' 0, atunci exist p, q! a. . m = s(p) sau n = s(q). n primul caz, ob #inem c m+n = s(p)+n = s(p+n) ' 0 absurd ! $i analog n al doilea caz. Deci m = n = 0 . & 3 nmul irea numerelor naturale PROPOZI'IA 3.1. Exist$ o unic$ opera ie algebric$ pe notat$ s$

!i numit$ nmul irea numerelor naturale a.. pentru orice m, n! avem : I1 : m0=0 I 2 : ms(n)=mn+m. Demonstra#ie Fie m! fm: este definit

fixat ; considernd tripletul ( , 0, fm ), unde

prin fm(n)=n+m pentru orice n! , atunci conform

Teoremei 1.3. exist o unic func#ie g m : - a.. gm (0)=0 $i fm(gm = gm (s. Definim mn = gm(n) $i astfel m0=gm(0)=0 iar ms(n)=g m(s(n)= =fm(gm(n))=fm(mn)=mn+m . Unicitatea opera#iei de nmul#ire cu propriet #ile I1 $i I2 se probeaz ca n cazul adun rii. & Observa#ie I1 $i I2 poart numele de axiomele nmul#irii numerelor naturale. n cele ce urmeaz , dac nu este pericol de confuzie, vom scrie mn= =mn pentru m, n! . Analog ca n cazul adun rii numerelor naturale, se demonstreaz c pentru oricare numere naturale m, n avem :

I 10 : 0m=0
0 I2 : s(n)m=nm+m.

LEMA 3.2. nmul irea numerelor naturale este distributiv $ la stnga fa $ de adunarea numerelor naturale. Demonstra#ie Fie P={p! n! }# . )innd cont de I1 deducem c 0!P. S presupunem acum c p!P $i fie m, n! . Avem m(n+s(p))=m(s(n+p))=m(n+p)+m=mn+mp+m=mn+ms(p), adic s(p)!P $i astfel P= .& 9 : m(n+p)=mn+mp pentru oricare m,

PROPOZI'IA 3. 3. Dubletul ( , ) este monoid comutativ. Demonstra#ie Pentru a proba asociativitatea nmul #irii fie P={p! : (mn)p=m(np) pentru oricare m, n! }# . n mod evident, 0!P. S presupunem acum c p!P $i s demonstr m c s(p)!P. Avem (mn)s(p)= =(mn)p+mn iar m(ns(p))=m(np+n)=m(np)+mn (conform Lemei 3.2.), de unde egalitatea (mn)s(p)=m(ns(p)), adic s(p)!P, deci P= . Deoarece pentru orice n! avem n1=ns(0)=n0+n=n iar 1n=s(0)n= =0n+n=n deducem c 1 este elementul neutru al nmul#irii numerelor naturale. Pentru a proba comutativitatea nmul#irii numerelor naturale fie P={n! : nm=mn pentru orice m! }# . n mod evident 0!P $i s presupunem c n! . Atunci pentru orice m! , s(n)m=nm+m iar ms(n)=mn+m, de unde s(n)m=ms(n), adic s(n)!P, deci P= . & 4 Rela ia natural$ de ordine de pe .

DEFINI'IA 4.1. Pentru m, n! vom scrie m)n (!i vom spune c$ m este mai mic sau egal dect n sau c$ n este mai mare sau egal dect m) dac$ exist$ p! a.. m+p=n ; convenim n acest caz s $ not$m p=n-m. Dac$ p! *, atunci m)n !i m*n ; n acest caz vom scrie m <n !i vom spune c$ m este strict mai mic dect n. LEMA 4.2. Dac$ m, n! Demonstra#ie exist k! !i m<n, atunci s(m))n.

Deoarece m<n, exist p! * a.. m+p=n. Cum p! *,

a. . p=s(k) (conform Lemei 1.2.). Atunci din m+p=n deducem c

m+s(k)=n $ s(m+k)=n $ s(m)+k=n $s(m)* n . & COROLAR 4.3. Pentru orice n ! , n<s(n). PROPOZI'IA 4.4. Dubletul ( , )) este o mul ime total ordonat $. Demonstra#ie p, q! Deoarece pentru orice n ! , n+0=n deducem c n.n, a. . m*n $i n*m. Atunci exist adic rela#ia * este reflexiv . Fie acum m, n! 10

a.. m+p=n $i n+q=m. Deducem c n+(p+q)=n, de unde p+q=0 (conform

Propozi#iei 2.3. ), iar de aici p=q=0 (conform Propozi #iei 2.4.), adic m=n, deci rela#ia * este antisimetric . Fie acum m, n, p! a. . m*n $i n*p. Atunci exist r, s! a. . m+r=n $i n+s=p. Deducem imediat c m+(r+s)=p, adic m*p, deci rela#ia * este $i tranzitiv , adic * este o rela#ie de ordine pe Pentru a proba c ordinea * de pe . este total , fie m! fixat iar

Pm ={n! : n*m sau m*n}# . n mod evident 0!Pm $i fie n!Pm. Dac n=m, atunci cum n<s(n) avem m<s(n), adic s(n)!Pm . Dac n<m, atunci conform Lemei 4.2. avem s(n)*m $i din nou s(n)!Pm . Dac m<n, cum n<s(n) avem c m<s(n) $i din nou s(n)!Pm. Rezult c Pm= total . & Observa#ie Rela#ia de ordine * definit . anterior pe poart numele de ordinea natural$ de pe $i cum m este oarecare deducem c ordinea * de pe este

TEOREMA 4.5. Dubletul ( , )) este o mul ime bine ordonat$ . Demonstra#ie Trebuie s demonstr m c orice submul#ime nevid A# are un cel mai mic element. Pentru aceasta fie: P={n! : n*x pentru orice x!A}# . Evident 0!P. Dac pentru orice n!P ar rezulta s(n)!P, atunci am deduce c P= . Astfel c alegnd un x0!A atunci x0!P, deci s(x0)!P. n particular ar rezulta c s(x0 )*x0 absurd !. Deducem c P' , adic exist a!P a.. s(a)"P. Vom demonstra c a!A $i c a este cel mai mic element al lui A. Dac a"A, atunci pentru orice x!A avem a<x, de unde s(a)*x (conform Lemei 4.2.), adic s(a)!P absurd !, deci a!A $i cum a !P deducem c a *x pentru orice x!A, adic a este cel mai mic element al lui A . & COROLAR 4.6. sta ionar. Orice !ir descresc$tor de numere naturale este

Demonstra#ie Fie (a n)n ! un $ir descresc tor de numere naturale iar

11

A={a

: n! }# . Conform Teoremei 4.5 mul #imea A are un cel mai mic

element a k ; atunci pentru orice m/k avem a m / a k $i cum a k * am deducem c am = a k , adic $irul (a n ) n ! este sta#ionar . & COROLAR 4.7. n infinit de numere naturale . nu putem g$si un !ir strict descresc$tor !i

COROLAR 4.8. Fie P# n!P. Atunci P= .

a.. pentru orice n!

(x<n $ x!P) $

Demonstra#ie Fie A= \P# $i s presupunem prin absurd c A'0. Conform Teoremei 4.5. mul #imea A va avea un cel mai mic element .&

a!A. Cum pentru x! , x<a $ x"A $ x!P, conform ipotezei P= , adic a!P $i astfel a"A absurd !. Deci A=0, de unde P=

COROLAR 4. 9. ( Teorema mp $r irii cu rest n ). Pentru oricare dou$ numere naturale m, n cu n*0, exist$ !i sunt unice dou$ numere naturale c !i r a.. m=nc+r !i r<n . Demonstra#ie Fie A={s! : exist p! a.. m=np+s}# . A'0. Conform

Deoarece m=0m+m deducem c a.. m=cn+r $i s demonstr m c r<n .

m!A, adic

Teoremei 4.5. mul#imea A posed un element minimal r!A. Atunci exist c! Dac prin absurd r 1n, atunci conform Propozi#iei 4.4., r /n, adic exist u! a.. r=n+u. Deducem c m=nc+r=nc+n+u=n(c+1)+u, adic u!A, deci r*u $i cum u*r deducem c u=r, adic n=0 absurd !. Pentru a demonstra unicitatea lui c $i r s presupunem c m=cn+r= =c'n+r', cu r, r'<n $i s ar t m c c=c' $i r=r'. S presupunem de exemplu c c<c', adic c+u=c' cu u! *. Atunci m=nc'+r'=n(c+u)+r'=nc+nu+r', deci r=nu+r ' >n absurd !. Deci c=c' $i deducem imediat c $i r=r'. & Observa#ie Num rul c poart numele de ctul mp$r#irii lui m la n iar r se zice restul acestei mp r#iri . TEOREMA 4.10. Fie m, n, m2, n2, p 12 a.. mn !i m2n2. Atunci:

i) ii) (m+m2)+(r+r2)=n+n2 deducem

m+m2 n+n2 !i mm2 nn2 mp np !i mp np. c m+m2n+n2. De asemenea deducem c

Demonstra#ie i) Putem scrie m+r=n $i m2+r2=n2, cu r, r2 . Din nn2=(m+r)(m2+r2)=mm2+mr2+rm2+rr2 $i cum mr2+rm2+rr2

mm2nn2. ii) Se deduce ca $i i) #innd cont de i) precum $i de regulile de calcul din stabilite mai nainte. &

5. Reprezentarea numerelor naturale ntr-o baz $ dat$ Din cele mai vechi timpuri s-a impus g sirea unor procedee de scriere a numerelor naturale care s permit o rapid estimare a ordinului lor de m rime, precum $i elaborarea unor reguli simple de a efectua principalele opera #ii cu acestea (adunarea, nmul#irea). Acestei probleme i s-au dat rezolv ri specifice diferitelor etape de dezvoltare a matematicilor (adaptarea sistemului de numera#ie zecimal cu care suntem obi$nui#i azi s-a ncheiat abia n secolele XVIXVII cnd acesta a cunoscut o larg r spndire n Europa). n cele ce urmeaz vom fundamenta ceea ce nseamn scrierea numerelor naturale n baza u , unde u , u2. LEMA 5.1. Fie u un num$r natural >1. Oricare ar fi num $rul natural a>0, exist$ numerele naturale n, q0, q1,, qn-1, a0, a1,, an a. .: a=uq 0+a0, 0a0<u q 0=uq1+a1, 0a1<u q n-2=uqn-1+an-1, 0an-1<u q n-1=an, 0an<u Demonstra#ie. Dac a<u, lu m n=0, a0=a $i lema este adev rat . Dac au, fie q0, a0 astfel nct a=uq0+a0, 0a0<u. Cum au, avem q0>0. Exist q1, a1 astfel nct q0=uq1+a1, 0a1<u $i a$a mai departe. Dac qi0, atunci din 1<u rezult qi<uqiuqi+ai=qi-1, de unde: a>q 0>q1>>qi-1>qi>0. Este clar c exist n astfel nct qn-10 $i qn=0. Rezult c 0<qn-1=an<u $i lema este demonstrat . & 13

LEMA 5.2. Fie u, a0, a1,,an 0.i<n !i 0<an<u. Atunci:


i =0 n n

astfel nct u>1, 0 ai<u pentru

ai u i < u n +1 .

Demonstra#ie Cum aiu-1 pentru i=0, 1,, n, atunci:


i =0

ai u i (u - 1)u i = u n+1 - 1 < u n+1 , de unde rezult lema. &


i =0

TEOREMA 5.3. Fie u un num$r natural >1. Oricare ar fi num$rul a>0, exist$ numerele naturale n, a n,an-1,,a0 unic determinate astfel nct: a=anun + an-1un-1 + + a1u + a0, unde 0<a0<u !i 0ai<u pentru orice 0in-1. Demonstra#ie Conform Lemei 5.1., exist n, q0,, qn-1 $i a0,a1,,an a..: a=uq 0+a0, 0a0<u q 0=uq1+a1, 0a1<u q n-2=uqn-1+an-1, 0an-1<u q n-1=an, 0an<u. nmul#im aceste egalit #i respectiv cu 1, u, u2,,un. Adunnd apoi termen cu termen egalit #ile ce se ob#in, rezult : a=a nun + an-1un-1 + + a1u + a0. R mne s dovedim unicitatea numerelor n, a n,,a1, a0. Fie de , a0 asemenea numerele naturale n , a a.. n , a n-1 ,..., a1 u n + a n -1u n-1 + ... + a1 u + a 0 cu 0 < a n < u $i 0 a i < u a = an 0i<n . Dac n < n , atunci n + 1 n $i din Lema 5.2. rezult : a=
n n i =0 i =0

pentru

ai u i < u n+1 u n aiu i = a , deci a<a (contradic#ie).

Analog se arat c nu este posibil ca n < n , de unde n = n . . S demonstr m acum c a i = ai , 0in. Dac n=0, atunci a 0 = a = a 0 Presupunem c n>0 $i c afirma#ia este adev rat pentru n-1. Din egalit #ile: + u (a n u n-1 + ... + a1 ) , unde 0 a 0 < u $i a = a 0 + u (a n u n -1 + ... + a1 ) = a 0 < u rezult , folosind unicitatea ctului mp r#irii lui a prin u c a 0 = a 0 0 a0
n-1 u + a1 . $i a n u n -1 + ... + a 2 u + a1 = a + ... + a 2 n u

Folosind ipoteza de induc#ie, din ultima egalitate deducem c i=1,2,,n. 14

a i = ai ,

Teorema este astfel complet demonstrat . & Suntem acum n m sur s definim ceea ce este cunoscut sub numele de sistem de numera#ie n baza u , unde u este un num r natural >1. La fiecare num r natural a>0 facem s corespund secven#a finit de numere naturale anan-1a1a0, unde ai<u, 0in, an0 $i a =
def i =0

ai u i .

A$adar, anan-1a1a0 = anun +an-1un-1 ++ a1u +a0. Din Teorema 5.3. rezult c se stabile$te astfel o coresponden# biunivoc ntre numerele naturale >0 $i secven#ele finite anan-1a1a0 de numere naturale ai<u, cu an0. Cnd se impune s atragem aten#ia asupra bazei sistemului de numera#ie, se obi$nuie$te s se scrie anan-1a1ao(u) sau anan-1a1ao(u). Dac baza sistemului de numera#ie este zece (notat 10) el este numit sistemul zecimal. Cifrele sistemului de numera#ie se numesc cifre zecimale. Ele sunt numerele mai mici ca zece $i se noteaz n ordine cu 0, 1, 2, 3, 4, 5, 6, 7, 8, 9. Secven#a de cifre zecimale 75038 sau mai precis 75038 (10) reprezint , a$adar, num rul natural: 7104 + 5103 + 0102 + 310 + 8. Dac u=2, atunci avem sistemul de numera#ie binar, cifrele binare fiind 0 $i 1. Astfel: 11010 (2)=124 + 123 + 022 + 12 + 0 =26(10). Printre sistemele de numera#ie mai des folosite se num r $i cel de baz u=16(10)=10000 (2) numit sistemul de numera #ie hexazecimal, cifrele hexazecimale fiind 0, 1, 2, 3, 4, 5, 6, 7, 8, 9, A, B, C, D, F. Astfel, avem 27(10)=1A(16)=11011(2). Iat o list de probleme care se pun n mod natural n leg tur cu reprezentarea numerelor ntr-o baz : (I) Stabilirea raportului de m rime ntre dou numere reprezentate n aceea$i baz . (II) Stabilirea unor reguli (algoritmi) de efectuare a sumei, produsului etc. a dou numere reprezentate n aceea $i baz . (III) Elaborarea unor algoritmi pentru reprezentarea unui num r ntr-o baz dat . n continuare se va ar ta cum pot fi solu#ionate aceste probleme pentru numere naturale. S ncepem cu problema (I). n teorema urm toare se d un criteriu foarte comod de a stabili raportul de m rime ntre dou numere naturale reprezentate n aceea $i baz . TEOREMA 5.4. Fie a !i b dou$ numere naturale, a=a mam-1a1a0(u) !i b=bnbn-1b1b0(u). Atunci a<b dac$ !i numai dac$ m<n !i ap<bp, unde p este cel mai mare i astfel nct a ibi. 15

Demonstra#ie Dac m<n, din Lema 5.2. rezult a < um+n un b, deci a<b. Dac m=n $i ap<bp, unde p=max{i|aibi}, atunci b-a=(bp-ap)up + (bp-1up-1 ++b0) (ap-1up-1 + + a0) > (bp-ap)up + (bp-1up-1 ++ + b0)- up up + (bp-1up-1 ++ b0)- up 0, de unde b-a>0, deci a<b. Reciproc, presupunem c a<b. Atunci mn, deoarece m>n implic b<a. Dac m<n, nu mai avem nimic de demonstrat. Dac m=n, fie p=max{i|aibi}. Avem ap<bp, ntruct ap>bp implic , conform primei p r#i a demonstra#iei, b<a. Teorema este demonstrat . & Astfel pentru numerele 125302 $i 95034 date n baza zece avem 125302>95034. La fel, pentru numerele 101101 $i 100110 date n baza doi avem 101101>100110. Referitor la problema (II) se va ar ta cum se face adunarea $i nmul#irea numerelor naturale reprezentate ntr-o baz u. n particular, dac u=10, se reg sesc cunoscutele procedee de adunare $i nmultire a numerelor naturale. Fie a $i b dou numere naturale, a=amam-1a1a0(u) , b=bnbn-1b1b0(u). Trebuie s g sim cifrele c0, c1, ale num rului a+b n baza u. Putem scrie a=a0+a1u+a2u2+ $i b=b0+b1u+b2u2+. Cum a0<u $i b0<u, rezult c a0+b0<2u, deci a0+b0=ue1+c0, 0c0<u, e1=0 sau e1=1; mai precis, avem e1=0 $i c0=a0+b0 dac a0+b0<u iar e1=1 $i c0=a0+b0-u dac ua0+b0<2u. Rezult a+b=c0+(a1+b1+e1)u+(a2+b2)u2+. Evident, a1+b1+e1<2u, de unde a1+b1+e1=ue2+c1, 0c1<u, unde e2=0 sau e2=1. Avem a+b=c0+c1u+ +( a2+b2+e2)u2+, $.a.m.d. Se deduce c cifrele c0, c1, c2, ale sumei a+b sunt c i= (ai+bi+ei) mod u, i=0, 1, 2, , unde e0=0, $i pentru i>0: ei=0 ai-1+bi-1+ei-1<u $i atunci ci-1= ai-1+bi-1+ei-1, ei=1 ai-1+bi-1+ei-1u $i atunci ci-1= ai-1+bi-1+ei-1-u. Cnd m=n, num rul a+b are: 1) m cifre dac an+bn+en<u, 2) m+1 cifre dac an+bn+enu, cifra de rang m+1 fiind n acest caz cm+1=1. Dac mn, de exemplu m>n, atunci cele de mai sus r mn adev rate lund bn+1==bm=0. Se observ c pentru a efectua a+b n baza u mai trebuie s cunoa$tem, sau s avem posibilitatea s consult m, tabla adun rii numerelor naturale <u. De exemplu, dac u=5, tabla adun rii numerelor naturale <5, cu rezultatele exprimate n baza 5, este cea din tabelul 1. n acest tabel la intersec #ia liniei num rului i cu coloana num rului j este pus i+j reprezentat n baza 5.

16

+ 0 1 2 3 4

0 0 1 2 3 4

1 1 2 3 4 10

2 2 3 4 10 11

3 3 4 10 11 12

4 4 10 11 12 13

Tabelul 1. Tabla adun rii n baza 5 Cititorul poate singur acum s redacteze un algoritm al adun rii numerelor naturale n baza u, lund ca motiva#ie teoretic a acestuia considera#iile de mai sus. Observ m c n acest algoritm apare variabila e care are valoarea ini#ial e0=0 iar valorile ei, i 1, sunt egale cu 1 cnd ai-1+ bi-1+ ei-1 / u, respectiv 0 cnd a i-1+bi-1+ei-1<u. Se spune c varibila e realizeaz transportul unit #ii de la cifrele de rang i la cele de rang i+1, i=0, 1,. n calculul cu creionul $i hrtia al sumei a dou numere naturale, opera#iile din algoritmul adun rii n baza u se sistematizeaz astfel: a mam-1 a1a0 + b mbm-1 b1b0 c m+1cmcm-1c1c0 em em-1 e1e0 ultima linie, care descrie transportul unit #ii, de regul se omite. Astfel, dac u=2, a=1011101(2), b=101101(2), atunci a+b se face dup cum urmeaz : 1011101+ 101101 10001010 1111101 deci a+b=10001010 (2). S-a folosit $i tabla adun rii numerelor naturale <2, care este: + 0 1 0 0 1 1 1 10 rezultatele fiind reprezentate n baza 2. n continuare se va ar ta c nmul#irea a dou numere naturale n baza u se reduce la urm toarele tipuri de opera #ii: 1) nmul#irea unui num r natural a cu o putere uj a bazei u; 17

2) nmul#irea unui num r natural a cu o cifr a sistemului de numera#ie (deci cu un num r natural j, 0j<u); 3) adunarea @n baza u. Fie a=amam-1a1a0(u) =amum+am-1um-1++a1u+a0. Atunci auj = amum+j + am-1um-1+j ++ a1u1+j + a0uj = amam-1a1a0 00 ... 0 1 2 3
j

(u)

$i acum este clar cum se face n baza u o nmul #ire de tipul 1). Dac i $i j sunt dou numere naturale <u, atunci ij<u2, de unde, folosind teorema mp r#irii cu rest pentru numerele naturale, avem: ij=uq(i, j)+r(i, j), 0 r(i, j)<u, 0q(i, j)<u (*) ctul q(i, j) $i restul r(i, j) mp r#irii num rului ij prin u depinznd de i $i j. Fie acum a un num r natural dat n baza u a = a m a m -1 ...a1a 0(u ) =
m m i =0

ai u i

$i j o cifr a sistemului de numera#ie de baz u, deci 0j<u. Avem: aj =


i =0

ai ju i = (uq (ai , j ) + r (ai , j ))u i = r (ai , j )u i + q(ai , j )u i +1 ,


i =0 i 0 i0

deci efectuarea produsului aj n baza u revine la a face suma n baza u a numerelor a2 $i a22 reprezentate n baza u: a2= r(a0, j) + r(a1, j)u + r(a2, j)u2+ $i a 22=q(a0, j) +q(a1, j)u2+ A$adar, s-a l murit cum se face n baza u $i o nmul#ire de tipul 2). n sfr$it, dac b = bn bn -1 ...b1b0(u ) =

b j u j , atunci ab = ab j u j ,
j =0 j =0

deci produsul ab se poate efectua f cnd suma n baza u a numerelor ab juj, j=0, 1, 2, , n. Dar abjuj = (abj)uj. A$adar abj este o opera#ie de tipul 2) $i n sfr$it (abj)uj e o opera#ie de tipul 1). Cititorul se poate convinge u$or c regula de nmul#ire a numerelor naturale n baza zece se motiveaz din punct de vedere teoretic prin considera #iile de mai sus, lund u=10. Un instrument important al nmul#irii numerelor n baza zece este tabla nmul#irii numerelor <10. Pe de alt parte, se observ c n regula de nmul#ire a numerelor n baza u trebuie s cunoa$tem numerele q(i, j) $i r(i, j), 0i, j<u,din rela#ia (*). Din rela#ia (*) rezult c q(i, j) $i r(i, j) sunt cifrele num rului ij, 0i, j<u, reprezentat n baza u [dac ij<u, avem q(i, j)=0]. A$adar, procedeul de nmul#ire expus uzeaz de tabla nmul#irii numerelor naturale <u, cu rezultatele reprezentate n baza u. 18

n tabelele 2 $i 3 sunt date tablele nmul #irii n baza u=5, respectiv u=2. 0 1 2 3 4 0 0 0 0 0 0 1 0 1 2 3 4 2 0 2 4 11 13 3 0 3 11 14 22 4 0 4 13 22 31

Tabelul 2: Tabla nmul #irii n baza 5 0 1 0 0 0 0 0 11

Tabelul 3: Tabla nmul #irii n baza 2 Pentru calculul cu creionul $i hrtia calculele pot fi sistematizate ca @n figura urm toare: a a0 u q0 u q1 a1 a2 u q2

q n-2 a n-1

u q n-1 u 0 an

S ne ocup m acum da problema (III).

19

Trebuie observat c num rul natural a ce urmeaz s fie reprezentat ntr-o baz u este dat, de regul , ntr-o baz v $i de fapt se face trecerea lui a din baza v n baza u. Se pot distinge 3 variante: 1) Trecerea lui a din baza v n baza u cu efectuarea calculelor n baza v; 2) Trecerea lui a din baza v n baza u cu efectuarea calculelor n baza u; 3)Trecerea lui a din baza v n baza u cu efectuarea calculelor ntr-o baz intermediar w; Pentru a trece pe a din baza v n baza u cu metoda 1) se reprezint mai nti u n baza v $i apoi se aplic algoritmul sistemelor de numera#ie pentru a $i u cu efectuarea calculelor n baza v. Cum n calculatoare numerele sunt, de regul , reprezentate n baza v=2, metoda 1) se aplic atunci cnd se livreaz rezultatele numerice (de regul n baza u=10), execu#ia algoritmului sistemelor de numera #ie putnd fi astfel ncredin#at calculatorului (calculele se fac n baza v=2). Aceea$i metod se aplic $i cnd se trece cu hrtia $i creionul un num r din baza v=10, ntr-o alt baz u, preferndu-se calculele n baza v=10 din motive lesne de n#eles. Pentru exemplificare, s trecem num rul a=234 dat n baza v=10 n baza u=7. Algoritmul sistemelor de numera#ie este n acest caz: 234 3 5 4 7 33 7 4 7 0

de unde a=453 (7). Pentru a trece pe a=anan-1a1a0(v) din baza v n baza u cu metoda 2) se reprezint mai nti a0, a1,,an $i v n baza u cu ajutorul algoritmului sistemelor de numera#ie. Se introduce a 0, a1, , an $i v astfel reprezenta#i n expresia a nvn + an-1vn-1 + + a1v + a0 $i se face calculul acesteia folosind algoritmului adun rii $i algoritmul nmul#irii n baza u. Se ob#ine, n final, reprezentarea lui a n calculator. Numerele date de

20

regul n baza u=2; efectuarea calculelor n baza u=2 poate fi ncredin #at calculatorului. Metoda 3) este evident o combinatie a primelor dou . Astfel, dac dorim s trecem un num r a dintr-o baz v2, ntr-o baz u2, folosind un calculator care lucreaz cu numere reprezentate n baza 2, atunci trecem pe a n baza 2 cu metoda 2) $i apoi l trecem n baza u cu metoda 1). Procednd astfel, toate calculele pot fi ncredin#ate calculatorului. Cnd v10 $i u10, iar trecerea de la baza b la baza u vrem s o facem cu creionul $i hrtia, prefer m baza intermediar w=10 pentru a putea executa toate calculele n baza 10, cu care suntem obi$nui#i. Observa#ii 1. Trecerea unui num r natural a din baza v n baza u se simplific considerabil c`nd v=ur, r num r natural >1. Metoda se justific prin faptul c un num r natural b<ur poate fi scris n mod unic sub forma b=c r-1ur-1 ++c1u+c0, 0ci<u, 0i<r. (**) De aici, rezult c pentru a reprezenta num rul a=anan-1a1a0(v)= = anvn + an-1vn-1 + + a1v + a0 n baza u, unde v=ur cu r >1, fiecare cifr ai se scrie ca n (**), anume ai=cir-1ur-1++ci1u+ci0 $i se nlocuie$te fiecare ai cu secven#a, cir-1ci1ci0, deci ob#inem secven#a cnr-1cn1cn0cn-1,r-1cn-1,1cn-1,0c01c00. nl turnd cifrele egale cu 0 de la nceputul secven#ei de mai sus se ob#ine repreprezentarea lui a n baza u. Astfel, pentru a reprenta num rul a=375(8) n baza u=2 (deci v=u3), scriem mai nti: a 0=5=122+02+11=c0222+c012+c00 a 1=7=122+12+11= c1222+c112+c10, a 3=3=022+12+11= c2222+c212+c20, a$adar secven#a de mai sus este n acest caz: 011 111 101. 2. Cnd vr = u, r>1, trecerea unui num r din baza v n baza u se face printr-o metod care urmeaz calea invers a metodei de la observa#ia 1. n acest caz, pentru a trece n baza u num rul a=anan-1a1a0(v) se separ de la dreapta la stnga grupe de cte r cifre (ultima grup avnd cel mult r cifre) $i fiecare grup va reprezenta o cifr n baza u, cu care vom nlocui grupa respectiv . Se ob#ine astfel reprezentarea lui a n baza u. Astfel, dac u=8 $i v=2, deci v3=u, num rul a=11 111 101 (2) are n baza 8 reprezentarea a=375 (8) pentru c cifrele lui a n baza 2 pot fi grupate astfel: 11 { 111 { 101 { $i grupele ob#inute reprezint n baza 2 respectiv cifrele 3, 7 $i 5 ale bazei 8. 21

3)Inconvenientul sistemului binar de numera#ie const n faptul c reprezentarea numerelor mari necesit secven#e de cifre binare exagerat de lungi. Aceasta complic mult lectura numerelor precum $i aprecierea ordinului lor de m rime. O metod de a atenua aceste inconveniente este de a folosi sisteme de numera#ie cu baze mixte. Un exemplu este sistemul de numera#ie zecimal codat n binar, rezervndu-se cte patru pozi#ii binare fiec rei cifre zecimale. Astfel, num rul a=793(10) se reprezint n sistemul zecimal codat n binar dup cum urmeaz :

0111 { 1001 { 0011 {


7 9 3

n practic se folose$te curent sistemul de numera#ie cu baz mixt . Astfel expresia: 8 ani, 3 luni, 2 s pt mni, 15 ore $i 35 minute este un model de reprezentare a timpului ntr-un sistem de numera #ie cu $ase baze. Observa#ie Acest paragraf a fost redactat n cea mai mare parte dup lucrarea [14]. CAPITOLUL 2 : INELUL NUMERELOR NTREGI 3 1 Construc ia lui 3

n vederea construirii mul#imii numerelor ntregi 3, vom prezenta la nceput Teorema lui Mal#ev de scufundare a unui monoid comutativ cu proprietatea de simplificare ntr-un grup comutativ urmnd ca prin particularizarea la cazul monoidului ( , +) s ob#inem grupul aditiv (3, +). TEOREMA 1.1. ( Mal ev ) Fie (M, ) un monoid comutativ cu proprietatea de simplificare. Atunci exist $ un grup comutativ G(M) !i un morfism injectiv de monoizi iM:M(G(M) ce verific$ urm$toarea proprietate de universalitate : Pentru orice grup comutativ G !i orice morfism de monoizi f:M (G exist$ un unic morfism de grupuri f':G(M)(G a.. diagrama M
f

iM

G(M)
f'

G 22

este comutativ$ (adic$ f'(iM =f ). Demonstra#ie


def

Pe mul#imea M'=MM definim rela#ia (x, y)4(x', y')

= xy'=yx' $i s prob m c 4 este o echivalen# pe M' compatibil cu structura de monoid a lui M' (adic 4 este o congruen# pe monoidul produs M'=MM ). n mod evident, rela#ia 4 este reflexiv xx'y'y''=x'x''yy', deci xy''= yx'' echivalen# pe M' . Fie acum (x, y), (x', y'), (a, b), (a', b')!M' a.. (x, y)4(a, b) $i (x', y')4(a', b') $i s prob m c $i (xx', yy')4(aa', bb' ). Avem deci xb=ya $i x'b'=y'a', de unde xx'bb'=yy'aa', adic (xx', yy')4(aa', bb'), adic rela#ia 4 este o congruen# pe monoidul produs M ' n care reamintim c opera#ia de compunere se define$te prin (x, y)(x', y')= =(xx',yy'). Vom considera monoidul ct G(M)=M '/4 iar pentru (x, y)!M' vom nota prin [x, y] clasa sa de echivalen# n G(M). Datorit faptului c rela#ia 4 este o congruen# pe M' deducem imediat c G(M) devine n mod canonic monoid comutativ, definind pentru [x, y], [x', y']!G(M), [x, y][x', y']=[xx', yy'] (elementul neutru al lui G(M) va fi [e, e], e fiind elementul neutru al lui M). Deoarece pentru [x, y] !G(M), [x, y][y, x]=[xy, xy]=[e, e] deducem c [y, x]=[x, y] 1 , adic G(M) este grup (comutativ). Definim iM :M(G(M) prin iM (x)=[x, e] pentru orice x!M. Pentru x, y!M avem iM (x)iM (y)=[x, e][y, e]=[xy, e]=i M (xy) adic i
M

$i simetric . Dac

(x, y)4(x', y') $i (x', y')4(x'', y'') atunci xy'=yx' $i x'y''=x''y', de unde (am simplificat prin x'y'), adic (x, y)4(x'', y''), deci rela#ia 4 este $i tranzitiv , de unde concluzia c 4 este o

este morfism de

monoizi. Dac iM (x)=iM (y), atunci [x, e]=[y, e] + xe=ye + x=y, adic iM este chiar morfism injectiv de monoizi . S ar t m acum c dubletul (G(M), iM) verific proprietatea de universalitate din enun#. Pentru aceasta fie G un grup comutativ oarecare $i f: M ( G un morfism de monoizi. Pentru [x, y] ! G(M), definim f ' ([ x, y])= =f(x)((f(y))1. Observ m c dac [x, y]=[x', y'], atunci xy'=x'y, deci
1

f(x)(f(y')=f(x')(f(y) + f(x)((f(y)) =f(x')((f(y'))-1, adic definit . 23

f' este corect

S prob m acum c f' este morfism de grupuri. Avem f'([x, y][x', y'])=f'([xx', yy'])=f (xx')[ f(yy')]-1=
=f(x)f(x')[f(y)f(y')]-1=(f(x)[f(y)]1)(

f(x')[f(y')]-1)=f'([x, y])f'([x', y']). Pentru

x!M avem (f'(iM)(x)=f'(iM (x))= f'([x, e])=f(x)[f(e)]-1=f(x), de unde concluzia c f'(iM=f . Pentru a proba unicitatea lui f' (cu proprietatea din enun #) s presupunem c mai exist un morfism de grupuri f'':G(M)(G a.. f''(iM=f. Atunci, pentru [x, y]!G(M) avem [x, y]=[x, e][e, y]=[x, e][y, e] -1, de unde f''([x, y])=f''([x, e][y, e]1)=f''(iM (x)((iM(y)-1))=f''(iM (x))((f''(iM (y)))-1= =f(x)((f(y))1=f'([x, y]), adic f''=f'. & Observa#ii 1. Dac f este un morfism injectiv de grupuri , atunci $i f' este morfism injectiv de grupuri . ntr-adev r, dac [x, y]!G(M) $i f'([x, y])=e, atunci f(x)(f(y))1 =e, deci f(x)=f(y), de unde x=y, adic [x, y]=[x, x]=e. 2. Dac pe mul#imea dubletelor (G, f) cu G grup abelian $i f:M(G morfism injectiv de monoizi definim rela#ia (G, f )*(G', f')+exist h:G(G' a.. h este morfism injectiv de grupuri $i h(f=f', atunci se verific imediat c rela#ia de mai sus este o rela#ie de ordine iar dubletul (G(M), i M ) din Teorema lui Mal#ev este cel mai mic element fa# de aceast rela#ie de ordine. DEFINI'IA 1.2. Consider$m monoidul ( , +) (ce are proprietatea de simplificare conform Propozi iei 2.3. de la Capitolul 1) !i urmnd tehnica dat$ de Teorema lui Mal ev, mul imea subiacent$ grupului aditiv (G( ), +) se noteaz$ prin 3 !i poart$ numele de mul imea numerelor ntregi . )innd cont de faptul c i : -3 , i (n)=[n, 0] pentru orice n! este prin

morfism injectiv de monoizi, vom identifica fiecare num r natural n! elementul ntreg [n, 0] astfel c 3.

va fi privit n continuare ca submul#ime a lui

Fie acum z=[m, n]!3. Dac m=n, atunci z=0. Dac m<n, atunci exist p! * a.. m+p=n (n acest caz convenim s not m p=n-m $i astfel m+(n-m)=n) iar z=[0, p]=-[p, 0] se identific cu num rul ntreg p iar dac 24

n<m, atunci exist q! * a.. n+q=m $i astfel z=[q, 0] identificndu-se cu num rul natural q. )innd cont de acestea putem scrie pe 3 sub forma 3=(- *)% - *={-n|n! *} sau 3={0 , 1 , 2 , .}. Vom nota 3* = 3 \ {0}. 2 nmul irea numerelor ntregi LEMA 2.1. Fie x, y, z, t, x', y', z', t'! a.. [x, y]=[x', y'] !i unde

[z, t]=[z', t']. Atunci [xz+yt, xt+yz]=[x'z'+y't', x't'+y'z'] . [xz+yt, xt+yz]=[x'z'+y't', x't'+y'z']+ ( xz+yt)+(x't'+y'z')=(xt+yz)+(x'z'+y't')+

Demonstra#ie Din ipotez avem x+y'=y+x' $i z+t'=z'+t astfel c

x(z-t)+y(t-z)=x'(z'-t')+y'(t'-z')+(x-y)(z-t)=(x'-y')(z'-t') ceea ce este adev rat deoarece x-y=x'-y' $i z-t=z'-t'. & Fie acum +=[x, y] $i ,=[z, t] dou numere ntregi. Definind +,=[xz+yt, xt+yz], conform Lemei 2.1. deducem c aceast defini#ie este corect . PROPOZI'IA 2.2. (3, +, ) este domeniu de integritate. Demonstra#ie

Conform celor de mai nainte (3, +) este grup

comutativ. S demonstr m acum c (3, ) este monoid comutativ iar pentru aceasta fie +=[x, y], +'=[x', y'], +''=[x'', y''] trei elemente oarecare din 3. Atunci : +(+'+'')=[x,y][x'x''+y'y'',x'y''+y'x''] =[x(x'x''+y'y'')+y(x'y''+y'x''), x(x'y''+y'x'')+y(x'x''+y'y'')] =[xx'x''+xy'y''+x'yy''+x''yy', xx'y''+xx''y'+x'x''y+yy'y''] iar (++')+''=[xx'+yy', xy'+x'y][x'', y''] =[(xx'+yy')x''+(xy'+x'y)y'', (xx'+yy')y''+(xy'+x'y)x''] =[xx'x''+xy'y''+x'yy''+x''yy', xx'y''+xx''y'+x'x''y+yy'y''] , de unde deducem c +(+'+'')=(++')+'' adic nmul#irea numerelor ntregi este asociativ . n mod evident, ++'=+'+ (deoarece nmul #irea numerelor naturale este comutativ ), adic nmul#irea numerelor ntregi este comutativ . 25

Deoarece +[1, 0]=[x, y][1, 0]=[x, y]=+, deducem c elementul neutru pentru nmul#irea numerelor ntregi este [1, 0]. S ar t m acum c nmul#irea numerelor ntregi este distributiv fa# de adunarea numerelor ntregi . ntr adev r, +(+'++'')=[x, y][x'+x'' , y'+y''] =[x (x'+x'')+y(y'+y''), x(y'+y'')+y (x'+x'')] =[xx'+xx''+yy'+yy'', xy'+xy''+yx'+yx''] iar ++'+++''=[x, y][x',y']+[x, y] [x'', y''] =[xx'+yy', xy'+yx']+[xx''+yy'', xy''+yx''] =[xx'+yy'+xx''+yy'', xy'+yx'+xy''+yx''] de unde se observ +(+'++'')=++'+++'' . Am probat pn acum c (3, +, ) este un inel comutativ unitar. Pentru a ar ta c inelul 3 nu are divizori ai lui zero, fie ++'=0=[0, 0] cu +'0. Atunci xx'+yy'=xy'+x'y, de unde (x-y)(x'-y')=0. Cum 560 (adic x-y'0) rezut c (x'-y')=0 +x'=y'+ +'=0. & 3 Rela ia de ordine natural$ de pe 3. DEFINI'IA 3.1. Pentru x, y!3 definim rela ia x.y prin x)y + y-x! . TEOREMA 3.2. Dubletul (3, )) este mul ime total ordonat $. Demonstra#ie Fie x, y, z!3 ; deoarece x-x=0! Dac x*y $i y*x atunci exist m, n! m+n=0 $i deci m=n=0, adic x=y. Dac x*y $i y*z, atunci exist m, n! ordonarea de pe 3 este total (- *)=0. & Observa#ie deducem c Din felul n care am definit rela#ia de ordine * pe 3 ={x!3 : x/0} iar - ={x!3 : x *0}. rezult a.. x+m=y $i y+n=z. Cum 3=(- *)% iar x+(m+n)=z deducem c x*z, adic ( 3, * ) este o mul#ime ordonat . Faptul c din aceea c deducem c x*x. a.. y-x=m $i x-y=n, de unde c

PROPOZI'IA 3.3. Fie x, y, z!3 a.. x)y . 26

Atunci i ) -y )-x ii ) dac $ z/0 atunci xz)yz iii ) dac $ z)0 atunci xz+yz . Demonstra#ie y *- x. i ) Din x*y deducem c y-x! ii ) Cum y-x! iii ) Cum z! (y-x)(-z)=xz-yz! $i z! $i cum (x)(-y) =y-x! rezult c

avem (y-x)z!

adic yz-xz! , deci xz*yz . $i (y-x)(-z)! iar cum

$i y-x!

deducem c

rezult c xz.yz. &

CAPITOLUL 3: CORPUL NUMERELOR RA'IONALE 7. 1 Construc ia corpului 7 al numerelor ra ionale

/i n cazul construc#iei corpului 7 al numerelor ra#ionale vom adopta

tehnica folosit n cazul construc #iei inelului 3 al numerelor ntregi. (n sensul c vom prezenta chestiunea ntr-un context mai general, urmnd ca printr-o particularizare la cazul domeniului de integritate ( 3, +, ) s ob#inem corpul 7). Fie (A, +, ) un domeniu de integritate (adic un inel unitar $i comutativ f r divizori ai lui zero) . DEFINI'IA 1.1. Numim sistem multiplicativ n A, orice

submul ime S#A a.. 0"S, 1!S, iar dac$ x, y!S atunci !i xy!S. Exemple 1. S=A*=A\{0} este un sistem multiplicativ al lui A. 2. Dac 8*A este un ideal prim, atunci S8=A\8 este de asemenea un sistem multiplicativ al lui A. 3. Dac a!A, a60, 1, atunci Sa={a k : k!3} este un sistem multiplicativ al lui A. Pentru un sistem multiplicativ S#A s consider m mul#imea AS={(a, s)|a!A, s!S} iar pe aceasta rela#ia binar definit prin (a,s)4(a',s')
def

= as'=a's. Analog ca n cazul Teoremei lui Mal#ev se demonstreaz facil c 4 27

este o echivalen# pe AS.

S not m A[S-1]=AS/4 iar pentru (a, s) !AS vom nota prin sa de echivalen# n A[S-1]. LEMA 1.2. Fie a, b, a', b'!A !i s, t, s', t'!S a.. Atunci as + sa bt + b t aa bb !i . = = ss tt ss tt Avem c at=bs $i a't'=b's'

a clasa s

a b a b = . = !i s t s t

Demonstra#ie as + sa bt + b t + = ss tt

astfel

(as'+sa')tt'=(bt'+b't)ss'+as'tt'+sa'tt'= bt'ss'+b'tss'+ats't'- bss't'=tsb's'-tsa't'+(at-bs)s't'=(b's'-a't')ts, ceea ce este adev rat (c ci at-bs=b's'-a't'=0). nmul#ind membru cu membru egalit #ile at=bs $i a't'=b's' ob#inem c aa bb ata't'=bsb's' + .& = ss tt Ca un corolar al Lemei 1.2. de mai nainte deducem c dac pentru a b at + bs a b a b ab , !A[S-1] definim + = $i = , atunci cele dou opera#ii s t s t st s t st sunt corect definite . PROPOZI'IA 1.3. ( A[S-1], +, ) este inel comutativ unitar n care a a { |a, s!S}#U(A[S-1]) iar iS:A(A[S-1] , iS(a)= pentru orice a!A este un 1 s morfism injectiv de inele ce verific $ urm$toarea proprietate de universalitate : Pentru orice inel comutativ unitar B !i orice morfism de inele f:A(B a.. f(S) #U(B), exist$ un unic morfism de inele f':A[S-1](B a.. f'(iS=f, (unde prin U(B) am notat mul imea elementelor inversabile ale lui B) . Demonstra#ie Deoarece sunt simple calcule ntr-un inel comutativ, l s m pe seama cititorului verificarea faptului c (A[S-1], +, ) este inel comutativ unitar .

28

Dac s!S, atunci elementul neutru al lui A[S-1] fa# de opera#ia de s 1 a nmul#ire este 1= = astfel c dac a, s!S, atunci !U(A[S1]) iar s s 1 a s
-1

s a s as 1 ( deoarece = = = 1 ). a s a as 1 Fie acum B un inel comutativ unitar $i f:A(B un morfism de inele cu a !A $i s!S, scriind se

pentru care f(S)#U(B). a !A[S-1], Pentru s a a 1 a s = = s 1 s 1 1


-1

= i S (a ) (i S (s ))-1 ,

a definind f = f (a ) o ( f (s ))-1 , s

verific imediat c f' are propriet #ile din enun# . & Observa#ie Din Propozi#ia 1.3. de mai nainte deducem c dac A este un domeniu de integritate $i S=A*=A\{0}, atunci A[S-1] este un corp comutativ, numit corpul total de frac #ii al lui A . DEFINI'IA 1.4. Corpul total de frac#ii al inelului (3, +, ) se noteaz prin 7 $i poart numele de corpul numerelor ra#ionale . Elementele lui 7 se mai p numesc $i frac#ii. Dac x= !7 atunci p se nume$te num$r$torul frac#iei x iar q q numitorul s u. a Deoarece i3:3(7, i3(a)= , pentru orice a!3 este n particular 1 func#ie injectiv , putem s l privim pe 3 ca o submul#ime a lui 7, adic 3#7. Prin urmare, #3#7 .

2 Rela ia de ordine natural$ de pe 7 p Fie x!7, adic x= cu p!3 iar q!3*. q Dac q<0, atunci q>0 $i cum x= num r x!7 se scrie sub forma x= p -p = putem presupune c orice q -q

p , cu q>0 (adic q! *). q 29

DEFINI'IA 2.1.

Fie x, y!7, x =

p r , y = cu q, s! *. Vom q s

defini pe 7 rela ia ) prin x)y +ps-qr )0. PROPOZI'IA 2.2. (7, ) ) este o mul ime total ordonat $ . Demonstra#ie Reflexivitatea este imediat . Pentru antisimetrie, s presupunem c x*y $i y*x. Atunci ps-qr *0 $i qr-ps *0, de unde ps-qr =0, adic ps=qr deci x=y. Pentru tranzitivitate, s mai alegem z= adic ps-qr *0 $i ur-st *0. Cum q, s, u! * deducem c (ps-qr)u*0 $i (ur-st)q*0, adic pus-qru*0 $i qru-stq *0, de unde pus-stq *0+s(pu-tq )*0, adic pu - tq *0, deci x*z . & Faptul c ordinea * de pe 7 este total rezult din aceea c ordinea natural * de pe 3 este total . Observa#ie Rela#ia de ordine * de pe 7 definit mai nainte poart . numele de ordinea natural $ de pe t cu u! * a.. x*y $i y*z, u

n continuare vom nota 7+ ={x!7 | x.0} iar prin 7+*={x!7 | x>0}.

30

CAPITOLUL 4: CORPUL NUMERELOR REALE 9 1.Inele ordonate Rela#iile de ordine de pe inelul 3 $i corpul 7 se nscriu ntr-un context mai general pe care l vom prezenta n cele ce urmeaz $i care ne va fi de folos $i pentru ordinea natural de pe mul#imea numerelor reale 9. DEFINI'IA 1.1. Dac$ A este un domeniu de integritate (adic $ un inel comutativ unitar f $r$ divizori ai lui zero), prin ordonare pe A n elegem o submul ime nevid$ P#A a.. : Ord 1: Pentru orice x!A avem n mod exclusiv x!P sau x=0 sau -x!P. Ord 2: Dac$ x, y!P atunci x+y, xy!P. n acest caz vom spune c$ inelul A este ordonat de P iar P este mul imea elementelor pozitive ale lui A. S presupunem acum c A este ordonat de P. Cum 1 '0 $i 1=12=(-1)2 deducem c 1!P (adic 1 este pozitiv). )innd cont de Ord 2 deducem inductiv c 1 + 1 + ... + 1 este pozitiv. 14 24 3
de n ori

pentru orice n! *,

Un element x!A, x'0, x"P (adic -x!P) se zice negativ . Dac x, y!A sunt negative, atunci xy este pozitiv (c ci x, -y!P iar (x)(-y)=xy!P). Analog deducem c dac x este negativ iar y este pozitiv, atunci xy este negativ $i c pentru orice x'0 din A, x2 este pozitiv. Dac A este corp, cum pentru x '0 pozitiv avem xx-1=1 deducem c $i x este pozitiv.
-1

Fie acum A'#A un subinel iar P'=P-A'. Se verific imediat c A' este ordonat de P' ( P'se va numi ordonarea indus$ de P pe A') . Mai general, fie A', A dou inele ordonate iar P ', P respectiv mul#imile elementelor pozitive din A' $i A . 31

Dac

f:A'(A este un morfism injectiv de inele, vom spune c

p$streaz$ ordinea dac pentru orice x!P' deducem c f(x)!P (echivalent cu a zice c P'#f -1(P)). Fie acum x, y!A. Definim x<y (sau y >x ) prin y-x !P. Astfel x >0 nseamn x!P iar x<0 nseamn c x!P (spunem atunci c x este negativ ). Se verific imediat c dac x, y, z!A, atunci : IN1: Dac x<y $i y<z, atunci x<z . IN2: Dac x<y $i z >0, atunci xz<yz . IN3: Dac x<y atunci x+z<y+z . IN4: Dac A este corp, x >0, y >0 $i x<y atunci y -1 < x 1. Dac x, y!A definim x*y prin x<y sau x=y. Fie acum A un domeniu de integritate ordonat de P iar K corpul s u total de frac#ii. a Dac PK={ !K| a, b>0 }, atunci P K define$te o ordonare pe K. b a ntr-adev r, dac x!K, x'0, x= atunci putem presupune c b>0 (deoarece b a -a -a x= = ). Dac a>0, atunci x!PK. Dac a>0 atunci -x= !P K . b -b b a c Nu putem avea simultan x,-x!PK c ci scriind x= $i -x= , cu a, b, b d a c deci (ad) =bc, absurd (c ci bc!P $i c, d!A $i a, b, c, d >0, atunci - = b d ad!P). Deci PK satisface Ord 1. ac Cum xy= (iar ac, bd >0) $i bd deducem c PK satisface $i Ord 2 . x+y= ad + bc (iar ad+bc, bc>0) bc

Observa#ie Aplicnd cele de mai sus lui 7 (care este corpul total de frac#ii al domeniului de integritate 3) ob#inem de fapt ceea ce am stabilit n leg tur cu ordonarea natural de pe 7 de la Capitolul 3 (evident ordonare pe 3). Fie acum A un inel ordonat. Pentru x!A definim : 32 * este o

x, dac x .0 |x| = -x, dac x <0 ( |x| poart numele de valoarea absolut $ sau modulul lui x ). LEMA 1.2. Pentru orice x !A, | x | este unicul element z!A a.. z+0 !i z =x2.
2

Demonstra#ie

S observ m c

| x | 2=x2 $i | x | .0 pentru orice x!A.

Pe de alt parte, dac a!A $i a >0 atunci exist cel mult dou elemente z!A a.. z2=a (c ci polinomul t2a!A[X] are cel mult dou r d cini). Dac w2=a, atunci w'0 $i (w)2=w2=a, deci exist cel mult un z!A pozitiv a.. z2=a $i cu aceasta lema este probat . & DEFINI'IA 1.3. Pentru a +0, definim elementul element z +0 a.. z =a (evident, dac$ un astfel de z exist$ !).
2

a ca fiind acel

Se verific acum u$or c dac pentru a, b .0, a , b exist , atunci ab exist $i ab = a b . Evident, pentru orice x!A, | x |= x 2 . LEMA 1.4. Dac$ A este un inel ordonat, atunci VA1: Pentru orice x!A, | x |/0, iar | x |>0 dac$ x*0 VA2 : Pentru orice x, y !A, | xy |=| x || y | VA3 : Pentru orice x, y !A, | x+y |.| x | +| y |. Demonstra#ie Cum VA1 $i VA2 sunt imediate, s prob m pe VA3 : | x+y |2 =(x+y)2 =x2 +2xy+y2 * | x |2 +2| xy | +| y |2=| x | 2 +2| x||y |+| y | 2= =( |x |+| y | )2 , de unde | x + y | * | x | + | y | . & Fie acum K un corp comutativ ordonat pentru care exist un morfism (injectiv) de corpuri f :7(K (deci K va fi de caracteristic 0). Se arat imediat c dac x!3, atunci 33

1 K + ... + 1 K , dac x .0 14243


de x ori

f(x) =

0 , dac x=0 - 1 K ) + ... + (- 1 K ) , dac x<0 ( 14 4 4 2444 3


de - x ori

Mai mult, dac

x!3*, cum n 7 avem x

1 = 1 deducem c x

1 1 1 1K= f (1) = f x = f (x ) f , de unde f = f (x )-1 n K. Atunci dac x x x x= m !7 avem f (x ) = n m f = n 1 f m = m n 1 f = m (n 1 K )-1 . Rezult c n

f este unic determinat ; vom identifica atunci pe 7 cu un subcorp al lui K (f se va numi scufundarea canonic m Dac x = , y = n deducem c a lui 7 n K ). m !7 (cu n, n'>0) $i x*y, atunci mn'-m'n*0, deci n

m'n-mn'.0, iar f(x)=m(n1K)-1, f(y)=m'(n'1K)-1. Din m'n-mn'.0 $i 1K.0 (m'n-mn')1K .0 +m'(n1K)-m(n'1K).0 +m'(n1K).m(n'1K), de unde m'(n'1K)-1.m(n 1K)-1+f(y) .f(x) . Ob#inem astfel urm torul rezultat : TEOREMA 1.5. Dac$ K este un corp ordonat de caracteristic $ 0, m atunci scufundarea canonic $ a lui 7 n K, f :7(K, f = m (n 1 K )-1 , n (cu n>0 ) p$streaz$ ordinea. n continuare prin K vom desemna un corp comutativ ordonat de caracteristic 0 iar un element x!3 l vom identifica cu f(x)=x1K .

34

DEFINI'IA 1.6.

Un !ir de elemente (xn)

n+0

din K se zice !ir

Cauchy dac$ pentru orice :!K, :>0, exist$ n :! m, n+n : s$ avem | xn xm |<: . Vom spune despre !irul (x n) avem | xn x |<:. Observa#ii 1.S presupunem c
n+0

a.. pentru orice m, n! ,

c$ este convergent la un element a.. pentru orice n+n


:

x!K, dac$ pentru orice :!K, :>0, exist$ n :!

s$

$irul (xn)n.0 este convergent la dou elemente

x,y!K. Atunci pentru :!K, :>0 $i n! * suficient de mare avem : | x-y | .| x-xn +xn-y | * | x-xn | +| xn y | * 2: iar cum : este oarecare deducem c | x-y |=0 ( c ci dac | x-y |'0, atunci | x-y | >0 $i am avea | x-y |< | x-y | , absurd !). Dac (x n ) n.0 este convergent la un element x!K, vom scrie x = lim x n .
n

2. Orice $ir convergent este $ir Cauchy. DEFINI'IA 1.7. Corpul ordonat K n care orice !ir Cauchy este convergent se zice complet . DEFINI'IA 1.8. Corpul ordonat K se nume !te arhimedean dac$ a.. x ) n1K .

pentru orice x!K, exist$ n!

TEOREMA 1.9. Corpul 7 al numerelor ra ionale nu este complet . Demonstra#ie ntr-adev r, s consider m $irul (xn)n.0 de numere 4 + 3x n ra#ionale dat prin x0=1 $i x n +1 = pentru orice n.0. Prin induc#ie 3 + 2xn matematic relativ la n se probeaz c xn2<2, $i c (xn) n.0 este cresc tor 2 2 - xn 2 4 + 3x n (c ci x n +1 - x n = - xn = > 0 ) iar de aici c el este $ir Cauchy. 3 + 2xn 3 + 2xn

Dac acest $ir ar avea limita l!7, atunci cu necesitate l = este complet. & 35

4 + 3l , de 3 + 2l

unde l2=2, absurd c ci l"7. Deci (xn) n/0 nu are limit n 7, adic corpul 7 nu

Pentru K corp ordonat $i S#K, prin majorant al lui S n K n#elegem un element z!K a.. x*z, pentru orice x!S. Prin marginea superioar a lui S, notat prin sup(S) n#elegem cel mai mic majorant al lui S din K (evident dac acesta exist ). TEOREMA 1.10. Fie K un corp arhimedean complet. Atunci orice submul ime nevid$ S a lui K ce admite un majorant are margine superioar $. Demonstra#ie Pentru n! , fie Tn={y!3| nx * y pentru orice x!S }. Atunci Tn este m rginit de orice element de forma nx cu x!S $i este nevid deoarece dac b este un majorant al lui S, atunci orice ntreg y a.. nb*y este n Tn (deoarece K este arhimedean) . Fie yn cel mai mic element al lui Tn .Atunci exist xn!S a.. y y 1 yn-1<nxn *yn , de unde n - < x n n . n n n yn S not m z n = $i s demonstr m c $irul (zn)n! este Cauchy. n y y y y 1 y Pentru aceasta fie m, n! a.. n m atunci m - < n m n m m m n m yn ym 1 ym 1 c ci n caz contrar, - , deci - este majorant pentru S, ceea ce n m m m n este absurd c ci xm este mai mare. y y 1 Atunci | n - m | * de unde deducem c (zn) n! este Cauchy. n m n Fie w = lim z n $i s demonstr m la nceput c w este un majorant
n

pentru S. S presupunem prin absurd c exist x!S a.. w<x . Exist atunci n! x-w a.. | zn w | * astfel c x-zn =x-w+w-zn . x-w-| w-zn | . 2 x-w x-w .x-w > 0 deci x >zn contrazicnd faptul c zn este majorant al lui 2 2 S. S demonstr m acum c w=sup S. 36

Fie u<w; atunci exist n!

suficient de mare a.. | z n xn |*

1 w-u < . 4 4

Putem alege n suficient de mare a.. | zn w |*

w-u c ci lim z n = w . n 4

Astfel, xnu =w-u+xn-zn+zn-w . w-u-| xn zn |-| zn-w | . w-u w-u w-u . w-u > 0 , deci u<xn (adic u nu este majorant 4 4 4 absurd !). & 2 Construc ia corpului 9 al numerelor reale Vom prezenta construc#ia corpului numerelor reale cu ajutorul $irurilor Cauchy de numere ra#ionale (definite mai nainte ntr-un context mai general). DEFINI'IA 2.1. Un !ir de numere ra ionale ,=(cn ) n+0 se zice !ir a.. pentru orice n+n 0, | cn |):

nul dac$ pentru orice :!7, :>0, exist$ n 0!

Dac$ -=(an)n+0 !i .=(bn)n+0 sunt dou$ !iruri de numere ra ionale, definim suma !i produsul lor prin -+.=(an+bn) n+0 !i respectiv -.=(anbn) n+0 LEMA 2.2. m$rginit. Orice !ir Cauchy -=(an ) n+0 de numere ra ionale este a.. pentru orice n . k , | an ak | *1, de unde |an | * M

Demonstra#ie Exist k! pentru orice n! . &

| an | * |ak|+1. Alegnd M=max ( | a0 |, . . ., |a k-1 |, | ak |+1) deducem c

n cele ce urmeaz prin C(7) vom nota mul#imea $irurilor Cauchy de numere ra#ionale. PROPOZI'IA 2.3. (C(7), +, ) este inel unitar comutativ.

Demonstra#ie Fie +=( xn ) n.0, ,=( yn ) n.0, 0=(0, 0, ) $i 1=(1, 1, ). S demonstr m la nceput c ++, $i +, sunt din C(7). Pentru :!7+*, exist n:', n:''! | xm-xn |< a.. pentru orice m, n . n:' s avem

$i pentru orice m, n . n:'', | ym-yn |< 37

. Alegnd n:=max (n:', n:''),

deducem c

pentru orice m, n . n:, | xm-xn |, | ym-yn |<

e
2

, astfel c = e , adic

| (xm+ym) (xn+yn) |=| (xm-xn) + (ym-yn) | * | xm-xn |+| ym-yn |<

e
2

e
2

++, !C(7). Pentru cazul produsului +, vom #ine cont de Propozi#ia 1.2. Conform acesteia, exist M1, M 2!7+* a.. | xn | * M1 $i | yn | * M2 pentru orice n! . Notnd M=max (M 1, M 2 ) $i alegnd :!7+*, exist n:', n:''! | x m xn | * | y m-yn | * a..

2M

, pentru m, n . n:' $i , pentru m, n . n:''.

e
2M

Astfel, pentru m, n / n: =max (n:', n:''), avem | xmym xnyn |=|xm(ym-yn) + yn(xm-xn) | = | xm | | ym-yn | +| yn | | xm-xn | * * M

2M

+M

2M

=:, adic $i +,!C(7).

n mod evident, - +=(-xn )n .0 !C(7) ca $i 0, 1!C(7). Deducem acum imediat c (C(7), +, ) este inel comutativ $i unitar. & n continuare, vom nota prin N(7)={( xn ) n.0 !C(7) | lim xn =0} .
n

( convenim s numim elementele lui N(7) 'iruri nule ). LEMA 2.4 N(7) este ideal al inelului C(7). Demonstra#ie Analog ca n cazul sumei din propozi#ia precedent , se demonstreaz imediat c dac +, ,!N(7), atunci +-,!N(7). Fie acum +=(an ) n.0 !C(7) $i ,=(bn) n.0 !N(7). Conform Lemei 2.2. exist M!Q+* a.. | an | * M pentru orice n! . Deoarece ,=(bn)n.0 !N(7) pentru :!Q+*, exist n:! orice n . n: s avem | bn | * a.. pentru

Atunci pentru n . n: , | an bn |=| an | | bn | * M

e
M

=:, astfel c

+,!N(7), adic N(7) este ideal al inelului comutativ C(7) . & 38

LEMA 2.5.

Fie -!C(7) a.. -"N(7), -=(an)n+0 . Atunci exist$

c!7+* !i n0! a.. pentru orice n + n0 , | a n | + c. Demonstra#ie Dac prin absurd lema nu ar fi adev rat , atunci pentru :!7+* exist o infinitate de numere naturale n1<n2<. . . a.. | a orice i .1. a.. pentru orice m, n . p s avem 2e |an am|* . Fie ni . p ; atunci pentru orice m .p, | am | * | am -an i |+| an i | * $i 3 3 e 2e pentru orice m, n.p, | an | * | an am |+| am | * + =:, adic +!N(7), 3 3 Cum + ! C( 7 ), exist p !
ni

|<

pentru

absurd ! . & TEOREMA 2.5. (C(7) / N(7) , +, ) este corp comutativ. Demonstra#ie aceea c C(7) este inel comutativ iar N(7) este ideal n C(7). Vom demonstra c exist b !C(7)/N(7) a.. a b = 1 , unde 1 =1+N(7) (reamintim c 1=(1, 1, . . . ) !C(7) ). Cum +"N(7), conform Lemei 2.4. exist :!7+* $i n0! orice n . n0 , | an | .:. n particular, deducem c pentru n . n0 , an'0. Fie ,=(bn ) n.0 cu bn= 1 dac 0 * n * n0 a n-1 dac n . n0 a.. pentru orice n . n0 , | an | .c>0 ; a.. pentru Fie acum +!C(7) a.. +"N(7) $i a =+ +N(7)!C(7) / N(7).

Faptul c C(7) / N(7) este inel comutativ rezult din

S ar t m c ,!C(7) $i c a b = 1 . Putem alege deci c!7+* $i n0! 1 1 de unde va rezulta c . c an

Pentru :!7+* exist p . n0 a.. pentru orice m, n . p s avem | a n a m | * :c2 . 39

Atunci pentru orice m, n . p avem

a - an e c2 1 1 = m 2 =e , an am am an c

adic ,!C(7). Cum +, difer de 1 numai ntr-un num r finit de termeni (eventual pentru n * n0 ) deducem c +,-1!N(7), adic a b = 1 , deci b = a C(7) / N(7) este corp . & DEFINI'IA 2.6. Mul imea C(7) / N(7) se noteaz$ prin 9 !i poart$ numele de mul imea numerelor reale. Observa#ie Deoarece se probeaz imediat c func#ia iQ:7-9, iQ(a) = (a, a,....) pentru orice a !7 este morfism de corpuri (deci n particular func#ie injectiv ) putem privi pe 7 ca subcorp al lui 9. Elementele din I=9\7 se zic numere ira#ionale. LEMA 2.7. Pentru -=(an) n+0 !C(7) este verificat $ doar una din condi iile : (1) -!N(7) (2) Exist$ c!7+* a.. pentru n suficient de mare s $ avem a n + c (3) Exist$ c!7+* a.. pentru n suficient de mare s $ avem a n ) - c Demonstra#ie Evident (2) $i (3) se exclud reciproc. S presupunem acum c +"N(7) . Conform Lemei 2.5. exist n0! dac an<0 . S presupunem acum c an>0 pentru suficient de mul#i n $i am<0 pentru suficient de mul#i m. Pentru astfel de n $i m avem anam.2c>0 ceea ce contrazice faptul c +!C(7). Deci (2) sau (3) n sens disjunctiv trebuie s aib loc . & 3 Ordonarea lui 9 Fie P={ a | +!C(7) $i verific (2) din Lema 2.7.} #9 40 $i c!7+* a.. pentru orice n . n0 , | an | . c astfel c a n . c dac an > 0 $i an * -c Corpul ( 9,+, ) poart$ numele de corpul numerelor reale.

()

-1

, adic

LEMA 3.1. P este o ordonare pe 9. Demonstra#ie Conform Lemei 2.7. deducem c P satisface Ord 1. Fie acum +=(an ) n.0 $i ,=(bn ) n.0 !C(7) a.. a , b !P. b n . c2 . Exist c1, c2!7+* $i n1, n2! a.. pentru n.n1 , an .c1 $i pentru n.n2,

Pentru n . max (n1, n2 ), an+bn . c1+c2 >0 $i anbn .c1c2 >0 astfel c ++,, +, verific (2) din Lema 2.7. ,adic a + b , a b !P, deci P satisface $i Ord 2. Observa#ii 1. Din cele de mai sus deducem c deci cu existen#a lui n0!

dac

a , b !9, +=(xn)n.0,

,=(yn)n.0, atunci a b este echivalent cu aceea c b - a !P, adic $i c!7+* a.. yn-xn .c pentru orice n .n0 .

( b - a ) !P ,

Convenim s numim ordinea de mai nainte ordonarea natural $ de pe 9. 2. Pentru a!7 convenim s not m pe i7(a) prin a , adic a = (a, a,....) . TEOREMA 3.2. arhimedeean$. Ordonarea natural $ de pe 9 (dat$ de P) este
n.0!C(7)

Demonstra#ie Conform Defini#iei 1.8., pentru +=(an) trebui s demonstr m c exist m+! a.. a ma .

va

Conform Lemei 2.2. exist M!7+* a.. an * M pentru orice n! . Alegnd m+! a.. M*m+ deducem c an*m+ pentru orice n! , adic a ma . & Urm torul rezultat este imediat. LEMA 3.3. Dac$ -=(an)n+0!C(7) !i exist$ c!7+* !i n0! Conform Teoremei 3.2., fiind a..

pentru orice n +n0, | an| ) c, atunci a c . Observa#ie dat :!9, :>0, exist :1!7+* a.. :<:1 astfel c n defini#ia limitei unui $ir din 9 nu conteaz dac : este real sau ra#ional. LEMA 3.4. Fie -=(an) n+0 !C(7). Atunci a = lim a n (adic$ orice
n

!ir Cauchy de numere ra ionale converge n 9). Demonstra#ie Fie :!7+*. Exist n0! 41

a.. pentru orice m, n / n0 ,

| am an |*:. Atunci pentru m.n0 avem | a - a m |= a - a m e =(an am)


n .0

(c ci +-am=

), adic

a = lim a n . &
n

TEOREMA 3.5. Corpul 9 este complet . Demonstra#ie Fie (xn) n.0 un $ir Cauchy de numere reale. Conform Lemei 3.4., pentru orice n ! g sim an!7 a.. | xn - a n |< 1 n

( n partea dreapt este vorba de fapt de ( n ) -1 ! ) Cum (xn ) n.0 este Cauchy, deducem c fiind dat :>0 (de exemplu :!7) exist n0! a.. pentru orice m, n . n0 s avem | xn xm |*

Fie n1! , n1/n0 a..

1 e . Atunci pentru orice m, n . n1 avem n1 3

an - am = an - xn + xn - xm + xm - am an - xn + xn - xm + xm - an

e
3

e
3

e
3

= e . Adic

(a )

n n 0

este $ir Cauchy de numere ra #ionale. Conform

Lemei 3.4. exist x = lim a n n 9. Deoarece pentru n suficient de mare | x n -x| *


n

. | xn - a n | + | a n -x | deducem c x = lim x n , adic 9 este complet. &


n

DEFINI'IA 3.6. Un corp ordonat K se zice complet ordonat dac$ orice parte nevid $ minorat$ a sa are o margine inferioar $. Observa#ie Fie K un corp complet ordonat $i A*K, A60, A majorat . Atunci A este minorat , sup A exist $i sup (A) =- inf ( A). LEMA 3.7. Dac$ x, y!7, atunci : (i) x )y +i7(x) ) i7 (y) ; (ii) x<y +i7 (x)<i7(y) ; (iii) pentru orice -!9 exist$ x, y!3 a.. iQ (x) ) - ) iQ (y) . Demonstra#ie (i) S presupunem c x*y, adic y-x.0. Cum i7(y)-i7(x)=i7(y-x) deducem c i7 (y) .i7 (x) + i7 (x) *i7 (y) .

42

Reciproc, s presupunem c iQ (x) * iQ (y), adic iQ (y-x) .0$ y-x!P, deci pentru :>0 y-x>:>0 $y.x + x*y . (ii) Rezult din injectivitatea lui iQ . (iii) Fie +!9 $i (xn) n.0!+ . Atunci (xn) n.0!C(7), deci pentru :!7+* exist n:! a.. | xn x ne |<: pentru orice n . n: sau x ne - :<xn< x ne +: pentru orice n . n: . Lund x, y!3 a.. x< x ne -: $i y xn>0 pentru orice n . n: deci ( xn ) n .0 ( x, x, . . . .) =( xn x ) n.0 !P ( y,y, . . . .) ( x n ) n.0 =( y- xn ) n.0 !P, adic iQ (x) * + * iQ (y) . LEMA 3.8. Fie -, .!9 !i (un ) n +0 , (vn ) n +0 !C(7) a.. i7 (um) ) - ) . ) i7 (vm ) pentru orice m ! !i ( um ) m +0 ( vm ) m +0 !N(7) . Atunci -=. . a.. | v m0 - u m0 |< Demonstra#ie Fie :>0. Exist m0! (xn )
n.0!+

x ne +:<y deducem c xnx>0 $i


$i

e
3

. Fie acum

$i (yn)

n.0!,

. Din condi#ia (1) deducem c i7(um)*+ , deci pentru a.. xn u m0 > -

m=m0 avem (xn u m0 )n.0!P prin urmare exist n:'! n.n:'.

pentru

Tot din (1) rezult c ,*iQ(vm) deci pentru m=m0 avem ( v m0 -yn) n.0!P, a.. v m0 yn > -

adic exist n:''! < v m0 + urmare,

e
3

, pentru orice n . n:'', de unde yn xn< 2e e 2e * + =e , 3 3 3 prin

e
3

- u m0 +

e
3

= v m0 - u m0 +

2e 3

v m0 - u m0 +

yn xn<: pentru orice n .max (n:', n :''). Dar + * ,. Atunci a.. yn-xn >-:, pentru orice n . n: '''. Atunci | xn yn |<: pentru orice n.max (n:', n:'', n:'''), de unde +=,.& TEOREMA 3.9. Corpul ( 9, ) ) este complet ordonat . 43

(ynxn) n.0!P, deci exist n:'''!

Demonstra#ie

Fie A*9 nevid

$i minorat

iar A0 mul#imea

minoran#ilor lui A. Cum A0 '0, exist ,!A0 a.. , * + pentru orice +!A. Din Lema 3.8., (iii) rezult existen#a unui z!3 a.. i7 (z) * ,, adic i7 (z)!A0 . Fie x0=max{z!3 | iQ (z)!A0} ; atunci i7(x0)!A0 $i i7(x0+1)"A0. 1 Presupunem c am ob#inut un xk!7 (k.0) a.. i7(xk )!A0 $i i7 ( xk + k )"A0 10 n n Notnd nk=max{0*n*9 | i7(xk)+ k +1 !A0} $i x k +1 = x k + kk+1 se 10 10 ob#ine, prin induc#ie, un $ir (xk ) k.0!7 a.. (1) i7 (xk )!A0 pentru orice k ! ; 1 (2) i7 ( x k + k )"A0 pentru orice k! 10 n (3) x k +1 = x k + kk+1 . 10 Din (3) $i din defini#ia lui nk rezult ;

x k +1 = x k +

nk 10 k +1

, de unde pentru

n >k ob#inem xn xk=xnxn-1+xn-1xn-2+...+xk+1xk* 9 10n 9 + 9 10n -1 + ... + 9 10k +1 = 9 1 1 1 + + ... + n 10 (k +1) 10 1-

10k +1

n -k 9 = < 10 k 1 + 1 10 110

<

10 1 = 10k +1 9 10 k

deci ( xn ) n.0!C(7). Fie += (x n )n 0 !9 $i s demonstr m c +=inf A. Pentru aceasta vom demonstra c 1 ( ;) i7( xk ) * + * i7 ( xk+ k ) pentru orice k! . 10 Din (3) deducem c x0 * x1 *. . . * xk *. . ., deci ( xn-xk ) n.0!P pentru orice k! , adic i7 ( xk ) * (x n )n 0 =+ pentru orice k! .

44

xnxk< 1 , pentru n>k, adic 10 k 1 1 x k + k - x n >0 pentru n >k, deci + * i7 x k + k pentru orice k! . 10 10 Am demonstrat mai nainte c Am ar tat astfel inegalit #ile (;). S demonstr m acum c + este minorant al lui A. S presupunem c 1 exist 1!A a.. 1<+. Atunci i7 (xk) * 1 * + * i7 x k + k pentru orice k! . 10 1 1 Dar lim x k + k - x k = lim k = 0, de unde #innd cont de Lema k 10 k 10 3.9. deducem c 1=+, absurd, deci +!A0 . S ar t m acum c + este cel mai mare minorant al lui A. Presupunem c exist ,!A0 a.. +<,. Din (3) deducem c pentru fiecare k! exist + k!A 1 a.. + k<i7 ( x k + k ). Cum , este minorant al lui A $i + k!A deducem c 10 1 ,.+ k de unde i7 (xk ) * + * , * i7( x k + k ) de unde deducem (conform Lemei 10 3.9.) c +=,, absurd !. Deci +=infA. & CAPITOLUL 5 : CORPUL NUMERELOR COMPLEXE 1 Construc ia corpului numerelor complexe < Scopul acestui paragraf este de a identifica corpul 9 al numerelor reale cu un subcorp al unui corp comutativ < n care ecua#ia x2 =-1 are solu#ie. Pentru aceasta vom considera <=99 iar pentru (x, y), (z, t)!< definim : (x, y) +(z, t)=(x+z, y+t) (x, y)(z, t ) =(xz-yt, xt+yz). TEOREMA 1.1. (<, +, ) este corp comutativ n care ecua ia x2=-1 are solu ie. 45

Demonstra#ie Faptul c (<, +) este grup abelian se probeaz imediat (elementul neutru este ( 0, 0 ), iar pentru (x, y) !<, -(x, y)=(-x, -y )). n mod evident nmul#irea este comutativ . Pentru a proba c (<*, ) este grup, fie (x, y), (z, t), (r, s)!<. Deoarece (x, y)[(z, t)(r, s)] =[(x, y)(z, t)](r, s) =(xzr-xts-yzs-ytr, xzs+xtr+yzr-yts) deducem c nmul#irea este asociativ . Cum (x, y)(1, 0)=(1, 0)(x, y) =(x, y) deducem c elementul neutru fa# de nmul#ire este (1, 0) . Fie acum (x, y)!<* (adic x'0 sau y'0). Egalitatea (x, y) (x', y')= x =(1, 0) este echivalent cu xx'-yy'=1 $i xy'+yx'=0, de unde x'= 2 $i x + y2 y'= y x . , adic (x, y) -1= 2 , - 2 2 2 x +y x +y x +y y
2 2

Cum pentru (x, y), (z, t), (r, s)!<, (x, y)[(z, t)+(r, s)] =(x, y)(z, t)+ +(x, y)(r, s)=(xz+xr-yt-ys, xt+xs+yz+yr) deducem c nmul#irea este distributiv fa# de adunare, adic (<, +, ) este corp comutativ. S not m i=(0, 1). Cum i2=(0, 1)(0, 1) =(-1, 0)=-(1, 0) deducem c ecua#ia x2=-1 are solu#ie n <. & Observa#ie Se probeaz imediat c i9:9(<, i9(x)=(x, 0) pentru orice x!9, este morfism de corpuri (deci func #ie injectiv ). n felul acesta 9 poate fi privit ca subcorp al lui <. Am construit astfel $irul de mul#imi #3#7#9#<. Deoarece pentru z=(x, y)!< putem scrie z=(x, 0)+(y, 0)(0, 1), #innd cont de identific rile anterioare deducem c z se poate scrie (formal) sub forma z=x+iy (cu i=(0, 1) iar i 2=-1). Mul#imea < poart numele de mul#imea numerelor complexe , iar (<, +, ) corpul numerelor complexe. Elementele din <\9 se zic pur imaginare . Dac z=x+iy!< cu x, y!9, atunci x se zice partea real$ a lui z iar yi partea imaginar $ a lui z ( y se nume$te coeficientul p$r#ii imaginare ). Pentru z!<, z=x+iy, definim z = x - iy (ce se va numi conjugatul lui z) $i z = x 2 + y 2 ( |z| poart numele de modulul lui z ). PROPOZI'IA 1.2. Fie z, z1, z2!<. Atunci 46

1) z!9 + z = z 2) z = z , z z = z
2

z 1 z1 3) z1 z 2 = z1 z 2 , z1 z 2 = z1 z 2 , z = 2 z2 4) z = z , z1 + z 2 z1 + z 2 , z1 z 2 = z1 z2 ,

(cu z2*0) z1 z1 = (cu z2*0). z2 z2

Demonstra#ie 1) Fie z=a+ib. Dac z!9, atunci b=0, deci z = a = z iar dac z = z atunci b =-b adic b=0, deci z!9. S mai prob m inegalitatea |z1+z2|*|z1|+|z2| (celelalte probndu-se imediat). Alegem z1=x1+iy1 $i z2=x2+iy2 cu x1, x2, y1, y2!9 $i astfel |z1+z2|*|z1|+|z2|+

( x1 + x 2 )2 + ( y1 + y 2 )2

2 2 2 2 x1 + y1 + x2 + y2 +

2 2 2 2 2 2 2 2 x1 + x2 + 2 x1 x 2 + y1 + y2 + 2 y1 y 2 x1 + y1 + x2 + y2 + 2 2 2 2 + 2 x1 + y1 x2 + y2

2 2 (x1 x 2 + y1 y 2 )2 (x12 + y12 )(x 2 + y2 ) (x1 y 2 - x 2 y1 )2 0 ceea ce este evident.

)(

Egalitate avem dac

x1 x 2 = = l cu l !9, adic z1 = l z 2 . & y1 y 2

a - b a -b

Observa#ie Asociind fiec rui num r complex z=a+ib matricea b se probeaz imediat c corpul (<,+,) este izomorf cu corpul a b a a, b !9 , opera#iile de adunare $i nmul#ire fiind cele obi$nuite

din M2 (9). 2 Teorema fundamental $ a algebrei Dac$ L !i K sunt dou$ corpuri a.. K este subcorp al lui L, spunem despre L c$ este o extindere a lui K. Reamintim un rezultat clasic din algebr :

47

LEMA 2.1. Dac$ K este un corp comutativ iar f !K[X], grad(f) +1, atunci exist$ o extindere L a lui K n care f are toate r $d$cinile. Utiliznd teorema fundamental imediat : LEMA 2.2. Fie f!K[X], cu grad(f) +1 iar K este corp comutativ. Dac$ L este o extensie a lui K n care f are toate r $d$cinile x1, xn iar g!K[X1, Xn] este un polinom simetric, atunci g(x 1, xn)!K. Teorema urm toare (ce se bazeaz pe cele dou rezultate anterioare) este cunoscut sub numele de teorema fundamental $ a algebrei (sau teorema D!Alembert-Gauss ): TEOREMA 2.3.(D2Alembert-Gauss) Orice polinom f!<[X] cu grad(f) +1 are cel pu in o r$d$cin$ n <. Demonstra#ie Fie f=a0+a1X++anXn !<[X] (an'0) $i
2n

a polinoamelor simetrice ob #inem

f = a 0 + a1 X + ... + a n X n unde pentru orice 0*i* n, a i este conjugatul lui ai . Atunci f f =


k =0

c k X k , unde c k = ai a j , 0*k*2n $i cum c k = c k


i+ j =k

pentru orice 0 .k*2n, deducem c f f !9[X]. S presupunem c teorema este demonstrat pentru polinoamele din 9[X]. Atunci exist a!< a.. f f (a ) = 0 f (a ) f (a ) = 0 f (a ) =0 sau f a = 0. Deci este suficient s presupunem c f!9[X]. Dac gradul lui f este impar, cum func#ia polinomial a lui f este continu iar la ia valori de semne contrare deducem c exist a!9 a.. f(a)=0. Fie acum n=grad(f), n=2kp, cu k! $i p impar ; facem induc#ie matematic dup k. Pentru k=0 totul rezult din cele de mai nainte (gradul lui f fiind impar n aceast situa#ie). S presupunem afirma#ia adev rat pentru toate polinoamele f!9[X] al c ror grad se divide prin 2 k-1 $i nu se divide prin 2 k. Conform Lemei 2.1. exist o extindere L a lui < n care f are toate r d cinile x1,xn. Pentru a!9 consider m elementele z ia j = x i x j + a x i + x j , 1*i<j*n

()

( )

( n num r de

2 Cn )

. 48

Considernd polinomul f a =
2 egal cu C n = k

n(n - 1) 2 p 2 p - 1 = = 2 k -1 p cu p' impar. Coeficien#ii lui fa 2 2 sunt polinoame simetrice de z ia j . Mai mult, avnd n vedere expresiile lui z ia j ,
k -1

1 i < j n

(X - z ) acesta va avea gradul


a i j

1*i<j*n, rezult c ace$ti coeficien#i, ca polinoame de x1, x2,xn sunt simetrice, deoarece orice permutare a acestora are ca efect schimbarea elementelor z ia j , 1*i<j*n ntre ele . Conform Lemei 2.2. ob #inem c fa !9[X]. Cum 2k-1| grad (fa) $i 2k =grad (fa), conform ipotezei de induc#ie rezult c fa are cel pu#in o r d cin complex . Exist deci o pereche (i, j) cu 1 *i<j*n a.. z ia j !<. F cnd pe a s parcurg mul#imea infinit 9 rezult c exist a, b!9, a'b a.. zia j $i zib j !<. Din zia j = xi x j + a xi + x j

z ia j - z ib j = (a - b ) xi + x j !<, deci xi+xj!< ; atunci xixj!<, adic xi , xj!< $i astfel teorema este demonstrat . & Observa ii 1. Din Teorema 2.3. deducem imediat c dac f!<[X], grad(f).1, atunci f are toate r d cinile n <. Acest lucru ne permite s afirm m c Teorema fundamental a algebrei exprim faptul c corpul < al numerelor complexe este algebric nchis . 2.Din Teorema 2.3. deducem imediat c precum $i cele de forma aX2+bX+c cu b2-4ac<0. n <[X] polinoamele ireductibile sunt exact polinoamele de gradul 1 iar n 9[X] sunt cele de gradul 1

$i

z ib j = xi x j + b xi + x j

rezult

49

CAPITOLUL 6: ELEMENTE DE ARITMETIC/ 1 Divizibilitate pe DEFINI'IA 1.1. Fie a, b , b0.Vom spune c$ b divide a !i vom scrie b|a, dac$ exist$ c a.. a=bc (nu definim divizibilitatea prin 0!). n acest caz vom spune c $ b este un divizor al lui a (sau c$ a este multiplu de b). n mod evident, rela#ia de divizibilitate de pe antisimetric $i tranzitiv , adic ( este reflexiv ,

, | ) este o mul#ime par#ial ordonat n care 1

este cel mai mic element (element ini#ial) iar 0 este cel mai mare element (element final). DEFINI'IA 1.2. Un num$r p , p2 se zice prim dac$ singurii s$i divizori sunt 1 !i p. Cele mai mici numere prime sunt 2, 3, 5, 7, etc. (vom demonstra mai trziu c exist o infinitate de numere prime). Reamintim c n Capitolul 1 [.4, Corolarul 4.9.] am demonstrat teorema mp r#irii cu rest n : dac a, b , b1, atunci exist $i sunt unici c, r a.. a =bc+r, iar 0r<b; num rul c numindu-se ctul mp r#irii lui b la a, iar r restul acestei mp r#irii (evident b|a dac $i numai dac r=0). TEOREMA 1.3. Fiind date dou$ numere a, b , exist$ d nota d=(a,b)) a.. d |a, d|b, iar dac$ mai avem d' a.. d' | a !i d'| (vom

b, atunci

d'| d (adic$ n mul imea par ial ordonat$ ( , |) pentru orice dou$ elemente a !i b exist$ (a, b) ). Demonstra#ie Conform teoremei mp r#irii cu rest, putem scrie a=bc1+r1, cu c1, r1 , iar 0 r1< b. Dac r1=0 atunci b|a $i n mod evident d=(a, b)=b. Dac r10, atunci conform aceleia$i teoreme de mp r#ire cu rest putem scrie b=r1c2+r2,cu c2, r2 , iar 0r2<r1. Dac r2=0, atunci d=r1. ntr-adev r, din b=r1c2 deducem c d|b, iar din a=bc1+r1 deducem c d|a . Dac mai avem d' r1=a-bc1, deducem c d' |r1=d. 50 a.. d' |a $i d' |b, atunci cum

Dac r20, atunci din nou putem scrie r1=r2c3+r3, cu 0r3<r2, $i algoritmul descris pn acum continu , ob#inndu-se un $ir descresc tor de numere naturale : r1, r2, a.. rj-2 = rj-1cj (j 3). Conform Corolarului 4.6. de la Capitolul 1, 4, $irul r1, r2, r3, este stationar. Astfel, dac pentru un anumit k, rk+1=0, atunci d=r k, pe cnd, dac rk+1=1 atunci d=1. & De exemplu : Dac a=49 $i b=35 avem : 49=135+14 (c 1=1, r1=14) 35=214+ 7 (c 2=2, r2= 7 ) 14=27 (c 3=2, r3=0 ) de unde deducem c (49, 35)=7. Dac a=187 $i b=35 avem: 187=535+12 (c 1=5, r1=12) 35=212+ 11 (c 2=2, r2= 11 ) 12=111+1 (c 3=1, r3=1 ) de unde deducem c (187, 35)=1. Observa#ii : 1. Num rul d poart numele de cel mai mare divizor comun al lui a $i b. 2. Algoritmul de g sire a celui mai mare divizor comun a dou numere naturale descris mai nainte poart numele de algoritmul lui Euclid . 3. Dac pentru a, b avem (a, b)=1, vom spune despre a $i b c sunt prime ntre ele. 4. Inductiv se arat c pentru oricare n numere naturale a1, a2, ,an (n2) exist d a.. d|ai pentru orice 1in $i dac mai avem d' a.. d'|ai pentru orice 1i n, atunci d'|d . Num rul d se noteaz prin d=(a1, a2, ,an) $i poart numele de cel mai mare divizor comun al numerelor a1, a2, ,an . 2. Divizibilate pe 3 DEFINI'IA 2.1. Dac$ a, b3, b0, vom spune c$ b divide a (vom scrie b|a ) dac$ exist$ c3 a.. a=bc ( ca !i n cazul lui nici n cazul lui 3 divizibilitatea prin 0). Evident, dac a3 atunci 1|a, -1|a $i a|0. nu vom defini,

51

Numerele prime n 3 se definesc ca fiind acele numere ntregi p cu proprietatea c p -1, 0, 1, iar singurii divizori ai lui p sunt 1, p. Evident, numerele prime din 3 sunt numerele de forma p, cu p2 num r prim n Se verific imediat c dac a, b, c3, atunci : 1) a|a (a0) 2) Dac a|b $i b|a, atunci a=b (deci n 3 rela#ia de divizibilitate nu mai este antisimetric ). 3) Dac a|b $i b|c, atunci a|c. TEOREMA 2.2. ( Teorema mp$r irii cu rest n 3 ) Dac$ a, b3 b>0, atunci exist$ c, r3 a.. a=cb+r, cu 0r<b . Demonstra#ie Fie P={a-xb / x3}; evident n P avem $i numere .

naturale. Fie r=a-cb cel mai mic num r natural din P (cu c3). (un astfel de num r exist conform Teoremei 4.5. de la Capitolul 1). Avem 0 *r<b c ci dac r=a-cb.b am 0a-(c+1)b<r, ceea ce contrazice minimalitatea lui r. & Observa#ie 1. Putem formula teorema mp r#irii cu rest din 3 $i sub forma : Dac a, b3, b0, atunci exist c, r3 a.. a=cb+r, iar 0 r<|b|. 2. Numerele c $i r cu proprietatea de mai sus poart numele de ctul, respectiv restul mp r#irii lui a la b, $i sunt unice cu proprietatea respectiv , c ci dac am mai avea c'$i r'3 a.. a=c'b+r',cu 0 r' <|b|, atunci cb+r=c'b+r' (c-c')b =r'-r, adic b|r'-r. Cum 0r, r' <|b|, dac am presupune, de exemplu, c r' > r, atunci r'-r < |b|, iar condi#ia b | r'-r implic r'-r = 0 r' = r $i cum (c-c')b=r'-r=0, deducem imediat c c=c'. DEFINI'IA 2.3. Numim ideal al inelului (3, +, ) orice submul ime nevid$ a 3 a.. i) Dac $ x, ya, atunci x-ya ii) Dac$ xa !i b3, atunci bxa . PROPOZI'IA 2.4. Fie a3 un ideal. Atunci exist$ d a.. a=d3. Demonstra#ie Dac a={0}, atunci d=0. S presupunem c a{0}. Atunci exist xa, x0. Dac x>0, atunci x! $i atunci -x
* *

, iar dac x<0, cum a este un ideal, -xa,

. 52

n concluzie aalege da*

. Conform Teoremei 4.5 de la Capitolul 1, putem


*

ca fiind cel mai mic element din a-

$i s demonstr m c a=d3.

Cum da $i a este ideal al inelului 3, incluziunea d3 a este imediat . Fie acum aa. Conform Teoremei 2.2. putem scrie a=cd+r, cu c, r 3 $i 0r<d. (c ci d
*

). Scriind r=a-cd cum a, da, deducem c r!a. Datorit minimalit #ii lui d

deducem c r=0 $i astfel a=cdd3, de unde $i incluziunea invers a d3, care ne asigur egalitatea a =d3. & PROPOZI'IA 2.5. Fie a1, a2, ,an3. Dac$ not$m prin < a1, a2, ,an > idealul generat de {a 1, a2, , an}, atunci < a1, a2, ,an >= ={k1a1++knan | | ki3, 1i n}. Demonstratie Dac not m a ={k1a1++knan | ki3, 1in}, se arat

imediat c a este ideal al lui 3 ce con#ine {a1, a2, , an}. Cum < a1, a2, , an > este cel mic ideal al lui 3 ce include {a1, a2, , an}, deducem c < a1, a2, ,an > a . Pentru incluziunea invers #inem cont de faptul c < a1, a2, , an> = I b $i fie deci b 3 un ideal a.. {a1, a2, , an } b.
{a1,..., a n}
b Z ideal b

Atunci pentru orice k1,,kn3 avem k1a1++knan b, adic a b $i cum b este oarecare, deducem c a >b =< a1, a2, , an >, de unde egalitatea dorit . & Fiind date a1, a2, , an3 prin cel mai mare divizor comun al numerelor a1,a2,,an n#elegem acel num r d3 a.. d|ai pentru orice 1 in $i n plus dac mai avem d' |ai pentru orice 1i n, atunci d'|d . Evident, dac un astfel de d exist , atunci $i d are aceea $i proprietate . Convenim s alegem pentru rolul de cel mai mare divizor comun al numerelor ntregi a1, a2, , an acel num r natural d cu propriet #ile de mai nainte $i vom nota d=(a1, a2, , an) (vezi $i 1 pentru cazul numerelor naturale). TEOREMA 2.6. Fiind date n numere ntregi a1, a2, , an (n2), dac$ not$m prin d num$rul natural a c$rui existen $ este asigurat$ de Propozi ia 2.4. pentru idealul a = < a1, a2, , an>, atunci d=( a1, a2, , an). Demonstra#ie ntr-adev r, cum fiecare ai< a1, a2, , an>=d3 deducem c aid3, adic d|ai pentru 1in . 53

a.. d= k i a i $i astfel deducem c d' |d, adic d=(a1, a2, , an). &
i =1

Fie acum d'3 a.. d'|ai pentru 1in. Cum dd3, exist k1, , kn3

COROLAR 2.7. Fiind date n numere ntregi a 1, a2, , an (n2), d=( a1, a2, , an) dac$ !i numai dac$ exist$ k1,,kn3 a.. d=k1a1++knan. 3.Teorema fundamental $ a aritmeticii Fie a3*$i p , p2, un num r prim. n mod evident, exist k p |a $i p
k k+1 k

a..

=a (altfel zis k este cel mai mare num r natural cu proprietatea p | a). Convenim s not m k=op(a) $i s -l numim ordinul sau exponentul lui p

n a. Dac a=0 vom lua op(0)=-, iar op(a)=0 p = a. PROPOZI'IA 3.1. Orice num$r natural nenul se scrie ca un produs de numere naturale prime. Demonstra#ie Fie A=mul#imea numerelor naturale nenule ce nu se scriu ca produs de numere naturale prime. Dac prin absurd propozi #ia nu ar fi adev rat , atunci A. Conform Teoremei 4.5 de la Capitolul 1 mul #imea A va con#ine un element minimal x. n particular, x >1 $i cum x nu este prim putem scrie x=mn cu 1<m, n<x. Cum m, n<x, iar x=inf(A), deducem c m, nA, deci m $i n se scriu ca produse de numere prime. Atunci $i x=mn se scrie ca produs de numere prime-absurd . Deci A= $i cu aceasta propozi #ia este demonstrat . & COROLAR 3.2. Pentru orice n3* exist$ numerele ntregi prime
k 1 p1,,pm a.. n = p1 ... p m cu k1,,km . km

Putem folosi $i nota#ia : n = (-1)

e (n)

p
p prim p2

e( p )

unde e(n){0,1}

(dup cum n este pozitiv sau negativ) iar exponen#ii e(p) sunt numere naturale nenule numai pentru un num r finit de p-uri. LEMA 3.3. Dac$ a, b, c3 a.. (a, b)=1 !i a|bc, atunci a|c. Demonstra#ie ntr-adev r, cum (a, b)=1 conform Corolarului 2.7. exist r, s3 a.. ra+sb=1, de unde c=rac+sbc. Cum a |bc deducem c a|rac+sbc=c, adic a| c . & 54

Observa#ie Dac (a, b)1, atunci lema de mai nainte nu mai este adev rat tot timpul c ci, de exemplu, 6|38=24, dar 6 =3 $i 6=8 . COROLAR 3.4.Dac$ p, a, b!3 a.. p este prim !i p|ab, atunci p|a Demonstra#ie ntr-adev r, singurii divizori ai lui p n 3 sunt 1, p. Atunci (p,b)=1 sau p |b. Dac p|b totul este n regul , iar dac (p, b)=1, atunci se aplic Lema 3.5. & Observa#ie Putem utiliza corolarul de mai nainte $i sub forma : dac p, a, b3 a.. p este prim iar p =a, p=b, atunci p=ab . COROLARUL 3.5. Presupunem c$ p, a, b3 iar p este prim. Atunci op(ab)=op(a)+op(b). p=d. Atunci ab=p a+bcd $i cum p?cd , deducem c op(ab)=a+b=op(a)+op(b). & Demonstra#ie Dac a=op(a), b=op(b), atunci a=pac $i b=pbd, cu p=c $i

sau p|b.

TEOREMA 3.6. (Teorema fundamental$ a aritmeticii) Pentru orice num$r ntreg nenul n, exist$ o descompunere a lui n factori primi n = (-1)
e (n)

p
p prim p2

e( p)

cu exponen ii e(p) n mod unic determina i de n (de

fapt e(p)=op(n)). Demonstra#ie Scrierea lui n sub forma din enun# rezult din Corolarul 3.2. S prob m acum unicitatea acestei scrieri. Aplicnd pentru un prim q, o q n ambii membrii ai egalit #ii n = (-1)
e (n)

p
p prim p2

e( p)

ob#inem : oq(n)=e(n)oq(-1)+ e(p) oq (p) .

ns oq(-1)=0 iar

oq ( p ) =

0, pentru p q 1, pentru p = q

de unde deducem c e(q)=oq(n) $i

astfel teorema este demonstrat . & COROLAR 3.7. Pentru orice n * exist$ !i sunt unice numerele prime distincte p1, p2, , pm !i numerele naturale k1, k2, , km a..
k1 km n = p1 ... p m (spunem c$ aceast$ scriere a lui n este descompunerea lui n n

factori primi) 55

COROLAR 3.8. Fie a, b, c, n x, y


*

a.. (a,b)=1 !i ab=cn. Atunci exist$


l 1 b = q1 ...q tl
t

a.. a=x !i b=y . Demonstra#ie Fie


k 1 k s a = p1 ... p s ,

descompunerea

numerelor a $i b n factori primi (deci ki1 $i lj1 pentru i=1, 2,,s $i j=1, 2,,t). Din (a,b)=1 deducem c {p1,,ps}{q1,,qt}=. Ob#inem deci c
k 1 k s l 1 c n = p1 ... p s q1 ...q tl t , egalitate ce d descompunerea lui cn n factori primi. ns , conform Teoremei 3.6., descompunerea unui num r natural n

produs de puteri de numere prime distincte este unic (abstrac#ie f cnd de ordinea factorilor).
n1 n s m1 Astfel, dac c = p1 ... p s q1 ...q tm t , atunci

nn 1 nn s nm 1 c n = p1 ... p s q1 ...q tnm t , de unde deducem c nni=ki $i nmj=lj 1i*s, 1jt.

Atunci putem considera

n1 n x = p1 ... p s

$i y = q1 1 ... qt t . &
m m

TEOREMA 3.9. (Legendre) Dac$ n! atunci exponentul lui p n n ! este dat de


k N *

iar p este un num$r prim,

n k p

Demonstra#ie n mod evident exponentul ep al lui p n n! este dat de e p = 1 k1 + 2 k 2 + .... , unde k1 este num rul numerelor luate dintre 1, 2, , n care se divid cu p dar nu cu p2, k2 este num rul numerelor luate dintre 1, 2, , n care se divid cu p2 dar nu cu p3, etc. S calcul m acum un ki . Numerele ce se divid prin pi dintre 1, 2, , n sunt 1pi, 2pi, , tipi , cu tipi.n< (ti+1)pi , deoarece dac j este luat dintre 1, n 2,..,n $i pi|j avem j=tpi $i cum 1.j.n avem 1.tpi .n. Dar t i i < t i + 1 , deci p n ti = i . p Numerele luate dintre 1, 2, , n care se divid cu p i+1 se afl toate printre numerele luate dintre 1, 2, , n care se divid cu p i.

56

Dac din numerele luate dintre 1, 2, , n care se divid cu pi (ce sunt n num r de ti ) extragem toate numerele luate dintre 1, 2, , n care se divid cu p i+1 n (ce sunt n num r de ti+1= i +1 ) ob#inem numai numerele luate dintre 1, 2,,n p care se divid cu pi dar nu se divid cu o putere mai mare a lui p (deoarece nu se divid cu pi+1). Conform celor de mai sus num rul acestora este egal cu ki=ti-ti+1. n n Avem deci e p = 1 (t1 - t 2 ) + 2 (t 2 - t 3 ) + .... = t1 + t 2 + .... = + 2 + .... . p p (aceast sum este finit deoarece va exista un k! n s = 0 pentru orice s/k+1). & p Observa#ie Dac p>n atunci ep=0. 4. Congruen e pe 3 DEFINI'IA 4.1. Fie n , n2 un num$r fixat. Vom spune c$ a, b3 sunt congruente modulo n dac$ n|a-b ; n acest caz scriem a b(n). PROPOZI'IA 4.2. Rela ia de congruen $ modulo n este o echivalen $ pe 3 compatibil$ cu opera iile de adunare !i nmul ire de pe 3 (adic$ este o congruen $ pe inelul (3, +, )). Demonstra#ie Faptul c rela#ia de congruent modulo n este o rela#ie de echivalen# pe 3 se probeaz imediat. Pentru a proba compatibilitatea acesteia cu opera#iile de adunare $i nmul#ire de pe 3, fie a, b, a', b'3 a.. ab(n) $i a'b'(n), adic a-b=kn $i a'-b'=k'n, cu k, k'3. Atunci a+a'-(b+b')=(k+k')n, adic a+a'b+b'(n) $i scriind aa'-bb'=a(a'-b')+b'(a-b)=ak'n+b'kn=(ak'+b'k)n & deducem c $i aa'bb'(n).
*

a.. pk.n<pk+1 $i atunci

COROLAR 4.3. Fie ai, bi3 a.. aibi(n) pentru orice i=1, 2,,k. Atunci:
i =1

ai bi (n) !i ai bi(n) . n particular, dac$ a, b3 a..


i =1 i =1
k

i =1

ab(n) !i k

, atunci a b (n). 57

clasa de echivalen# a lui x modulo n. Pentru x3 vom nota prin x


Deoarece resturile mp r#irii unui num r oarecare din 3 prin n sunt 0, 1,,n-1, se deduce imediat c dac not m mul#imea claselor de echivalen# modulo n prin ,1 ,..., 3n , atunci 3n = {0 n -1} , iar pentru k{0, 1,,n-1} avem + y = x

={k+nt | t3}. k

Pe mul#imea 3n se definesc opera#iile de adunare $i nmul#ire astfel: y = x y (#innd cont de Propozi#ia 4.2. deducem c acestea x + y $i x

sunt bine definite). PROPOZI'IA 4.4. (3n , +, ) este inel comutativ n care unit $ ile sale 3n | (x, n)=1} . sunt U(3n , +, )={ x Demonstra#ie Cum verificarea anumitor axiome nu ridic probleme deosebite, vom reaminti doar c elementul neutru din 3n fa# de adunare este , -x . = n - x , iar elementul neutru fa# de nmul#ire este 1 0

x y =1 n | xy-1, U(3n), atunci exist y 3n a.. x y =1 Dac x de unde deducem c (x, n)=1. Reciproc, dac x{0,1,,n-1} $i (x, n)=1, atunci, conform Corolarului -1 x =s x =1 , deci 2.7. exist r, s3 a.. rn+sx=1, de unde deducem c s xU(3n). & ,5 ,7 , 11 }. De exemplu : U(312)={ 1 Observa#ie Dac pentru un num r natural n1definim 2(1) =1 iar pentru n 2, 2(n)=num rul numerelor naturale m<n a.. (m, n)=1, atunci | U(3n) |=2(n). Euler. Func#ia 2: * definit mai sus poart numele de indicatorul lui

Observa#ie Dac n inelul 3 consider m idealul a=n3, urm rind tehnica factoriz rii unui inel (comutativ ) printr-un ideal, dac am fi construit inelul factor 3/n3 se ob#inea de fapt tot 3n . Fie acum a, b
*

COROLARUL 4.5. (3n , +, ) este corp n este prim.

, n3, n 2 $i d=(a, n). 58

are solu ie n 3n dac$ !i numai x =b PROPOZI'IA 4.6. Ecua ia a dac$ d|b ; dac$ d|b atunci ecua ia are exact d solu ii n 3n . Demonstra#ie Dac , atunci x =b 0 3n este o solu#ie a ecua#iei a x

n|ax0-b, de unde deducem c d|b (c ci d|n $i d|a). Reciproc, s presupunem c d|b. Cum d=(a, n), conform Corolarului 2.7., exist x 0 , y 0 3 a.. d = a x 0 - n y 0 . Dac c=b/d, atunci , deci x c ( x0 c) = b a( x 0 c) - n( y 0 c) = b , adic a 0

. x =b este o solu#ie a ecua#iei a S presupunem acum c

x 0 $i x1 sunt dou solu#ii ale

ecua#iei

. Atunci n|ax0-b $i n|ax1-b, de unde n|a(x1-x0). Dac not m n'=n/d $i x =b a a'=a/d, atunci (a', n')=1 $i ob#inem c n'|x1-x0, adic x1=x0+kn', cu k3. Pe de alt parte se verific imediat c cu k{0, 1,,d-1}. x =b a

x0 + kn este solu#ie a ecua#iei

Cum nu e posibil s avem x 0 + k = x 0 + k pentru k, k2{0, 1,,d-1} $i kk2 (c ci ar trebui ca n|n2(k-k) d|k-k2-absurd !), deducem c dac x 0 3n , atunci aceast ecua#ie are d solu#ii $i anume: x =b este o solu#ie a ecua#iei a x 0 , x 0 + n ,, x 0 + (d - 1)n . x . Avem d=(6, 15)=3 $i =3 Exemplu. S consider m n 315 ecua#ia 6 este o solu#ie 3|3, deci ecua#ia va avea solu#ie n 315. Cum n'=15/3=5 iar 3 $i 3 + 2 5 = 13 . n concluzie, ecua#ia particular , celelalte solu#ii vor fi 3 + 5 = 8 x are n 315 d=3 solu#ii: 3 ,8 $i 13 . & =3 6

59

are x =b COROLAR 4.7. Dac$ n este num$r prim, atunci ecua ia a solu ie unic$ 3n dac$ !i numai dac$ (a, n)=1 n = a. 5. Frac ii periodice
p q

Fiind dat frac#ia a =

!7, (cu q

), prin mp r#irea lui p la q putem

scrie pe a sub form de frac#ie zecimal$: a=a0, a1 a2, cu a0, a1, a2, (n cele ce urmeaz prin diferite exemplific ri se va deduce cu claritate modalitatea general de reprezentare a numerelor ra #ionale sub forma de frac#ii zecimale). n cele ce urmeaz vom presupune c frac#ia a este subunitar ( dac ea este supraunitar , mp r#ind pe p la q putem scrie p=cq+r, cu c3 $i 0r<q $i atunci a =
p q

=c+

r q

, astfel c

se continu

studiul lui a cu
p q

r q

care este
35 21

subunitar ; convenim n acest caz s scriem a =

r =c q . De exemplu

=12 ). 3

n cazul n care 0<a<1, a0=0 astfel c prin mp r#iri repetate vom scrie a=0,a1a2, cu ai .(dup cum se va vedea n continuare $irul a1, a2, poate fi finit sau infinit; (n cazul infinit anumite grupuri de cifre se vor repeta periodic). Iat cteva exemplific ri: 7 E1: a= 20 =0,35 E2: a=
2 3

=0,666 (se repet cifra 6; convenim s scriem a=0,(6) )


8 21

E3: a=

=0,380952380952(se repet grupul de cifre 380952 $i vom scrie

a=0,(380952)) E4: a= 1 7 =0,142857142857=0,(142857) E5: a= E6: a=


5 24 7 22

=0,208333 (se repet 3 caz n care vom scrie a=0,208(3) ) =0,31818(se repet 18 caz n care vom scrie a=0,3(18) ).

S facem acum cteva observa#ii: 1. n exemplul 1 mp$r irea se termin$ cu a doua zecimal $. 2. n exemplele 2 !i 3 mp$r irea se continu$ indefinit, grupurile de cifre 6 !i 380952 repetndu-se de o infinitate de ori. n aceste cazuri convenim s $ spunem c$ avem de a face cu frac ii periodice simple. 60

n cazul exemplului 6, frac#ia zecimal ob#inut este tot periodic , cu perioada 18, dar observ m c perioada nu ncepe imediat dup virgul (ca n exemplul 2) ci este precedat de o parte care nu se repet (cifra 3). Convenim s spunem c avem de a face cu o frac#ie periodic$ mixt$. n cele ce urmeaz$ vom proba c$ n general dac$ avem o frac ie subunitar$, atunci !irul a1, a2, este sau finit sau periodic. S urm rim exemplul 4: resturile par#iale trebuie s fie mai mici dect 7. n cazul exemplului 3 sunt posibile a priori 20 de resturi, deci dup cel mult 20 de mp r#iri par#iale trebuie s ntlnim un rest care a mai fost ob#inut $i $tim c de @ndat ce restul se repet $i cifrele ncep s se repete. n general, dac q este ctul, resturile par#iale fiind mai mici dect q, dup cel mult q mp r#iri par#iale resturile par#iale $i deci cifrele ctului ncep s se repete. Am subliniat cel mult q mp r#iri, deoarece exemplele ne arat c repetarea resturilor par #iale poate ncepe $i nainte de a fi trecut prin toate resturile posibile a priori. S adncim acum chestiunea : Observa#ia de baz este urm toarea: fiind dat frac#ia subunitar b a , pentru a g si primele n cifre ale frac#iei zecimale n care se transform ea, facem mp r#irea ntreag$ 10nb:a. 8 Exemplu. Pentru a g si primele 4 zecimale ale frac#iei 21 , facem mp r#irea. 80 000:21= 3809. 170 200 11 S consider m acum o frac#ie cu num r torul 1, de exemplu
1 21

$i s

facem mp r#irile ntregi 10:21;100:21; 1000:21, etc. Resturile acestor mp r#iri reoroduc tocmai resturile par #iale din mp r#irea 10 : 21=0,47619. 100 84 160 147 130 126 40 21 61

190 189 1 10:21=0 100:21=4 1000:21=47 10 000:21=476 10 16 13 4 ( 1) 100 000: 21=4 761 19 Pentru a $ti n ce fel se transform frac#ia
2 3

1 a

1 000 000: 21=47619 1 , trebuie deci s urm rim

resturile ob#inute prin mp r#irea lui 10, 10 , 10 ,prin a. Este o chestiune deja studiat . 1). S ncepem cu cazul a este prim cu 10 (adic a descompus n factori primi nu are nici pe 2 nici pe 5 ca factori) /tim din cele expuse mai nainte c , n acest caz, resturile ncep s se repete dup ce ntlnim restul 1, pn acolo resturile fiind toate diferite. /tim c dac 10d 1 (a), d este un divizor al lui 2(a). /tim c , dac a=paqbrg, cel mai mic exponent n, astfel ca s avem bn 1 (a) oricare ar fi b prim cu a, este c. m. m. m. c al numerelor 2(pa), 2(qb), 2(rg),(vezi Corolarul 6.2. ). Rezult c : dac a este prim cu 10, primul rest care se repet n mp r#irea 1:a este 1 (adic num rul cu care am nceput), deci frac#ia zecimal$ este periodic$ simpl$. 1 De exemplu: 21 , 21=37; 2(3)=2; 2(7)=6; c.m.m.m.c. al numerelor 2(3) $i 2(7) este 6. Frac#ia al lui 6. Dac num r torul nu este 1, ci un alt num r prim cu a, rezultatele enun#ate se men#in. De exemplu, n mp r#irea 8:21 ob #inem ca resturile mp r#irilor ntregi succesive 80:21; 8 102:21; 8103:21 Aceste resturi se pot ob#ine dac nmul#im resturile (2) cu 8 (21). 810=8017 (21); 816=1282 (21); 813=104 20 (21) 84=3211 (21); 819=1525 (21); 81=8 8 (21). Dac resturile $irului (1) sunt toate diferite ntre ele, prin nmul #irea lor cu 8 ob#inem tot resturi diferite (dac 8r1 ar fi congruent cu 8r2, atunci 8r1-8r2 0 (21); 8(r1-r2) 0 (21), 8 este prim cu 21 pentru c frac#ia a fost reductibil ; r2r1<21, deci nu putem avea 8(r 2-r1)= multiplu de 21. 8 este tot periodoc simpl , iar num rul cifrelor Rezult c frac#ia 21 perioadei este acela $i ca $i la frac#ia
a b

1 21

este periodic simpl $i perioada ei este un divizor

1 21

Fie acum cazul a=2 5 , adic a are numai factori primi ai lui 10. 62

(De exemplu, a=40=2 35 sau a=25=52, etc). n acest caz, 10 ridicat la puterea a, dac a>b, sau la puterea b, dac b>a se divide cu a (dac a=40, 10 3=2353 se divide cu 235; dac a=25, 102=2252 se divide cu 52). Rezult c , n acest caz, frac#ia zecimal rezultnd din 1 are un num r a finit de zecimale, egal cu cel mai mare dintre numerele a $i b. De exemplu : 20=225; 7:20=0,35. n general :
b 2a 5 b

=
a

b5a - b 10a

(dac a>b) sau = b5 b


10

b -a

(dac a<b), ns

mp r#irea unui num r cu 10 se face desp r#ind prin virgul a cifre. 30a=2a5bpmqn n acest caz, frac#ia Frac#ia
b5a - b p m q n ... b a

poate fi scris

b a

1 10a

5 bm

a -b

p q n ...

(dac a>b).

se transform ntr-o frac#ie periodic simpl . Dac ea este

mai mare dect 1 ceea ce se poate ntmpla din cauza nmul#irii cu 5a-b - ea se transform tot ntr-o frac#ie periodic simpl , avnd ns $i ntregi. Aceast frac#ie nmul#it cu 1a (adic mutnd virgula cu a cifre spre stnga ), ne d
10

frac#ia

b a

, care va avea ca parte neperiodic cele a cifre, iar partea periodic


b5a - b p m q n ...

aceea$i ca $i a frac#iei

.
35 11

Dac b>a proced m analog. 7 1 57 Exemplu 22 = 27 = 10 11 ; 11 Dar


1 22

= 3,1818..., deci

7 22

= 3,1818... = 0,3(18) .

1 211

1 5 = 10 11 ;

5 11

= 0,4545...... Deci

1 = 0,04545.... = 0,0( 45) partea 22

neperiodic este 0. Rezumnd cele de mai sus ob #inem: TEOREMA 5.1. Orice frac ie se transform$ ntr-o frac ie zecimal$ cu un num$r finit de zecimale sau ntr-o frac ie zecimal$ cu un num$r nfinit de zecimale, n care caz zecimalele admit o perioad $ ce se repet$. Reciproc, s vedem cum rescriem o frac#ie zecimal
p q

a (simpl ,
a0 a1 ... ak 10 k

periodic sau periodic mixt ) sub forma

cu p, q . . De

Cazul 1. Dac a=a0, a1a2an, atunci n mod evident a = exemplu: a=1,7 =


17 10 3 , a=0,3 = 10

63

Cazul 2. S presupunem acum c a= a 0 , (a1 ...a n ) . Atunci: a= a 0


an an a1 a2 + ( 10 + 10 ) + ( 10an1+1 + 10an2+ 2 + ... + 10 2 + ... + 2 n ) + ... = 10 n a 1 10 n 1 10 n a2 10 2

1 (1 + = a 0 + 10

+ +

1 10 2 n 1 10 2 n

+ ...) + + ...)
n

(1 +

1 10 n

1 10 2 n

+ ...) + +

+ ns

an 10 n

(1 +

1 1 10 1 + 10 + ... = 1-1 1 = 10 astfel c n + n 10 2 n -1


10 n 1 a= a 0 + ( 10 +

a2 10 2

+ ... +

an 10 n

10 n 10 n -1

=
99 ... 9 1 2 3
n ori

= a0 + De exemplu

a n + a n -110 +...+10 10 n -1

n -1

= a0 +
6 9

a 1a 2 ...a n

a=3,(6)=3+
2152 999

27 + 6 9

33 9

= 11 iar dac a=2,(154), 3

+154 atunci a=2+ 154 = 1998 = 999 999

Cazul 3. S presupunem c a este o frac#ie zecimal periodic mixt : a=a0, a1a2ak(ak+1ak+2ak+n). Atunci a= a0, a1a2ak +0,000(a k+1ak+2ak+n)=

a0 a1 ... ak 10 k

0 ,( a k +1 ...a k + n ) 10 k

a0 a1 ... ak 10 k

a k +1 ...a k + n 99 ... 9 00 ... 0 1 2 3 1 2 3


n ori k ori

. = .
37 9 + 2 90

37 + De exemplu dac a=3,7(2)= 10

2 90

333 + 2 90

335 90

67 18

iar dac

a=2,15(172)=

215 100

172 + 99900 =

215999 +172 99900

214957 99900

Rezumnd cele trei cazuri de mai sus ob #inem: TEOREMA 5.2. i) Dac$ a=a0,a1a2ak, atunci ii) Dac$ a = a0 , (a1...an ) , atunci a= a 0 +

a=

a0 a1 ...a k 10 k
n -1

a n +a n -110+...+10 10n -1

= a0 +

a1a 2 ...a n 99 ... 9 1 2 3


n ori

iii) Dac$ a=a0,a1ak(ak+1ak+n), atunci

a=

a 0 a 1 ...a k 10 k

a k +1 ...a k + n 99 ... 9 00 ... 0 1 2 3 1 2 3


n ori k ori

Observa#ie Acest paragraf a fost redactat n cea mai mare parte dup lucrarea [8].

64

6.Teoremele lui Euler, Fermat !i Wilson LEMA 6.1. Dac$ G este un grup (multiplicativ) finit cu n elemente (n2), atunci xn=1, pentru orice xG. Demonstra#ie Fie xG, iar k=o(x) (ordinul lui x). Atunci x k=1 $i conform Teoremei lui Lagrange k|n, adic n=kp cu p . Deducem imediat c xn=xkp=(xk)p=1p=1. & Observa#ie n cazul c G este comutativ exist o demonstra#ie elementar ce evit Teorema lui Lagrange. Pentru aceasta se alege G={x1, x2,,xn} $i xG. Cum {xx1, xx2,,xxn}=G={x1,,xn}, deducem c (xx1)(xxn)=x1xn xn(x1xn) = x1xn xn=1. & COROLAR 6.2. (Euler) Dac$ n 2 este un num$r natural iar a3 a.. (a, n)=1, atunci a j(n) 1(n) (1 fiind indicatorul lui Euler). Demonstra#ie Am v zut mai nainte c (3n , ) este un monoid cu 2(n) elemente inversabile. Astfel, dac aplic m Lema 6.1. grupului G=U(3n , ) (ce are G ob#inem c : 2(n) elemente) pentru a

aj (n) = 1 n j (n ) = 1 a

j (n )

-1 a

j (n )

1(n ). &

COROLAR 6.3. (Mica teorem$ a lui Fermat) Dac $ p2 este un num$r prim, iar a3 a.. p?a, atunci ap-1 1(p). Demonstra#ie Cum p este un num r prim, 2(p)=p-1 $i acum totul rezult din Corolarul 6.2. &
xG

LEMA 6.4. Fie G un grup (multiplicativ) finit comutativ iar produsul tuturor elementelor din G. Atunci Demonstra#ie Vom scrie produsului
xG

x= x.
xG o ( x ) 2

xG

x = ( x)( x) . ns
xG o ( x ) 2 xG o( x ) > 2

n cadrul

xG o ( x )> 2

x vom grupa fiecare element x cu x-1 (avem xx-1 c ci dac x=x-1

65

atunci x2=1 $i deci o(x)=2, absurd) $i astfel

xG o ( x )> 2 xG

x =1, de unde concluzia c

x = x. &
xG o ( x ) 2

COROLAR 6.5. (Wilson) Dac$ p2 este un num$r prim, atunci (p-1)!+1 0(p). Demonstra#ie Cum p este prim (3p*, ) este grup cu p-1 elemente $i = x . R m ne s punem n eviden# elementele conform Lemei 5.4., x
Z * x p Z * x p )2 o( x 2

x2 =1 p|x2-1=(x-1)(x+1) ) = 2 x =1 3p* cu proprietatea c o( x x p|x-1 sau p|x+1 de unde deducem c

2 ... p - 1 = -1 ( p - 1) ! +1 =0 (p-1)!+10(p). & 1 Vom prezenta n continuare diferite variante de generalizare a Teoremei lui Wilson. LEMA 6.6. Fie p2 un num$r prim, iar n2 un num$r natural. Atunci : i)

= p - 1 sau x = 1 astfel c = -1 x

,-1 , 2 n -1 + 1, n -1 1 2 -1 au ordinul cel mult 2. ,-1 au ordinul ii) Dac$ p>2 atunci n grupul U(3pn,) numai elementele 1 cel mult 2. Z* Demonstra#ie Avem c U( Z* n , )={ a pn , (a,p)=1}. S determin m n
p

Dac$

p=2

!i

n>2

grupul

U(32n,)

numai

elementele

U( Z acest grup elementele a

2 =1, adic acele numere naturale a a.. , ) a. a

1 a<pn, cu (a, p)=1 $i pn | a2-1 (*). Evident a=1 verific (*). Dac a>1, atunci putem scrie a-1=pku $i t a+1=p v cu k, t 0, (p, u)=(p, v)=1, iar k+t n.

66

Dac k=0 t n pn | a+1 $i cum a < pn a+1=pn a= pn-1 $i ce verific de asemenea (*). = p n - 1 = -1 astfel ob#inem $i elementul a Dac t=0 k >n pn |a-1 $i cum a < pn a-1=0 a=1, contradic #ie. Dac k0, t0 2=ptv-pku p|2, deci dac p2, ob#inem o contradic#ie. $i n concluzie : dac p >2, atunci n U ( * n ) avem numai elementele 1 = p n - 1 ce au ordinul cel mult 2, ob#innd astfel concluzia de la ii). -1 Dac p=2, atunci din 2=2tv-2ku t=1 sau k=1. Dac t=1 k n-1 a-1=2ku 2n-1u $i cum 1<a<2n u=2 $i k=n-1. Deci, n acest caz, dac a verific (*)a=2n-1+1. Dac k=1 t n-1 a+1=2tv 2n-1v $i cum 1<a<2n v=1 sau v=2 (cazul v=2 este exclus c ci (v, 2)=1) Dac v=1 t = n-1 sau t = n. n cazul t=n-1a=2n-1-1, iar dac t=n n a=2 -1. n concluzie : dac p=2 $i n>2 n U ( Z * ) numai elemente 2n ,1 , 2 n-1 + 1, 2 n -1 - 1 au ordinul cel mult 2, ob#innd astfel concluzia de la i). -1 & COROLAR 6.7. (O generalizare a teoremei lui Wilson) Dac $ p este un num$r prim !i n un num$r natural, atunci: a) Dac $ p>2 !i n 2 atunci pn | ( a )+1
1 a < p n ( a , p ) =1

Zp

b) Dac $ p=2 !i n >2 atunci : 2 n | ( c) Dac$ p=2 !i n=2 atunci : 22 | (

1 a < 2 n ( a , 2 ) =1 1 a < 2 2 ( a , 2 ) =1

a )-1

a )+1

Demonstra#ie Totul rezult imediat din Lema 5.4 #innd cont de cele stabilite n Lema 6.6. &

67

7.Teorema chinezeasc $ a resturilor Fie m1, m 2, ,m t iar b1, b2,,bt3. TEOREMA 7.1. (Teorema chinezeasc$ a resturilor) Sistemul x 2b1 (m1) ........ x 2bt (m t) a.. (mi, mj)=1 pentru orice ij, m=m1m2m t,

(S)

are solu ie n 3 !i oricare dou$ solu ii difer$ printr-un multiplu de m. m Demonstra#ie Dac ni= , atunci (mi, ni)=1 pentru orice 1i t. Astfel

mi
t

exist ri, si3 a.. ri mi+sini=1 pentru orice 1 i t. Dac not m ei=sini, atunci ei 1(mi) $i ei 0(mj) pentru 1 i, j t, ij. Dac vom considera x0= bi ei , atunci vom avea x0biei (mi) $i astfel
i =1

x0bi(mi) pentru orice 1 i t, de unde concluzia c x0 solu#ie a lui (S). S presupunem c x1 este o alt solu#ie a lui (S). Atunci x1-x00 (mi) pentru 1 i t, adic mi | x1-x0 pentru orice 1 it, $i cum (mi, mj)=1 pentru ij, deducem c m=m1m2m t | x0-x1, adic x0x1(m). & S interpret m acum teorema chinezeasc a resturilor din punct de vedere al teoriei inelelor. Fie pentru aceasta (Ai)iI o familie nevid de inele (unitare).Vom considera un nou inel notat Ai $i avnd mul#imea subiacent
iI

Ai ={(xi)iI|xiAi pentru orice iI}, iar pentru x,y Ai , x=(xi)iI $i


iI iI

y=(yi)iI, x+y=(xi+yi)iI, iar xy=(xiyi)iI. Se verific imediat c ( Ai , +, ) devine inel unitar n care elementul
iI

nul este 0=(xi)iI cu xi=0 pentru orice iI, iar pentru x=(xi)iIA, -x=(-xi)iI; elementul unitate este 1=(xi)iI cu xi=1 pentru orice iI, iar dac x=(xi)iI

68

! Ai , atunci xU( Ai ) dac $i numai dac xiU(Ai) pentru orice iI,


iI iI

altfel zis U( Ai )= U ( Ai ) .
iI iI

Dac I este finit not m

Ai = X Ai .
iI iI
*

Fie acum m1, m2, , m t m=m1m2mt .

a.. (mi, mj)=1 pentru orice ij, 1i, jt $i

TEOREMA 7.2. Avem urm$torul izomorfism de inele : Z m1 Z m 2 ... Z mt Z m . Demonstra#ie. Pentru fiecare 1i t fie pi: 3 Z mi morfismul surjectiv canonic de inele ce duce fiecare element x3 n clasa sa de echivalen# modulo mi. Definim f : Zm Zm1 Zm 2 ... Zm t prin f(x)=(p1(x),,pt(x)) pentru orice x3. Dac x, y3 $i f(x)=f(y) atunci xy(m) xy(mi) pentru orice 1 it (c ci (mi, mj)=1 pentru 1ijt) pi(x)=pi(y) pentru orice 1it. Deducem astfel c f este bine definit $i c func#ia f este o injec#ie. Se verific imediat c f este morfism de inele unitare . Surjectivitatea lui f rezult fie din teorema chinezeasc a resturilor, fie observnd c | Z m1 Z m 2 ... Z m t |=|Zm| = m = m1mt. Deci f este un izomorfism de inele unitare. & COROLAR 7.3. Cu nota iile de la teorema precedent $ avem urm$torul izomorfism de grupuri multiplcative : U ( Z m) U (Z m ) ... U ( Z m ) .
1 t

COROLAR 7.4. Fie 1: i) Dac$ m1, m2, ,mt 1(m1mt)=1(m1)1 (mt) = p (1-1/p)
n *

indicatorul lui Euler. a.. (mi, mj)=1 pentru ij, atunci


*

ii) Dac$ p 2 este num$r prim !i n

, atunci 1(pn) = pn - pn-1 =

69

iii) Dac$

k1 k ... pt t este descompunerea n factori primi a lui n, n = p1

atunci 1(n) = n (1-1/p1)(1-1/pt). Demonstra#ie. i) Am v zut c |U(Zm)|=2(n) pentru orice n , n 2. Dac #inem cont de Corolarul 7.3. deducem c U (Z m) = U (Z ) ... U (Z m ) = U ( Z m ) ... U (Z m )
m1
t 1 t

2(m)=2(m1)2(mt) ii) Prin calcul direct se deduce c ntre 1 $i pn exist pn - pn-1 numere naturale mai mici strict dect pn $i prime cu pn (adic cu p), de unde egalitatea 2(pn)=pn-pn-1 iii))innd cont de i) $i ii) deducem c -1 -1 j (n) = j ( p k 1)...j ( p k t ) = ( p k 1 - p k 1 )...( p k t - p k t ) =
1 t 1 1 t t

k 1 ... p k t (1 - 1 / = p1 t

p1)...(1 - 1 / pt ) = n(1 - 1 / p1)...(1 - 1 / pt ).

De exemplu , 2(12)=2(233)=12(1-1/2)(1-1/3)=12(1/2)(2/3)=4. & 8. R$d$cini primitive modulo un num$r prim


k1 k ... p s s este descompunerea n factori primi a lui n, conform n = p1 Corolarului 6.3., U ( Z n) U ( Z p k1) ... U ( Z p k s ) astfel, pentru a determina s 1

Dac

structura grupului multiplicativ U(3n) este suficient s grupurilor de forma U(3p ) cu p prim $i n .
n

studiem structura

Vom ncepe cu cazul cel mai simplu $i anume cu U(3p) cu p prim. Cum 3p este corp, U(3p)=3p*. Dac n = + X + ... + f a0 a1 an X 3p[X]. f=a0+a1X++anXn3[X], vom nota

LEMA 8.1. Fie K un corp comutativ !i fK[X] cu grad(f)=n. Atunci f are cel mult n r $d$cini distincte. Demonstra#ie Facem induc#ie matematic dup n. Cum pentru n=1 totul este clar, s presupunem c afirma#ia din enun# este adev rat pentru orice polinom din K[x] de grad n-1. Dac f nu are r d cini n K totul este clar. Dac exist ak a.. f(a)=0, atunci f(x)=q(x)(x-a) $i grad (q)=n-1. Dac b este o alt r d cin a lui f, ba, atunci 0=f(b)=(b-a)q(b) ceea ce implic q(b)=0. Cum prin ipoteza de induc#ie q are cel mult n-1 r d cini distincte, deducem c f are cel mult n r d cini distincte.3 70

COROLAR 8.2. Fie K un corp comutativ f, g K[X] a.. grad(f)=grad(g)=n. Dac $ avem n+1 elemente distincte a1,a2,,an+1, a.. f(- i )=g(- i ) pentru orice 1)i)n+1atunci f=g. Demonstra#ie Considernd h=f-g, atunci grad(h)n $i cum h are n+1 r d cini distincte a1,a2,,an+1, deducem c h=0, adic f=g. & COROLAR 8.3. Dac$ p2 este un num$r prim, atunci orice x3, avem: x p-1-1(x-1)(x-2)(x-p+1) (p). Demonstra#ie Cum p este prim, 3p este corp comutativ. Considernd ) - ( x - 1 )( x - 2 )...( x - ( p -1) ) f = ( x p -1 - 1 Z p [ X ] avem c

pentru x ,2 ,..., p -1 (#innd cont $i de mica teorem a lui Fermat, )= 0 =1 f( x adic de Corolarul 6.3.). Conform Corolarului 8.2., f=0. & Observa#ie Dac n corolarul 8.3. consider m x=0 ob#inem c (p-1)!-1(p), adic teorema lui Wilson (Corolarul 6.5.). PROPOZI'IA 8.4. Fie p2 un num$r prim !i d|p-1. Atunci congruen a xd1(p) are exact d solu ii. Demonstra#ie Dac p-1=dd', atunci:
d x - 1 = ( x ) - 1 = ( d )d -1 + ( d )d -2 + ... + d + 1 =g(x), adic x x x d d x -1 x -1

grad(f)p-2 $i

p -1

= (xd - 1 )g ( x ) . xp-1-1=(xd-1)g(x) $i astfel x p -1 - 1 Cum

p -1

,2 ,..., are exact p-1 r d cini ($i anume 1 -1

p -1 -conform
d are x -1

micii teoreme a lui Fermat ), #innd cont de Lema 8.1. deducem c

exact d r d cini n 3p $i astfel congruen#a xd 1(p) are exact d solu #ii n 3p.3 TEOREMA 8.5. Dac$ p este un num$r prim, atunci U(3p) este un grup ciclic.

71

Demonstra#ie Solu#ia 1: Evident | U( 3p ) | = | 3p* | = p-1 iar pentru d | p-1, fie y(d) num rul elementelor din 3p* de ordin d. Conform Propozi #iei 8.4. elementele din 3p* ce satisfac congruen#a xd1(p) formeaz un grup de ordin d. ns y (c) = d , de unde se deduce c y(d) = 2(d) (2 fiind indicatorul lui Euler). n
cd

particular, y(p-1) = 2(p-1)>1 (dac p 3). Deducem c n 3p*, 2(p-1) elemente au ordinul p-1 $i astfel oricare dintre ace$tia genereaz pe 3p*, adic 3p* este grup multiplicativ ciclic. Solu#ia 2: Fie p - 1 = q1 1 q 2 l 2 ... qt lui p-1 $i s consider m congruen#ele: (1) ( 2)
qi x i 1( p) qi x i 1( p)
l l -1

lt

descompunerea n factori primi a

cu 1 i t

n mod evident orice solu#ie a congruentei (1) este solu#ie $i a congruen#ei (2). Mai mult, congruen#a (2) are mai multe solu#ii dect congruen#a (1). Pentru fiecare 1 i t fie gi o solu#ie a congruentei (2) ce nu este solu#ie a congruen#ei (1) iar g = g1g2gt. Evident, qi genereaza un subgrup al lui 3p* de ordin >=3p* . Atunci < g &

q i l i , 1 i t .

genereaza un subgrup al lui 3p* de ordin p - 1 = q1 l1 q 2 l 2 ... qt l t . Deducem ca g

DEFINI'IA 8.6. Fie p2 un numar prim. Un element aZ se zice genereaz$ 3p*. r!d!cin! primitiv! modulo p dac$ a De exemplu, 2 este r d cin primitiv modulo 5 (se verific imediat c 4=5-1 este cel mai mic num r natural n pentru care 2n 1(5)), pe cnd 2 nu este r d cin primitiv modulo 7 (de ex. 2 3 1(7) ). No#iunea de r d cin primitiv se poate generaliza astfel: DEFINI'IA 8.7. Fie n . Un element a3 se zice r!d!cin! n 3n genereaz$ U(3n). (echivalent cu a spune c $ primitiv! modulo n dac$ a 1(n) este cel mai mic num$r natural pentru care a j(n) 1(n) ). Observa#ie n general nu rezult c U(3n)este ciclic. 72

De
2 2

exemplu,
2

elementele
2

lui

U(38)

sunt

, 3 , 5 , 7 1

iar

, = 1 , = 1 , = 1 neexistnd deci n U(38) elemente de ordin =1 3 5 7 1 4=2(8). Rezult c nu orice ntreg posed r d cini primitive. LEMA 8.8. Dac$ p este un num$r natural prim !i 1k<p atunci p

k p.

Demonstra#ie

Avem

Ck p=

p! k! ( p - k )!

$i

cum

p! ( p - 1)! = p iar p nu divide nici pe k! $i nici pe (p-k)!, deducem c k!( p - k )! k!( p - k )! ( p - 1)! k dac not m q = , atunci q $i cum C k C p .3 p = p q ,atunci p k!( p - k )! Observa#ie Utiliznd Lema 8.8. putem prezenta o nou demonstra#ie a micii teoreme a lui Fermat : Dac p este un num r prim $i a3 a.. p? a, atunci p|ap-1-1. ntr-adev r, s not m sa=ap-a. Cum sa+1 = (a+1)p-(a+1)=
-1 p = a p + C1p a p -1 + ... + C p p a + 1 - ( a + 1) = (a - a ) + p -1 k =1

C kp a p - k =

= sa +

p -1 k =1

C kp a p - k

)innd cont de Lema 8.8. deducem c sa+1sa(p). Astfel sasa-1sa-2s1(p) $i cum s1=1a-1=0 deducem c sa0(p), adic p|ap-a =a(ap-1-1) $i cum p?a ob#inem c p | ap-1-1. LEMA 8.9. Dac$ n 1 este un num$r natural, p2 un num$r prim !i a, b3 a.. ab( pn ), atunci ap bp( pn+1 ). Demonstra#ie Putem scrie a=b+cpn, cu c3. Atunci ap=(b+cpn)p =bp+ C1p bp-1c pn+x (cu x3 $i pn+2 | x) astfel c

ap

= bp+bp-1 c pn+1+x, de unde ap bp(pn+1).3 COROLAR 8.10. Dac$ p este un num$r prim, p3, n , n2, atunci

(1+ ap)

pn-2

1+ a

n -1

( p ) pentru orice a3. 73

Demonstra#ie Facem induc#ie dup n, pentru n=2 afirma#ia fiind trivial S presupunem acum c afirma#ia din enun# este adev rat pentru n $i s ar t m c este adev rat pentru n+1. Conform Lemei 8.9. avem: Dezvoltnd cu

(1+ ap)

p n -1

(1+ a p n -1) ( p lui

n +1

). ob#inem

ajutorul

binomului

Newton

(1+ a p ) = 1 + C

n -1 p

1 pa

n -1

+ b , unde b este o sum de p-2 termeni. Utiliznd

din nou Lema 7.9. se verific imediat c to#i termenii lui b sunt divizibili prin p1+2(n-1), exceptnd eventual ultimul termen appp(n-1). Cum n2, 1+2(n-1)n+1 $i cum p(n-1) n+1, adic c.c.t.d.3 Observa#ie Fie a, n3 a.. (a, n)=1. Vom spune c a are ordinul k modulo n dac este cel mai mic num r natural pentru care ak1(n). Acest lucru din 3n are ordinul k n grupul U(3n). este echivalent cu a spune c a COROLARUL 8.11. Dac$ p 2 este un num$r prim a.. p?a, atunci ordinul lui 1+ap modulo pn este egal cu pn-1 (n , n 2). Demonstra#ie Conform Corolarului 8.10., (1+ ap) de unde deducem c
pn-2

pn+1|b $i astfel

(1+ ap)

p n- 2

1+ a

n n +1 p ( p ) adic

1+ a

n n +1 p (p ),

(1+ ap)

pn-2

1+ a

n -1

( p ) adic pn-2 nu este de ordinul

lui 1+ap, rezultnd astfel c ordinul lui 1+ap modulo p n este egal cu pn-1.3 TEOREMA 8.12. Fie p3 un num$r prim !i n *. Atunci U(3pn) este grup ciclic (adic$ exist$ n acest grup r$d$cini primitive modulo pn). Demonstra#ie Conform Teoremei 8.5. exist o r d cin primitiv modulo p. Dac g3 este o astfel de r d cin , atunci n mod evident $i g+p este. Dac gp-1 0(p2), atunci (g+p)p-1 gp-1+(p-1)gp-2p 1+(p-1)gp-2p (p2). Cum p2 nu divide (p-1)gp-2 p putem presupune pentru nceput c g este o r d cinin primitiv modulo p $i c gp-1 @1(p2). S ar t m c un astfel de g poate fi r d cin primitiv modulo pn iar pentru aceasta este suficient s demonstr m c dac gm 1(pn), atunci 2(pn)|m. Avem c gp-1=1+ap, unde p ?a. Conform Corolarului 8.11., pm-1 este de ordinul lui 1+ap modulo p m. Deoarece (1+ap) m 1(pn) atunci pn-1 | m; Fie m=pn-1 m2. Atunci
m g 1( p) . Deoarece g este o r d cin primitiv modulo

p, p-1|m2 $i astfel pn-1(p-1)=2(pn)|m.3 74

Pentru cazul p=2 vom demonstra: TEOREMA 8.13. Num$rul 2n are r$d$cini primitive pentru n=1 sau 2 iar pentru n3 nu are. Dac$ n3, atunci {(-1)a5b|a=0, 1 !i 0b<2n-2} constituie un sistem redus de resturi modulo 2 n. Rezult$ c$ pentru n3 U(32n) este produsul direct a dou$ grupuri ciclice (unul de ordin 2 iar cel$lalt de ordin 2 n-2). Demonstra#ie Num rul 1 este r d cin primitiv modulo 2 iar 3 este r d cin primitiv modulo 22=4, deci putem presupune n 3. Inten#ion m s demonstr m c : (1)
n -1 n 52 1 + 2 (2 )
n -1

Evident, pentru n=3 (1) este adev rat . S presupunem c (1) este adev rat pentru n $i s demonstr m pentru n+1. n 3. Aplicnd Lema 8.9. congruen#ei (1) ob#inem (2) astfel (1) este probat prin induc#ie. Din (2) se vede c
n n 52 1 (2 ) pe cnd din (1) avem c 52 @ 1(2 ) .
n- 2 n -3

La nceput s not m c : (1+2n-1)2=1+2n+22n-2 $i c 2n-2 n+1 pentru


n n +1 52 1 + 2 (2 ) $i
n -1

Atunci 5 are ordinul 2 n-2 modulo 2 n. S consider m mul#imea {(-1)a5b|a=0,1 $i 0b<2n-2} format din 2n-1 numere $i s prob m c acestea nu sunt congruente modulo 2n (deoarece 2(2n)=2n-1 deducem c mul#imea de mai sus con#ine un sistem redus de resturi modulo 2n ). Dac prin
a

absurd

a a (-1) 5b (-1) 5b (2n) ,

n3,

atunci

(-1) (-1) 5
b - b

(4) , adic aa' ( 2 ), deci a=a'. Atunci

b b n 5 5 (2 ) $i astfel

1 (2n) , de unde bb'( 2n ), deci b=b'. n final s not m c (-1)a 5b ridicat la puterea 2n-2 este congruent cu 1

modulo 2n, astfel c 2n nu are r d cini primitive modulo 2 n, dac n 3. & Din Teoremele 8.12. $i 8.13. deducem urm toarea descriere complet a grupurilor U( 3n ) pentru n arbitrar: TEOREMA 8.14. Fie n = 2a
a1 a ... p nn descompunerea lui n n p1

factori primi distinc i. Atunci: U ( Z n ) U ( Z 2 a ) U ( Z p1 a1) ... U ( Z p a n ). n 75

Grupurile U ( Z piai ) sunt grupuri ciclice de ordin

-1 pia i ( pi - 1) , 1in

iar U(32n) este grup ciclic de ordin 1 $i 2 pentru a=1, respectiv a=2. Dac a3, atunci U( 32a ) este produsul direct a dou grupuri ciclice de ordine 2 $i respectiv 2n-2. Putem acum r spunde la ntrebarea: care numere ntregi posed r d cini primitive? TEOREMA 8.15. Num$rul n
a

posed$ r$d$cini primitive dac$ !i


a

numai dac$ n este de forma 2, 4, p sau 2p cu a iar p3 un num$r prim. Demonstra#ie Conform Teoremei 8.13., putem presupune c n 2k cu k 3. Dac n nu este de forma din enunt, este u$or de a vedea c n se poate atunci scrie ca produs m1m2 cu (m1, m2)=1 $i m1, m2 >2. Atunci 2(m1) $i 2(m2) sunt simultan pare iar U ( Z n ) U ( Z m1 ) U ( Z m 2 ) . ns

U(Zm1 ) $i U ( Z m 2 ) au elemente de

ordin 2 iar acest lucru ne arat c U(3n ) nu este ciclic. (deoarece con #ine cel mult un element de ordin 2). Atunci n nu posed r d cini primitive. Reciproc, am v zut c 2, 4, $i pa posed r d cini primitive. Deoarece U ( Z 2 p a ) U ( Z 2) U ( Z p a ) deducem c U (Z 2 pa) este ciclic, adic 2pa posed r d cini primitive $i cu aceasta teorema este demonstrat .3 CAPITOLUL 7: MUL'IMEA NUMERELOR PRIME 1 Teoreme referitoare la infinitatea numerelor prime Reamintim c un num r n! , n.2 se zice prim dac singurii s i divizori naturali sunt 1 $i n. Num rul natural 2 este singurul num r prim par iar pentru n.3 dac n este prim atunci cu necesitate n este impar (condi#ie insuficient dup cum se poate dovedi facil n cazul lui 9 care este impar dar nu este prim). S-a pus de foarte mult timp ntrebarea cte numere prime exist ? n cadrul acestui paragraf vom prezenta anumite rezultate ce r spund ntr-un fel la aceast ntrebare. Vom nota prin P mul#imea numerelor prime. 76

TEOREMA 1.1.( Euclid ) Mul imea P este infinit$. Demonstra#ie S presupunem prin absurd c mul#imea P este finit , P={p1, p2, pn } (unde n mod evident p1=2, p2=3, p3=5, etc.). Vom considera p=p1p2pn +1 $i s observ m c p >1 iar pi?p pentru 1*i*n. )innd cont de teorema fundamental a aritmeticii (teorema de la cap.6), va exista un num r prim q >1 care s divid pe p. Cum toate numerele prime sunt presupuse a fi doar p 1,, pn deducem c q=pi cu 1*i*n, ceea ce este absurd c ci pi?p pentru orice 1*i*n. Deci P este mul#ime infinit . & Observa#ie n continuare pentru fiecare num r natural n.1 vom nota prin pn al n-ulea num r prim, astfel c P={p1, p2, ,pn,} (evident p1=2, p2=3, p3=5, etc). O alt ntrebare fireasc legat de mul#imea numerelor prime a fost dac anumite submul#imi infinite ale lui con#in sau nu o infinitate de numere prime. n acest sens merit amintit un rezultat celebru al lui Dirichlet : TEOREMA 1.2. (Dirichlet) Dac$ a, b! * iar (a, b)=1, atunci mul imea {an+b | n! } con ine o infinitate de numere prime. n cadrul acestei lucr ri nu vom prezenta o demonstra #ie a Teoremei 1.2. (cititorul poate consulta n acest sens lucr rile [21] $i [23] ). Totu$i pentru anumite valori particulare ale lui a $i b vom prezenta n cadrul acestei lucr ri demonstra#ii complete. Iat la nceput dou exemple: TEOREMA 1.3. Exist$ o infinitate de numere prime de forma 4n-1 cu n! *. Demonstra#ie S presupunem prin reducere la absurd c mul#imea

{4n-1| n! *} con#ine numai un num r finit de numere prime, fie acestea q 1,, q t $i s consider m num rul q=4q1q2q t 1. Num rul q trebuie s aib un factor prim de forma 4k-1 (c ci dac to#i factorii primi ai lui q ar fi de forma 4k+1 atunci $i q ar trebui s fie de forma 4k+1. Deci ar trebui ca q i s divid pe q, ceea ce este absurd.), de unde concluzia din enun#. &

77

TEOREMA 1.4. Exist$ o infinitate de numere prime de forma 6n-1, Demonstra#ie S presupunem prin absurd c exist doar un num r finit de numere prime de forma 6n-1 $i anume q1, q2,,qk. S consider m num rul q=6q1q2qk -1. Cum un num r prim este de forma 6t-1 sau 6t+1, deducem c q trebuie s con#in un factor prim de forma 6t-1 (c ci n caz contrar ar trebui ca q s fie de forma 6k+1. Deci ar trebui ca un q i s divid pe q, ceea ce este absurd.), de unde concluzia din enun#. & 2. Ciurul lui Eratostene Fiind dat un num r natural n.2, pentru a stabili dac el este prim sau nu este suficient s verific m dac el este divizibil doar prin acele numere prime p* n . ntr-adev r, s presupunem c n este compus $i c toate numerele prime ce-l divid verific inegalit #ile n < p n . Dac un anumit num r prim p0 divide pe n, atunci putem scrie p=p0n0 pentru un n0.2. n n < = n $i n0 | n. Num rul n0 va avea cel pu#in un Atunci n 0 = p0 n factor prim (care va fi mai mic dect n ) - absurd !. Ob#inem astfel un criteriu simplu de a determina dac un num r natural este prim sau nu : Dac$ un num$r natural n nu este divizibil prin nici un num $r prim p n atunci num$rul n este prim. Acest criteriu st la baza ,,ciurului prin care Eratostene a stabilit care numere dintr-o mul#ime finit de numere naturale sunt prime. Mai precis, el a scris de exemplu toate numerele de la 2 la n n ordine cresc toare. A t iat to#i multiplii proprii ai lui 2, apoi to #i multiplii proprii ai lui 3, pe urm pe cei ai lui 5. Se observ c cel mai mic num r natural superior lui 5 care nu a fost t iat este 7 $i se taie atunci $i to#i multiplii lui 7. Se continu n felul acesta procedeul de t iere pn se ajunge la etapa cnd cel mai mic num r natural din $irul 2, 3, ,n care nu a fost t iat este . n . Atunci procedeul se opre $te deoarece conform criteriului enun#at mai nainte toate numerele net iate din $irul 2, 3, ,n sunt numere prime p *n. n! *.

78

De exemplu num rul 223 nu se divide cu 2, 3, 5, 7, 11 $i 13. Este inutil s verific m dac se mai divide cu 17 c ci 172=289 >223, rezultnd astfel c 223 este prim. Procedeul descris mai sus poart numele de ciurul lui Eratostene . Pe aceast cale se poate ob#ine urm torul $ir de numere prime mai mici dect 100 : 2, 3, 5, 7, 11, 13, 17, 19, 23, 29, 31, 37, 41, 43, 47, 51, 53, 59, 61, 67, 71, 73, 79, 83, 89, 97. n anul 1909 au fost editate tabele cu numerele prime <10.000.000, n care se dau cei mai mici divizori primi pentru fiecare num r natural *10.170.600 care nu se divid la 2, 3, 5 sau 7. n anul 1951 au fost publicate tabele de numere prime pn la 11.000.000. Jacob Philipp Kulik (1793-1863) a ntocmit tabele de numere prime pn la 100.000.000 (manuscrisul se p streaz la Academia Austriac de /tiin#e din Viena). n finalul lucr rii, n cadrul Anexei 1 prezent m numerele prime de la 1 la 10.000. C. L. Baker $i J. F. Gruenberger au ntocmit n anul 1959 un microfilm care con#ine toate numerele prime mai mici dect p 6000000=104. 395. 301 . 3 Teorema Bertrand-Ceb !ev n cadrul acestui paragraf vom demonstra urm torul rezultat: TEOREMA 3.1. Dac$ n! , n+4, atunci ntre n !i 2(n-1) se afl$ cel pu in un num$r natural prim. Acest rezultat a fost formulat nc din anul 1845 de c tre J. Bertrand ns cel care a prezentat primul o solu#ie a acestuia a fost P. L. Ceb$ev n anul 1850. n cele ce urmeaz vom prezenta o solu#ie a lui P. Erds (adaptat de L. Kalmar). Aceast solu#ie se bazeaz pe demonstrarea ctorva leme: LEMA 3.2. Dac$ n! , n>1, atunci 4n n (1). C2 n > 2 n Demonstra#ie Facem induc#ie dup n. Pentru n=2, (1) este adev rat
2 deoarece C 4 =6>

42

2 2

8 2

6 2 > 8 3 2 > 4 18 > 16 ceea ce este

evident.

79

n +1 Cum C 2 n+ 2 = 2

2n + 1 n C 2 n , pentru a proba (1) pentru n+1, este n +1 2 n +1 2n + 1 > 4n(n + 1) +

suficient s demonstr m c 2 2n + 1 4 n 4 n +1 2n + 1 1 > > n +1 2 n 2 n +1 n +1 n

+ 4n2+4n+1>4n2+4n +1>0 ceea ce este evident . & LEMA 3.3. Dac$ definim P1=1 iar pentru n+2, Pn = Pn<4n, pentru orice n! *.
p prim p n

p , atunci

Demonstra#ie Facem din nou induc#ie dup n. Pentru n =1, 2 totul este

clar. Presupunem lema adev rat pentru toate numerele <n $i s o demonstr m pentru n. Dac n este par, atunci Pn=Pn-1 $i totul este clar. Dac n este impar, n=2k+1 (k! *), atunci orice num r prim p a.. k+2*p*2k+1 este un divizor al
k lui C 2 k +1 =

(2k + 1)

k k +1 k k k Din (1 + 1)2 k +1 > C 2 k +1 + C 2 k +1 = 2C 2 k +1 deducem c C 2 k +1 < 4 . (2) k Produsul tuturor numerelor prime p a.. k+2 *p*2k+1 diviznd C 2 k +1

2k (2k - 1).....(k + 2 ) . 1 2..... k

este inferior lui 4k (#innd cont de (2)) . Scriind c Pn=P2k+1=Pk+1

p prim k + 2 p n

p $i

#innd cont de ipoteza de induc #ie Pk+1<4k+1 $i de (2) deducem c Pn<4k+1 4k=42k+1=4n $i astfel Lema 3.3. este demonstrat . &
n LEMA 3.4. Dac$ p este un num$r prim ce divide C 2 n a.. p 2n , n atunci p apare cu exponentul 1 n descompunerea lui C 2 n n factori primi.

Demonstra#ie

Exponentul

lui

n C2 n =

(2n ) !

(n !)

va

fi

a = k - 2 k k 1 p p

2n

n . 80

Dac p. 2n (avem p= 2n +n=2 n care caz lema este adev rat c ci


2 C4

= 2 3 ), atunci pentru n.3 avem p. 2n , de unde deducem imediat c n

a = - 2 < 2 , de unde +=1 $i astfel lema este demonstrat . & p p


n LEMA 3.5. Dac$ p este un num$r prim, r! * a.. pr| C 2 n , atunci p (2 n ) r n p )2n !i C 2 n (2n ) (unde pentru x!9+ prin 3(x) desemn$m num$rul numerelor prime q)x). n Demonstra#ie Din pr| C 2 exponentul lui p n n , deducem c

2n

descompunerea lui

x!9 avem [2 x ] - 2 [x ] 1 ar trebui s avem + *r-1 ceea ce contrazice faptul c + . r. Deci pr *2n . )innd cont $i de lucrul acesta, pentru a demonstra partea a n doua a lemei #inem cont de faptul c n descompunerea n factori primi a lui C 2 n p (2 n ) n nu pot s apar dect numere prime q *2n, de unde deducem c C (2n ) .
2n

2n n 2 ) k k p k 1 p verific inegalitatea + .r. Dac am avea pr >2n, pentru k .r am avea r -1 2n 2n n n . Cum pentru orice k - 2 k =0 $i atunci a = k - 2 k k =1 p p p p
n C2 n n factori primi (care este a =

& LEMA 3.6. Dac$ n! , n>2, atunci nici un num$r prim p a.. 2 n n < p)n nu poate s$ divid$ C 2 n. 3 2n 2 2n n Demonstra#ie Dac n<p*n, atunci <3 $i 1, deci 2 3 p p p n $i 1 , de unde deducem c p 2n n - 2 2 - 2 1 = 0 . Cum pentru p p

2n n orice x!9, [2 x ] - 2 [x ] 0 , deducem c - 2 = 0 . p p

81

Pentru k>1, avem pk > 2n n k - 2 k p p c ci


3 C6

4 2 2n 9 n $i atunci k < < 1 pentru n>1, deci 9 2n p

n = 0 pentru k>1$i n >4. Rezult astfel c pentru n>4, p? C 2 n .

Pentru n=3 sau n=4, cu necesitate p=3 $i din nou lema este adev rat 4 = 20 iar C8 = 70 ce nu se divid prin 3. & LEMA 3.7. Un num$r prim p a.. n<p<2n apare n descompunerea n factori primi cu exponentul 1 (n /2). 2n n < 2 $i < 1 , deci Demonstra#ie Dac n<p<2n, atunci 1 < p p

lui

n C2 n

2n n 2n 2n 2 = 1 $i = 0 . Pentru k/2, avem k 2 < , deci pentru n>1 avem n p p p p 2n 2n n ca $ i < 1 $ i = 0 k =0. k pk p p
n Deci exponentul + al lui p n C 2 n este 1. &

n -1. 2 14 Demonstra#ie Se verific imediat c 4(14)=6= - 1 , adic lema este 2 LEMA 3.8. Dac$ n! , n/14, atunci 3(n) . adev rat pentru n=14. n n n $irul 1, 2, ,n numerele 4, 6, , 2 (n num r de - 1 ) sunt 2 2

compuse. Pe de alt parte, pentru n/15, $irul 1, 2, ,n con#ine $i numerele impare compuse 1, 9 $i 15, de unde deducem c n n n n n 4(n) . n - 2 - 1 + 3 = n - 2 - 2 < 2 - 1 . (c ci 2 > 2 - 1 ) $i astfel lema n este probat (observnd c pentru n/15 avem chiar p (n ) < - 1 ). & 2

82

LEMA 3.9. Fie Rn =

p prim n< p <2 n

p (sau Rn=1 dac$ nu exist$ astfel de


3

numere prime). Atunci, pentru n/98 avem Rn >

4n
n 2

2 n (2n )

(3) .

n Demonstra#ie Dup felul n care am definit pe Rn deducem c Rn | C 2 n, n deci putem scrie C 2 n = RnQn , cu Qn ! *.

Conform Lemei 3.7., dac p este un num r prim a.. n<p<2n , atunci p?Qn $i prin urmare dac p este prim $i p|Qn , cu necesitate p.n. Conform Lemei 2 3.6. avem chiar mai mult, p n , astfel c produsul divizorilor primi ai lui Q n 3 va fi cel mult egal cu P 2 n iar conform Lemei 3.3. acest produs va fi
3

<

2n 4 3

2n

4 3 . n Conform Lemei 3.4., cum Qn | C 2 n se vede c exponentul unui num r

prim p din descompunerea lui Q n nu va fi >1 dect dac p< 2 n . Num rul acestor numere prime va fi conform Lemei 3.8. (nlocuind n aceasta pe n prin

[ 2n ] 14 ) inferior lui
final c Qn
2n <4 3

[ 2n ] , lucru posibil deoarece n/98 $

2n 14 , de unde $i

2n . 2 Conform Lemei 3.5., produsul puterilor acestor numere prime (care


2n 2

n divid Qn, deci $i pe C 2 n ) va fi cel mult egal cu (2n )

, de unde deducem n

(2n )

2n 2

. (4)
n C2 n Qn

Astfel, cum Rn= inegalitatea (4) inegalitatea (3). & c

deducem, #innd cont de Lema 3.2. 1


2n 43

$i exact

Rn >

4n 2 n

(2n )

2n 2

4n
n 2

2 n (2n )

adic

LEMA 3.10. Dac$ k! , k+8, atunci 2k >18(k+1). 83

Demonstra#ie Cum 28=256 >189 iar dac 2k+1=22k>2(18(k+1)) =36k+36>18k+36 =18(k+2),

2k>18(k+1), atunci conform

deducem

principiului induc#iei matematice c lema este adev rat pentru orice k .8. & LEMA 3.11. Dac$ x!9, x+8, atunci 2x >18x .

Demonstra#ie Pentru x!9, x.8 avem [x].8 $i conform Lemei 3.10. avem 2 .2[x] .18([x]+1) >18x .&
x

LEMA 3.12. Dac$ k! , k+6, atunci 2k >6(k+1). Demonstra#ie Se face induc#ie matematic dup k (sau, dac #inem cont de Lema 3.10. mai avem de demonstrat inegalit #ile pentru k=6 $i k=7 care sunt adev rate deoarece 2 6 >64 > 67 $i 27>128>68) .& LEMA 3.13. Dac$ x!9, x+6, atunci 2x >6x .

Demonstra#ie Analog ca n cazul Lemei 3.11. & LEMA 3.14. Dac$ n! , n+648, atunci Rn >2n. Demonstra#ie )innd cont de Lema 3.9. este suficient s demonstr m c pentru n.648 avem
3

4 n > 4n n (2n )
2n 6

. Cum pentru n.648,

2n 6, 6

conform Lemei 3.13. avem 2 puterea 2n deducem c


3

> 2n , de unde ridicnd ambii membrii la


n 2

2 n > (2n )

. 2n > 8 $i atunci conform Lemei 9

De asemenea, din n.648, deducem c 3.11. avem 2 Deci


3 2n 9 n

> 4n , de unde 2 3 > 4n 4n > 4n n . pentru


n 2

n.648,

n 23

> (2n )

$i

n 23

> 4n n

de

unde

4 n > 4n n (2n )

$i cu aceasta lema este demonstrat . &

LEMA 3.15. Dac$ n+6, atunci ntre n !i 2n se afl$ cel pu in dou$ numere prime distincte. 84

Demonstra#ie Dac n.648, atunci conform definirii lui Rn, dac n intervalul (n, 2n) nu ar exista nici un num r prim, sau numai unul, atunci Rn *2n, ceea ce ar fi n contradic #ie cu Lema 3.14. 7 $i 11. Dac n=6, lema este adev rat c ci ntre 6 $i 12 se afl numerele prime

Mai avem de demonstrat Lema 3.15. pentru 7 *n*647. Acest lucru poate fi f cut fie direct (utiliznd un tabel de numere prime *1000), fie construind un $ir de numere prime q 0, q1,qm a.. q0=7, qk < 2qk-2, 2*k*m $i qm-1>a=647. O dat construit un astfel de $ir (cum ar fi de exemplu $irul 7, 11, 13, 19, 23, 37, 43, 73, 83, 139, 163, 277, 317, 547, 631, 653, 1259 pentru m =16), s vedem cum rezult Lema 3.15. pentru 7*n*a=647. Primul termen al $irului q0, q1, qm nu dep $e$te pe n dect dac qm>qm-1 >a.n, deci qm>n. Exist deci un indice maximal k<m-1 a.. qk<n. Atunci k+2 *m, n<qk+1 $i cum qk+2<2qk*2n, ntre n $i 2n exist cel pu#in numerele prime q k+1 $i qk+2 $i cu aceasta lema este complet demonstrat . & TEOREMA 3.16. (Ceb!ev) Dac$ n! , n+4, atunci ntre n !i 2(n-1) avem cel pu in un num$r prim. Demonstra#ie Pentru n=4 $i n=5 teorema este adev rat n mod evident deoarece ntre 4 $i 6 se afl 5 iar ntre 5 $i 8 se afl 7. Pentru n.6, conform Lemei 3.15. ntre n $i 2n se afl cel pu#in dou numere prime distincte p $i q cu p<q. Dac cel mai mare dinte acestea este q =2n-1, cel lalt trebuie s fie <2n-2 c ci 2(n-1) este par $i compus pentru n.6. Deci n<p<2(n-1). Dac q<2n-1, cum p<q, din p<q deducem c n<p<2n-2 $i cu aceasta Teorema lui Ceb$ev este complet demonstrat . & n continuare vom prezenta cteva corolare la Teorema lui Ceb $ev: COROLARUL 3.17. Dac$ n! , n+2, atunci ntre n !i 2n se afl$ cel pu in un num$r prim. Demonstra#ie Dac n.4 totul rezult din teorema lui Ceb$ev. Dac n=2 ntre 2 $i 4 se afl 3 iar dac n=3 atunci ntre 3 $i 6 se afl 5. Astfel Corolarul este demonstrat pentru orice n .2. & Observa#ie n anul 1892 J. J. Sylvester a demonstrat urm toarea generalizare a Corolarului 3.17.: 85

Dac$ n, k ! , n>k, atunci n !irul n, n+1, n+k-1 se afl$ cel pu in un num$r admi nd un divizor prim > k. Corolarul 3. 17. Se deduce acum din acest rezultat pentru n =k+1. Aceast generalizare a mai fost demonstrat $i de I. Schur n 1929 ca $i de P. Erds n 1934. COROLARUL 3.18. Dac$ k! , k>1, atunci pk<2k (unde pk este al k-lea num$r prim). Demonstra#ie Facem induc#ie dup k. Pentru k=2 avem p2=3<22. Dac pk<2 , conform Corolarului 3.17. exist
k

cel pu#in un num r prim p a..

2k<p<22k=2k+1 $i astfel corolarul este demonstrat. & COROLARUL 3.19. Dac$ n! , n+2, atunci n descompunerea lui n! n factori primi g$sim cel pu in un num$r prim cu exponentul egal cu 1. Demonstra#ie Corolarul este n mod evident adev rat pentru n=2 $i n=3. (2!=2, 3!=23) . Fie acum n.4. Dac n este par, n=2k, atunci k.2 $i conform Corolarului 3.17. ntre k $i 2k=n g sim cel pu#in un num r prim p a.. k<p<2k=n. Vrem s demonstr m c p apare cu exponentul 1 n descompunerea n factori primi a lui n!. ntr-adev r, urm torul num r din n! ce ar fi multiplu de p este 2p ns din k<p $2k<2p +2p>n. Dac n este impar, n=2k+1$k.2 $i din nou conform Corolarului 3.17. p apare n ntre k $i 2k g sim cel pu#in un num r prim p (k <p<2k). Avem deci p<2k<n $i 2p>2k $2p>2k+1=n $i din nou ajungem la concluzia c descompunerea lui n! cu exponentul 1. & Observa#ie De fapt, Corolarele 3.17. $i 3. 19. sunt echivalente. ntr-adev r, mai nainte am v zut cum Corolarul 3.17. implic Corolarul 3.19.. Reciproc, s admitem c ceea ce afirm Corolarul 3.19. este adev rat (adic pentru orice num r natural n.1 n n! exist cel pu#in un num r prim cu exponentul 1) $i s demonstr m Corolarul 3.17. (adic pentru orice n.2, ntre n $i 2n se afl cel pu#in un num r prim). 86

ntr-adev r, fie p num rul prim ce apare n descompunerea n factori primi a lui (2n)! cu exponentul 1. Avem p<2n<2p c ci dac am avea 2p*2n, atunci n (2n)!=12(n-1)n (n+1)(2n) apar $i p $i 2p $i astfel exponentul lui p n (2n)! ar fi cel pu#in 2. n concluzie, 2n<2p, adic n<p $i cum n<2p deducem c n<p<2n. & Deducem imediat: COROLARUL 3.20. Dac$ n! , n+2 atunci n! nu poate fi puterea unui num$r natural cu exponentul >1. COROLARUL 3.21. Pentru orice k! , k+4, avem inegalitatea pk+2<2pk. Demonstra#ie Pentru k.4 avem pk>p3=5 $i atunci conform Lemei 3.15. ntre pk $i 2pk exist cel pu#in dou numere prime distincte. Cum cele mai mici dintre aceste numere vor fi p k+1 $i pk+2 avem pk+2<2pk . & COROLARUL 3.22. Pentru orice k! , k+2 avem p k+2<pk+1+pk .

Demonstra#ie Pentru k=2, 3 se verific imediat prin calcul iar pentru k.4 totul rezult din corolarul precedent. & COROLARUL 3.23. Dac$ n, k! , n/2, atunci 1 1 1 + + ... + . n n +1 n+k 1 1 1 Demonstra#ie Dac x = + + ... + ! , atunci x/1 $i cum n n +1 n+k k +1 x< , cu necesitate k+1>n $i deci k/n. Fie p cel mai mare num r prim n .n+k. Atunci 2p>n+k . Conform Corolarului 3.17., ntre p $i 2p g sim cel pu#in un num r prim q, iar dac am avea 2p.n+k, atunci p<q<n+k, n contradic#ie cu alegerea lui p. Deci n+k<2p.

87

Cum k/n, atunci n+k/2n $i din nou conform Corolarului 3.17, ntre n $i 2n exist un num r prim r. Cum r<2n.n+k, #innd cont de felul n care l-am ales pe p deducem c r.p. 1 1 1 + + ... + n n +1 n+k exist numai unul al c rui numitor s fie divizibil prin p. Punnd pe x sub form de frac#ie (cu numitorul n (n + 1) ... (n + k ) ) se observ c printre termenii ce dau num r torul lui x exist unul ce nu se divide prin p. m (cu t = n (n + 1) ... (n + k ) ), p|t $i p?m, de Atunci, dac scriem x = t Deducem de aici c printre termenii sumei x = unde concluzia c x" .& 4 Inegalit$ ile lui Ceb!ev Reamintim c pentru x!9+, prin 4(x) am notat num rul numerelor prime p*x. Astfel, 4(1)=0, 4(2)=1, 4(3)=4(4)=2, 4(5)=4(6)=3, 4(100)=25, 4(1000)=168, etc. n anul 1958, D. H. Lehmer a calculat 4(108) $i 4(109) ar tnd c 4(108)=5761455 $i 4(109)=50847534. Evident, 4(pn)=n pentru orice n.1. Reamintim c n cadrul Lemei 3.9. am definit pentru n.1, Rn= De asemenea, deoarece n<r, avem n<p .n+k<2p.

p prim n< p 2 n

p.

Exist 4(2n)4(n) numere prime p a.. n <p*2n $i cum toate aceste numere prime sunt *2n deducem c R * (2n )p (2 n )-p (n ) . )innd cont de Lema 3.9., deducem c
n

pentru

2 n (2n ) logaritmnd n baza 10 deducem inegalitatea n 3 lg(4n ) 3 lg(2n ) (1) 4(2n)4(n) > . lg 4 3 lg(2n ) 2n 2n Cum lim lg x x
x

n.98

avem

inegalitatea

(2n )p (2n )-p (n ) >

3 n 2

de

unde,

= lim

lg x = 0 , din (1) deducem c x 88

lim [p (2n ) - p (n )] = .

De aici deducem urm torul COROLAR 4.1. Pentru orice num$r natural k exist$ un num$r natural mk a.. pentru orice n+mk , exist$ cel pu in k numere prime ntre n !i 2n. Fie acum p1,pr numerele prime ce intr n descompunerea n factori n primi a lui C 2 n (evident p1, p2, pr *2n). Fiecare num r pi apare la puterea 2n - 2 n + .... + 2n p p qi pi i i natural pentru care p iqi * 2n. Cum pentru orice a.0, 2n n - 2 k k k =1 pi pi
qi

n - 2 q , unde qi este cel mai mare num r pi i

[a] - 2

a 2 = 0 sau 1, deducem c

suma

+2 ...4 + 1 = q i , astfel c fiecare pi apare n descompunerea 1 1 4 3 q i


r ori p (2 n )

q1 qr n n lui C 2 2n )...(2n ) = (2n )r . n la o putere *qi, deci C 2 n p1 ..... p r ( 1 4 24 3

Cum r=4(2n) deducem c (2n ) (este de fapt o redemonstrare a inegalit #ii din cadrul Lemei 3.5. !). (2n ) (2n - 1)....(n + 1) se divide prin produsul n Pe de alt parte, C 2 n = 1 2 ..... n tuturor numerelor prime ps+1, ps+2,pr mai mari dect n $i mai mici dect 2n (am notat prin p1, ps toate numerele prime mai mici dect n). n n2 .... n = n r-s. Astfel, C 2 n p s +1 p s + 2 ... p r > n 1 4 4 3
n C2 n r - s ori

Cum r=4(2n) $i s=4(n), deducem c (2) n p (2 n )-p (n ) < C n < (2n )p (2 n ) .


2n

De asemenea, pentru orice num r natural n.1, avem n n (3) 2 n < C 2 n <4 . Comparnd (1) cu (2) deducem c 2 n < (2n )p (2 n ) , de unde prin logaritmare n baza 10 deducem: lg 2 2n 2n (4) p (2n ) > = 0,15051... 2 lg(2n ) lg(2n )

89

2n 2 deducem c 2n + 1 3 2 p (2n + 1) lg(2n + 1) > p (2n ) lg(2n ) > 0,15051...2n > 0,15051...(2n + 1) = 3 = 0,10034... (2n + 1) 2n + 1 sau p (2n + 1) > 0,10034.... . lg(2n + 1) Ob#inem astfel urm torul rezultat: PROPOZI'IA 4.2. Pentru orice num$r natural n>1, avem n . inegalitatea p (n ) > 0,1 lg n Tot din combina#ia inegalit #ilor (2) $i (3) deducem c n p (2 n )-p (n ) < 2 2 n Cum pentru n.1 avem pentru logaritmare deducem c n n [p (2n ) - p (n )]lg n < 2n lg 2 , adic p (2n ) - p (n ) < 2 lg 2 = 0,60206.... . lg n lg n x Fie acum x.0 un num r real. Dac not m = n , atunci n mod 2 evident x=2n sau 2n+1 $i vom avea n n x p (x ) - p p (2n ) - p (n ) + 1 < 0.60206... + 1 < 1,60206.... lg n lg n 2 n (deoarece > 1 ). lg n n x Se verific imediat c pentru n.3, din n<x rezult < , deci lg n lg x x x x pentru 3 avem p (x ) - p < 1,60206..... . lg x 2 2 x (Este u$or de verificat c ultima inegalitate este valabil $i pentru < 3 ; 2 x x ntr-adev r, dac < 3 , diferen#a p (x ) - p evident poate fi egal cu 2 2 2 orice n>1, de unde prin

90

(pentru 2,5* 1,60206...

x <3), cu unu sau cu zero; n toate aceste cazuri, produsul 2

x va lua valoarea cea mai mare). lg x

Astfel, pentru orice x!9+ x x (5) p (x ) - p < 1,60206... . lg x 2 Din (5) deducem mai departe c x x x x x p ( x ) lg x - p lg = p (x ) - p lg x + p lg x - lg < 2 2 2 2 2 < (lg x ) 1,60206... lg 2 x x + lg 2 p < 1,60206.... + x 1,75257 x 2 lg x 2

x x (am folosit faptul evident: p < ). 2 2 x x Deci p ( x ) lg x - p lg < 1,75257 x . 2 2 Fie acum n! , n>1. Conform ultimei inegalit #i avem n n p (n ) lg n - p lg < 1,75257... n 2 2 n n n n n 2 2 2 4 4 .............................. n n n n n p k -1 lg k -1 - p k lg k < 1,75257... k -1 2 2 2 2 2 (vom alege pe k a.. 2k>n). Adunnd aceste inegalit #i deducem c :

p lg - p lg < 1,75257...

p (n ) lg n - p

n n n n lg k < 1,75257... n + + ... + k -1 k 2 2 2 2

2k < = 1,75257... 1 12

n-

< 3,50514... n < 4n

91

Cum

pentru

2k>n,

n 2
k

< 1 $i

deci

n = 0, 2k

deducem

p (n ) < 4

n . lg n Am ob#inut astfel: n . lg n

PROPOZI'IA 4.3. Dac$ n>1, p (n ) < 4

Din Propozi#iile 4.2. $i 4.3. deducem: PROPOZI'IA 4.4. Pentru orice num$r natural n>1, avem dubla n n inegalitate 0,1 < p (n ) < 4 . lg n lg n Observa#ii 1. Dac trecem la logaritmi naturali, Propozi#ia 4.4 cap t o n n < p (n ) < 1,11 , astfel c varia#ia func#iei formulare mai elegant 0,92 lg n lg n n 4(n) este redat cu o mai mare exactitate de func#ia (factorii numerici 0,92 lg n $i 1,11 difer pu#in de 1). Aceste rezultate apar #in de asemenea lui Ceb$ev. n 2. Ceb$ev a demonstrat de asemenea c dac raportul p (n ) : tinde lg n n (pentru n ) la o limit l , atunci l =1. Faptul c limita raportului 4(n): lg n exist pentru n ($i deci este egal cu 1) a fost demonstrat pentru prima dat de J. Hadamard (la aproximativ 50 de ani de la lucr rile remarcabile ale lui P. L. Ceb$ev) utiliznd un aparat matematic complicat, specific matematicilor superioare (o demonstra #ie elementar a fost totu$i dat ceva mai trziu de matematicianul danez A. Selberg; recomand m cititorului lucrarea [21]). n Ob#inem deci p (n ) pentru n>1. lg n TEOREMA 4.5. (Ceb!ev) Pentru x!9, x+2 avem dubla inegalitate lg 2 x x < p ( x ) < 9 lg 2 . 4 lg x lg x Demonstra#ie Pentru prima inegalitate #inem cont de dou inegalit #i stabilite mai nainte $i anume n p (2 n )-p (n ) < C n < (2n )p (2 n ) $i
2n

92

n n 2 n < C2 pentru n! , n.2, de unde deducem c n < 4 n n p (2n ) - p (n ) 2 lg 2 $i p (2n ) lg 2 . lg n lg(2n ) x Pentru x!9, x.2, alegem acum n! a.. n < n + 1 , astfel c 2 lg 2 (2n + 2 ) lg 2 x n n p ( x ) p (2n ) lg 2 lg 2 > . lg(2n ) lg x 4 lg x 4 lg x S stabilim acum a doua inegalitate. y Pentru un num r real oarecare y.4, alegem n! a.. n - 1 < n . 2 Astfel, ( y + 2)lg 2 + 1 2n lg 2 y +1 p ( y ) - p p (2n ) - p (n ) + 1 lg n y 2 lg 2 2( y + 2 ) lg 2 3 y lg 2 4 y lg 2 +1 +1 < lg y lg y lg y

Am demonstrat y y p ( y ) - p < (4 lg 2) . lg y 2 Evident c y y lg y $i

astfel

pentru

y!9,

y.4,

avem

y pentru 2*y<4 avem p ( y ) - p 2 $i cum func#ia 2 atinge valoarea minim n y=e, deducem c

2 y y p (y) - p e pentru 2*y*4. lg y 2 2 y y Cum ns < 4 lg 2 , deducem c p ( y ) - p < (4 lg 2 ) pentru e lg y 2 orice y.2. Astfel, pentru y.2, avem: y y y y y p ( y ) lg y - p lg = p ( y ) - p lg y + p lg 2 < 4 y lg 2 + lg 2 2 2 2 2 2 9 = lg 2 . 2 93

( )

a.. 2r+1*x<2r+2. x x x nlocuind n ultima egalitate pe rnd pe y cu x, , 2 ,...., r ob#inem r+1 2 2 2 inegalit #i ; adunnd membru cu membru aceste inegalit #i $i #innd cont de x faptul c p r +1 = 0 ob#inem n final c 2 9 x x p ( x )lg x < x + + ... + r lg 2 < (9 lg 2 )x , adic a doua inegalitate din enun#. 2 2 2 & Observa#ie n cartea lui G.Tenenbaum : Introduction la thorie analitique des nombres (Universit de Nancy, 1991, p. 22) se demonstreaz c x 1 x 3 . 1+ 1+ pentru x.52 avem < p (x ) < lg x 2 lg x lg x 2 lg x TEOREMA 4.6. Pentru n! , n +2 avem n lg n 8n lg n < pn < . 9 lg 2 lg 2

Fie acum x!9, x.2 $i r!

Demonstra#ie )innd cont de Teorema 4.5., pentru n! , n.1 avem : p n = p ( p n ) < (9 lg 2 ) n , de unde deducem prima inegalitate din enun#. Cum lg p n lg x func#ia f:(0,+ )(9, f(x)= pentru x>0, este descresc toare pentru x >e2 x lg 2 lg x lg 2 deducem c pentru x .e9 avem < . Deci, dac pn .e9 iar f (e9)< 4 4 x avem lg p n pn < lg 2 . 4 lg 2 p n . Combinnd 4 lg p n < lg 2 n lg p n < , ceea 4 pn

Pe de alt parte, pentru n.1, avem n = p ( p n ) > cele dou inegalit #i ob#inem c dac pn.e9, atunci ce implic printre altele c

lg p n pn

p n < n $i c lg pn <2 lg n.

94

lg 2 p n < n lg p n < 2n lg n $i astfel 4 membrul drept al inegalit #ii din enun# este verificat pentru pn.e9. Pentru Deducem c pentru pn.e9, 2*pn<e9 inegalitatea din enun# se verific prin calcul direct. & Observa#ie n lucrarea lui B. Rosser : The n-th Prime is Greater than n lg(n) din Proc. London Math. Soc., vol. 49, 1939, pp. 21- 44 se demonstreaz c dac n.4, atunci n lg n +n lg (lg n) 10n < pn<n lg n +n lg (lg n)+8n. ntr-o lucrare mai recent a lui B. Rosser $i L. Schoenfeld: Aproximate formulas for some functions of prime numbers din Ilinois J. Math vol. 6, 1962, pp. 64-89 se demonstrez urm toarele: 3 1) Pentru orice n! , n/2 avem p n > n ln n + ln ln n - 2 1 2) Pentru orice n! , n/20 avem p n < n ln n + ln ln n - . 2 TEOREMA 4.7. Pentru orice x!9, x+3, exist$ dou$ constante reale 1 pozitive c1, c2 > 0, a.. c1 lg(lg x ) < < c 2 lg(lg x ) . p prim p
p x

1 pentru n prim Demonstra#ie Fie x!9, x.3. Cum 4(n)-4(n-1)= 0 in rest avem:
p prim p x

p (n ) - p (n - 1) 1 1 p (x ) 1 = = p (n ) = + p 2 n x n n n + 1 [x ] + 1 2 n x

p (n ) p (x ) + ( ) [ + n n 1 x] + 1 2 n x Conform inegalit #ilor lui Ceb$ev (Teorema 4.5.) deducem c pentru lg 2 p (n ) 9 lg 2 x.2 avem < < , de unde deducem c 4 lg n n lg n lg 2 1 p (n ) 1 < < 9 lg 2 . 4 2n x (n + 1) lg n 2 n x n(n + 1) n + 1 `) lg n ( 2 n x
=

Prin induc#ie matematic se probeaz c pentru orice k! , k.1 avem :

95

lg k <

1 n =1 n

lg k + 1 . De asemenea, pentru orice x!9, x.1 avem

1 - lg x *1. nN -{0} n

n x

Din cele de mai nainte deducem existen#a unei constante c > 0 a..
2 n x (n + 1) lg n

- lg(lg x ) < c . Evalund acum

p (n ) [x] + 1

ob#inem constantele c1 $i

c2 din enun#. & Observa#ie Dac pentru dou func#ii reale f $i g scriem f ~ g dac f (x ) lim = 1 , atunci vom men#iona urm toarele rezultate : x g ( x ) x . Acest rezultat cunoscut $i sub numele de Teorema lg x elementului prim sau Legea de reparti #ie a numerelor prime a fost intuit de Legendre $i Gauss n secolul al 18-lea $i demonstrat n 1896, independent de J. Hadamard (1865-1963) $i G. J. de la Valle-Poussin cu metode specifice analizei complexe. Pentru o demonstra#ie elementar a Teoremei num rului prim cititorul este rugat s consulte P. Erds : ,, On a New Method in Elementary Number Theory which leads to an Elementary Proof of the Prime Number Theorem, Proc. Nat. Acad. Sci. , Washington , vol 35, 1949, pp. 347-383 sau A. Selberg : ,, An Elementary Proof of the Prime Number Theorem, Ann. Math. Vol 50, 1949, pp. 303-313. 1. p (x ) ~ 2. La 15 ani Gauss a conjecturat c Deoarece 0< <
x x

p (x ) ~ Li ( x ) =
1

1 dt . lg t 2

dt = + dt $i 2 lg 2 2 (lg t )2 2 lg t

(lg t )
2

dt =

(lg t )

dt +

x x

(lg t )2

dt <

(lg 2 )

x -2

x- x x 4x < + , deducem c 2 2 1 2 lg 2 lg x ( ) ( ) (lg x ) 4 96


0<

(lg t )2
x lg x

dt <

x (lg 2 )

lg x

4 , de unde acum se deduce facil c lg x

x ~Li(x). lg x 1 este divergent$. pn Demonstra#ie Fie p1, p2, , pl(n) toate numerele prime * n $i s definim -1 -1 l (n ) 1 1 1 , atunci 1 l (n ) = . Deoarece = 1 a pi pi ai = 0 pi i i =1 TEOREMA 4.8. Seria
n1

l (n ) = p1a1 ... plal

-1

(unde sumarea se face dup

toate l-upurile de numere

naturale (a1, ,al)). n particular 1 + n . Avem :


l 1 lg l (n ) = - lg 1 - p = i =1 i

1 1 + .... + < l (n ) $i astfel l (n ) pentru 2 n

= ns

i =1 m =1

(m

p im

-1

-1 p1

-1 p2

+ .... +

p l-1

+
-1

i =1 m = 2

(m

-1 p im

m= 2

(m pim ) < pi- m = pi-2 (1 - pi-1 ) 2 pi-2 astfel c


lg l (n )
m=2 -1 < p1 -2 + ... + pl-1 + 2 p1 + ... + pl-2 .

-1

Este ns

cunoscut faptul c dac

convergent , astfel c

. Atunci pi-2 este 6 n 1 n i 1 1 presupunem c este convergent , atunci n1 p n

trebuie s existe o constant M a..

imposibil (deoarece am stabilit c l (n ) pentru n ), de unde deducem 1 c este divergent . & n1 p n

lg(l (n )) < M l (n ) < e M , ceea ce este

97

5 Teorema lui Scherk Rezultatul pe care l prezent m n continuare este datorat lui H. F. Scherk $i prezint un fel de recuren# ,,slab pentru $irul ( p k )k 1 al numerelor prime. Mai precis, vom demonstra : TEOREMA 5.1. (H. F. Scherk) Pentru orice num$r natural n+1 exist$ o alegere convenabil $ a semnelor + sau a.. : (1) p 2 n = 1 p1 p 2 ... p 2 n - 2 + p 2 n -1 !i (2) p 2 n +1 = 1 p1 p 2 ... p 2 n -1 + 2 p 2 n . Observa#ie Formulele (1) $i (2) au fost enun#ate de Scherk n anul 1830 iar S. S. Pillai a fost primul care a prezentat o demonstra #ie a lor n anul 1928. n cele ce urmeaz vom prezenta o solu#ie dat de W. Sierpinski n anul 1952. Vom spune c un $ir (q n )n1 de numere naturale impare are proprietate (P) dac el este strict cresc tor, q1=2, q 2=3, q3=5, q4=7, q5=11, q 6=13, q7=17 $i qn+1 <2qn , pentru orice n! *. )innd cont de rela#iile de la Teorema lui Ceb$ev deducem imediat c $irul ( p n )n 1 al numerelor prime este un exemplu de $ir cu proprietatea (P). Astfel, pentru probarea formulelor (1) $i (2) ale lui Scherk, este suficient s le prob m pe acestea pentru un $ir (q n )n1 ce are proprietatea (P). LEMA 5.2. Dac$ (q n )n1 este un !ir cu proprietatea (P), atunci pentru orice num$r natural impar m ) q2n+1 (n +3), exist$ o alegere convenabil$ a semnelor ,,+ sau ,, a.. m = q1 q 2 .... q 2 n-1 + q 2 n . Demonstra#ie Vom demonstra aceast lem f cnd induc#ie matematic dup n.3. Dac n=3, atunci q7=17 iar numerele impare m * 17 sunt 1, 3, 5, 7, 9, 11, 13, 15, 17. Deoarece prin calcul direct se verific egalit #ile :

98

1 = -q1 + q 2 + q 3 - q 4 - q 5 + q 6 3 = q1 - q 2 - q 3 + q 4 - q 5 + q 6 5 = q1 + q 2 + q 3 - q 4 - q 5 + q 6 7 = - q1 - q 2 - q 3 - q 4 + q 5 + q 6 9 = q1 + q 2 - q 3 + q 4 - q 5 + q 6 11 = q1 - q 2 - q 3 - q 4 + q 5 + q 6 13 = q1 - q 2 + q 3 + q 4 - q 5 + q 6 15 = -q1 + q 2 + q 3 + q 4 - q 5 + q 6 17 = q1 + q 2 - q 3 - q 4 + q 5 + q 6 deducem c lema este adev rat pentru n=3. S observ m c pentru n=2 lema este fals c ci atunci q2=11 iar 5 de exemplu nu se poate scrie sub forma 2 3 5 + 7 pentru nici o alegere a lui ,,+ sau ,,-. S presupunem acum c lema este adev rat pentru n.3, $i fie 2k-1 un num r impar a.. 2k-1 *q2n+3. Cum $irul (q n )n1 are proprietatea (P) deducem c q2n+3<2q2n+2 $i prin urmare deducem c q2n+2<2k-1-q2n+2<q2n+2 astfel c pentru o alegere convenabil a semnelor ,,+ sau ,, avem 0 (2k - 1 - q 2 n + 2 ) < q 2 n + 2 . Cum din q2n+2<2q2n+1 deducem c - q 2 n +1 (2k - 1 - q 2 n + 2 ) - q 2 n +1 < q 2 n +1 $i astfel pentru o nou alegere convenabil a semnelor ,,+ sau ,, avem 0 [ (2k - 1 - q 2 n + 2 ) - q 2 n +1 ] q 2 n +1 . Cum q2n+2 $i q2n+1sunt numere impare, deducem c $i num rul m = [ (2k - 1 - q 2 n + 2 ) - q 2 n +1 ] este impar $i cum m*q2n+1, conform ipotezei de induc#ie g sim o alegere convenabil a semnelor ,,+ sau ,, a.. m = [ (2k - 1 - q 2 n + 2 ) - q 2 n+1 ] = q1 q 2 .... q 2 n -1 q 2 n , de unde deducem c la o alegere convenabil a semnelor ,,+ sau ,, avem 2k - 1 = q1 q 2 .... q 2 n +1 q 2 n + 2 $i astfel Lema 5.2. este demonstrat . & COROLAR 5.3. Pentru o alegere convenabil $ a semnelor ,,+ sau ,, avem egalitatea q 2 n +1 = q1 q 2 ..... q 2 n -1 + q 2 n .

99

Pentru n=1 $i n=2 se verific

imediat rela#iile q3=q1+q2 $i

q5=q1-q2+q3+q4. S demonstr m acum formulele (1) $i (2) din Teorema lui Scherk. ntr-adev r, pentru n.3, num rul q2n+1-q2n-1 este impar $i <q2n+1 $i deci conform lemei anterioare, la o alegere convenabil a semnelor ,,+ sau ,, avem egalitatea q 2 n +1 - q 2 n - 1 = q1 q 2 ... q 2 n -1 + q 2 n , de unde q 2 n+1 = 1 q1 ... q 2 n-1 + 2q 2 n $i astfel formula (2) rezult imediat considernd pentru n.1, qn=pn. Pentru n=1 sau n=2, prin calcul direct se verific orice n! *. Pentru a proba formulele (1) s observ m c q2n+2<2q2n+1 $i q2n+2-q2n+1-1 este impar $i <q2n+1, deci conform lemei putem alege convenabil semnele ,,+ sau ,, a.. q 2 n + 2 - q 2 n +1 - 1 = q1 ... q 2 n -1 + q 2 n , de unde q 2 n + 2 = 1 q1 ... q 2 n -1 + q 2 n + q 2 n +1 deci ( lund n loc de n+1 pe n ) q 2 n = 1 q1 ... q 2 n -3 + q 2 n - 2 + q 2 n -1 $i astfel $i (1) sunt verificate pentru n.3. Pentru n=0, 1 sau 2, cum q2=1+q1, q4=1-q1+q2+q3 iar q6=1+q1-q2-q3+q4+q5 deducem c formulele (1) sunt valabile pentru orice egalit #ile q3=1-q1+2q2 $i q5=1-q1+q2-q3+2q4, astfel c formulele (2) sunt valabile pentru

n! *. (lund din nou qn=pn ). & 6 Exist$ func ii care definesc numerele prime ? n cele ce urmeaz dorim s clarific m existen#a unor func#ii

(calculabile) f: *( * care s satisfac una din urm toarele condi#ii : prim). a) f(n)=pn , pentru orice n.1 (unde reamintim c pn este al n-ulea num r b) Pentru orice n! *, f(n) este num r prim iar f este func #ie injectiv . 1. Func ii satisf$cnd condi ia a) Hardy $i Wright $i-au pus urm toarele probleme : 1) Exist o formul care s ne dea al n-ulea num r prim pn ? 2) Exist o formul care s ne dea expresia fiec rui num r prim n func#ie de numerele prime precedente ? n cele ce urmeaz vom prezenta o formul pentru calculul lui pn. 100

Reamintim c pentru orice num r real strict pozitiv x prin 4(x) am notat num rul numerelor prime p a.. p * x. La nceput vom prezenta o formul pentru 4(m) dat de Willans n anul 1964. Pentru aceasta, pentru fiecare num r natural j.1 fie ( j - 1) ! +1 F ( j ) = cos 2 p . j Astfel, pentru orice num r natural j>1, F( j )=1 pentru j prim iar F( j )=0 n caz contrar (evident F(1) =1). Deducem c p (m ) = -1 + F ( j ) .
j =1 m

Willans a dat formula p (m ) = sin


2

H ( j ) , m=2, 3, .. unde

(( j - 1) !)
j sin
2

j =1

H ( j) =

j Min5 a dat o alt expresie pentru 4(m) n care nu mai intervine sinusul sau cosinusul $i anume m ( j - 1) !+1 ( j - 1) ! p (m ) = - . j j j = 2 Iat o demonstra#ie simpl pentru formula lui Min5. ncepem cu observa#ia c pentru n64, care nu este prim, n divide (n-1)!. ntr-adev r, fie n este de forma n=ab cu 2 * a, b * n-1, $i a ' b, fie n=p2'4. n primul caz, n divide (n-1)! n timp ce n al doilea caz 2<p*n-1= =p2-1, $i atunci 2p*p2-1 $i n divide 2p2=p2p care la rndul s u divide pe (n-1)!. Conform Teoremei lui Wilson pentru fiecare num r prim j putem scrie (j-1)!+1=kj, (k! *), deci ( j - 1) ! +1 ( j - 1) - j j ! 1 = k - k - = 1 . j

Dac j nu este num r prim, atunci dup remarca precedent (j-1)!=kj ( j - 1) ! +1 ( j - 1) ! 1 (k! *) $i astfel - = k + - k = 0 . j j j 101

natural dat m.2, a fost dat de Ernvall n 1975 : Fie d = m ! m ! - 1, (2m ) ! , t=

3 ! +1 3 ! - = 0 $i astfel formula lui n fine, dac j=4, atunci 4 4 Min5 este demonstrat . Utiliznd cele de mai sus se ob#ine formula lui Willans pentru pn : 1 n 1 n n n n n , sau p n = 1 + pn = 1 + m . m =1 m =1 1 + p (m ) F ( j ) j =1 O alt formul pentru cel mai mic num r prim superior unui num r

(( )

(d

dd
d

,d !

) iar a unicul num r natural pentru care d

divide t iar da+1 nu divide t. d t a ,d d .

Atunci cel mai mic num r prim p superior lui m este p =

Dac vom lua m=pn-1 ob#inem din nou o formul pentru pn . Reamintim cum se define$te func#ia lui Mbius : 8(1)=1, 8(n)=(-1)r dac n este un produs de r numere prime distincte iar 8(n)=0 dac n are ca factor un p trat. Cu ajutorul acestei func#ii, n 1971 Ghandi a ar tat c dac not m 1 m (d ) 1 sau, analog, P Pn-1=p1p2pn-1, atunci Pn = 1 log - + n 2 d P 2 d - 1 log 2 n -1 1 m (d ) <2. este singurul num r natural pentru care 1 < 2 Pn - + 2 d P 2d -1 n -1 Iat o demonstra#ie a formulei lui Ghandi prezentat n 1972 de Vanden Eynden : m (d ) S not m Q=Pn-1pn=p $i S= d . Atunci d Q 2 -1

102

(2

-1 S =
d

m (d )
Q
t

2Q - 1 2 -1
d

=
d

m (d )(1 + 2 d + 2 2 d + ... + 2 Q -d ) . Dac 0*t<Q


Q

termenul r(d)2 apare exact atunci cnd d|(t, Q). Deci coeficientul lui 2t n sum este m (d ) ; n particular, pentru t=0, coeficientul este egal cu m (d ) . d (t ,Q ) d Q Reamintim c
d

m (d ) =
m

1 daca m = 1 . Dac scriem 0 daca m > 1

0<t <Q

/ pentru

suma extins la toate valorile lui t a.. 0<t<Q $i (t, Q)=1, atunci (2Q-1) S= / 2t ; cel mai mare indice n aceast sum este t=Q-1. Rezult c
0<t <Q

1 2 2 Q - 1 - + S = - 2 Q - 1 + / 2 t +1 = 1 + / 2 t +1 . 2 0 <t < Q 0<t <Q -1 Dac 2*j<pn=p, exist un num r prim q a.. q<pn<p (deci q|Q) $i q|Q-j. Fiecare indice t din suma considerat mai nainte satisface deci condi#ia 0<t*Q-p. Atunci 2
Q - p +1

2 2Q

<-

1 +S= 2

1+

2 2 -1

0 <t Q - p Q

/ 2 t +1

<

2Q- p+2 2 2Q

. nmul#ind cu 2p

1 deducem c 1 < 2 p - + S < 2 .& 2 2. Func ii satisf!cnd condi ia b) Num rul f (n ) = q 3 este prim pentru orice n .1, (aici q 1,3064 vezi W. H. Mills : Prime- representing function, Bull. Amer. Math. Soc.,53 , pp 604) . Nw De asemenea, g (n ) = 2 2 (cu un $ir de n exponen#i) este un num r prim pentru orice num r natural n.1 (aici w 1,9287800-vezi E. M. Wright: A prime representing function, Amer. Math. Monthly, 58, 1951, pp.616-618). Din p cate, numerele q $i w se cunosc doar cu aproxima #ie iar valorile lui f(n) $i g(n) cresc foarte repede, a$a c cele dou func#ii nu sunt prea utile
n

[ ]

103

r mnnd doar ca ni$te curiozit #i (de ex, g(1)=3, g(2)=13, g(3)=16381, g(4) are deja mai mult de 5000 de cifre !). Tenta#ia de a g si o func#ie polinomial cu coeficien#i din 3 a.. valorile sale s fie numere prime este sortit e$ecului deoarece dac f!3[X] este neconstant, atunci exist o infinitate de ntregi n cu proprietatea c |f(n)| nu este num r prim. ntr-adev r, deoarece f este neconstant problema este trivial dac toate valorile lui f sunt numere compuse. S presupunem deci c exist n0.0 un num r natural a.. |f(n0)|=p este num r prim. Cum f nu este constant deducem c lim f ( x ) = + , deci exist n1>n0 a.. dac n.n1$|f(n)| > p. Astfel pentru orice
x

ntreg h pentru care n0+ph.n1 avem f(n0+ph)=f(n0)+Mp=Mp. |f(n0+ph)| > p, atunci |f(n0+ph)| este num r compus.

Dac

Cum dac f!<[X1,Xm] (m.2) are proprietatea c ia valori numere prime pentru orice X 1, ..Xn naturale, atunci cu necesitate f este constant, deducem c $i tenta#ia de a g si o func#ie polinomial neconstant de mai multe nedeterminate care s ia valori numere prime pentru oricare valori naturale ale nedeterminatelor este sortit e$ecului. Dac f(x)=x2+x+41 (faimosul polinom al lui Euler) atunci pentru k=0, 1, .., 39 f(k) este prim : 41, 43, 47, 53, 61, 71, 83, 97, 113, 131, 151, 173, 197, 223, 251, 281, 313, 347, 383, 421, 461, 503, 547, 593, 641, 691, 743, 797, 853, 911, 971, 1033, 1097, 1163, 1231, 1301, 1373, 1447, 1523, 1601 (pentru k=40$f(40)=1681=412). Dac vom considera f(x)=x2+x+q, (q prim) atunci sunt echivalente : 1) x2+x+q este prim pentru x=0, 1, , q-2 2) q=2, 3, 5, 11, 17, sau 41. Frobenius (1912) $i Hendy (1974) au demonstrat c : i) Singurele polinoame f(x)=2x2+p (cu p prim) a.. f(k) este prim pentru x=0, 1, .., p-1 sunt pentru p =3, 5, 11, 29. ii) Singurele polinoame de forma f (x ) = 2 x 2 + 2 x + pA1(mod 4)) a.. f(x) este prim pentru x=0, 1, , 13, 37. 104 p +1 (cu p prim, 2

p-3 sunt cele pentru p=5, 2

7. Numere prime gemene Dac p $i p+2 sunt simultan numere prime, vom spune despre ele c sunt gemene. Exemple : (3, 5), (5, 7), (11, 13), (17, 19), etc. n 1949, Clment [Clement, P. A. : Congruences for sets of primes, Amer. Math. Monthly, 56, 1949, 23-25 ] a prezentat urm torul rezultat legat de numerele gemene : Pentru n . 2, n $i n+2 sunt simultan prime +4[(n-1)!+1]+nA0 (mod n(n+2)). (din p cate din punct de vedere practic acest rezultat nu are nici o utilitate). Problema principal este de a decide dac exist sau nu o infinitate de numere gemene. Dac not m pentru x > 1 prin 42(x)=num rul numerelor prime p a.. p+2 este prim $i p+2 *x, atunci Brun a demonstrat n 1920 c exist un num r 100 x natural x0 (efectiv calculabil) a.. pentru orice x.x0 s avem p 2 (x ) < . (lg x )2 ntr-un alt articol celebru din 1919 ( La serie 1 1 1 1 1 1 + + + + + + .... , ou les denominateurs sont nombres premiers 5 7 11 13 17 19 jumeaux est convergente ou finie din Bull. Sc. Math., vol.43, pp. 100-104 $i 1 1 124-128 ) tot Brun a demonstrat c seria B = p + p+2 (unde suma este extins dup perechile de numere gemene (p, p+2)) este convergent sau mul#imea acestor numere gemene este finit . Num rul B poart numele de constanta lui Brun iar Shanks $i Wrench (n 1974) iar Brent (n 1976) au ar tat c BB1,90216054 Cele mai mari numere prime gemene cunoscute sunt 17065952 112351 $i 5713052 7701 1. ([26]) . De aici rezult c mul#imea numerelor prime gemene, dac este infinit , (lucru neprobat pn acum), atunci ele se apropie foarte mult unele de altele.

105

CAPITOLUL 8: FUNC$II ARITMETICE 1. Generalit$ i. Opera ii cu func ii aritmetice DEFINI'IA 1.1. Numim func ie aritmetic! orice func ie f: <. n cadrul acestui capitol vom prezenta mai multe exemple de astfel de func#ii. Fie A={f: <} mul#imea func#iilor aritmetice. Pentru f, gA definim f+g, fg, f*g: < astfel: (f+g)(n)=f(n)+g(n), (f*g)(n)= f (d ) g (n / d ) pentru orice n .
dn

(fg)(n)=f(n)g(n)

$i

Observa#ie f*g poart numele de ,, produsul Dirichlet de convolu #ie al lui f $i g. PROPOZI'IA 1.2. (A , + , *) este inel comutativ unitar . Demonstra#ie Faptul c (A, +) , este grup abelian este imediat. S prob m c (A , *) este monoid comutativ. ntr-adev r, dac f, g, hA, atunci: ( f * ( g * h))(n) = f (d ) g (e)h(n / de) =
d /n e/ n d

Dn eD

( f ( D / e) g (e)h(n / D)) = (( f * g ) * h)(n)

( D=de), pentru orice n , adic ,,* este asociativ . (am #inut cont de faptul c atunci cnd d parcurge divizorii lui d, acela$i lucru l face $i n/d). Cu acela$i argument rezult $i comutativitatea produsului de convolu#ie. 1 pentru n = 1 Elementul neutru pentru * este d : <, d (n ) = 0 pentru n 1 deoarece se verific imediat c f*d=d*f =f, pentru orice fA. Pentru a ncheia, s mai prob m c dac f, g, hA, atunci f*(g+h)=(f*g)+(f*h). ntr-adev r, dac n , atunci: ( f * ( g + h))(n) =

d /n

f (d )( g (n / d ) + h(n / d )) = f (d ) g (n / d ) +
d /n

106

d /n

f (d )h(n / d ) = ( f * g )(n) + ( f * h)(n) = ( f * g + f * h)(n) . &

PROPOZI'IA 1.3. fU(A) f(1) 0. Demonstra#ie: Dac f!U(A), atunci exist g = f 1A a.. 1 f*f =f 1 *f=d. Deci 1=d(1)=f(1)f 1(1), adic f(1)0. Reciproc, dac f(1)0, dac definim inductiv 1 dac n=1 f (1) 1 g(n)= f (d ) g (n / d ) dac n >1, f (1) d / n
d >1 not

se verific imediat c g=f 1 . & Iat cteva exemple de func#ii aritmetice: 1. Func ia 1 a lui Euler definit n 4 de la Capitolul 6. 2. Pentru k definim s k : < astfel s k= d k iar x k(n)=nk.
d /n

n particular s1 se va nota cu s (deci s(n)=suma divizorilor lui n ), s0 cu t (deci t(n)=num$rul divizorilor lui n) iar x0=x (x poart numele de func#ia zeta $i deci x(n)=1 pentru orice n ).
a1 ak ... p k este descompunerea canonic a lui n n produs de Dac n = p1 bk numere prime, atunci 9(n) va fi suma produselor de forma p1b1 ... p k cu ,i.+i ,

1.i.k adic

ak a1 2 = s (n ) = 1 + p1 + ... + p1 1 + p 2 + ... + p a 2 .... 1 + p k + ... + p k

)(

) (

p1

a1 +1

p -1 -1 p2 -1 ... k p1 - 1 p2 -1 pk -1

a 2 +1

a k +1

a1 ak ... p k atunci t (n ) = (a 1 + 1)...(a k + 1) . Se verific imediat c dac n = p1

3. Func#ia :: ( , :(n)=num rul divizorilor naturali ai lui n (n! ).

Observa#ie Conform Propozi#iei 1.3. func#ia zeta x are invers n inelul A; x-1 se noteaz cu m $i poart numele de func#ia lui Mbius. Deoarece m*x=d , deducem c : 107

dn

m (d ) =

1 daca n = 1 0 daca n 1
m

n particular, dac p este un num r prim iar a 2 atunci


a

m(p a)=0.
j =0

Astfel m(1)=1, m(p)=-1, iar m(p )=0, pentru orice a 2. Observa#ie Dac f, gA $i f=g*x , atunci g=f*m. Acest fapt este cunoscut sub numele de formula clasic$ de inversare a lui Mbius . Dac scriem explicit ob#inem: PROPOZI'IA 1.4. Dac$ f !i g sunt func ii aritmetice atunci f (n) = g (d ) pentru orice n g (n) = f (d ) m (n / d ) pentru orice
d /n d /n

n . Ca un exemplu avem c : sk(n)= nk=


d /n d /n

d k pentru orice n

astfel c

sk(d)m(n/d) pentru orice n .


LEMA 1.5. Pentru n !i d|n , fie Sd={ xn/d : 1xd, x , (x, d)=1} Atunci pentru d|n, e|n, d e, Sd Se = iar U Sd = {1,2,,n}.
d/n

Demonstra#ie: Presupunnd c Sd Se , exist x, y * a.. 1x d, 1 y e, (x, d)=(y, e)=1 $i xn /d=yn /e xe=yd. Cum (x, d)=1, x|y $i analog y|x, deci x=y, adic d=e - absurd !. Cum pentru d|n, 1mn $i (m, n)=n/d, dac m=xn /d, atunci (x, d)=1 $i 1xdm/nd, deducem c mSd adic {1,2,, n} U Sd $i cum incluziunea
d/n

invers este imediat deducem egalitatea cerut . & =

d /n

COROLAR 1.6. Cum Sd are 1(d) elemente, deducem c$ n= 1(d), pentru orice n . Conform Propozi#iei 1.4. deducem c j (n) =

dm ( n / d )
d /n

pentru orice

n . n particular, dac p este prim $i a1 natural, 2(p a) =

p j m(p a - j) = pa - pa -1 = pa (1j =0

1 ). p

108

2. Func ii multiplicative DEFINI'IA 2.1. O func ie aritmetic$ f se zice func ie multiplicativ! dac$ f0 !i f(mn)=f(m)f(n), pentru orice m, n cu (m, n)=1. Observa#ie Dac f este multiplicativ atunci din f0 exist un n a.. f(n)0 $i cum f(n)=f(1n)=f(1)f(n) deducem c f(1)=1, adic n inelul A, f este inversabil . Dac n
k

iar
i =1

a1 ak n = p1 ... p k este descompunerea n factori primi a lui a

n, atunci f (n) = f ( pi i ) , astfel c

o func#ie multiplicativ

este complet

determinat de valorile ei pe mul #imile de forma p a cu p prim $i a . S not m cu M familia func#iilor aritmetice multiplicative. PROPOZI)IA 2.2. Dac$ f M atunci 'i f 1 M . Demonstra#ie Fie m, n cu (m, n)=1. Dac m=n=1 atunci f 1(mn) = =f (m) f (n). Presupunem acum c mn 1 $i c f 1(m1 n1)=f 1(m1)f 1(n1) pentru orice pereche (m1, n1) de numere naturale cu m1n1 < mn $i (m1, n1) =1. Cum dac m=1 sau n=1 din nou f 1(mn)=f 1(n) f 1(m), r mne s analiz m cazul m1 $i n1. Conform Propozi #iei 1.4. avem: f -1 (mn) = - f (d ) f -1 (mn / d ) .
1 1

d / mn d >1

Deoarece (m, n)=1 orice divizor d al lui mn se scrie unic sub forma d =d1d2, unde d1|m $i d2 |n. Atunci (d1, d2)=1 $i (m/d1 , n/d2)=1. Astfel c : f -1 (mn) = - f (d1 d 2 ) f -1 (mn / d1 d 2 ) = (deoarece (m / d1 )(n / d 2 ) < mn) =
d1 m d2 n d 1d 2 >1

=-

d1 m d2 n d 1d 2 >1

f (d1 ) f (d 2 ) f -1 (m / d1 ) f -1 (n / d 2 ) = - f -1 (m) f (d 2 ) f -1 (n / d 2 ) d2 n d 2 >1

-f

-1

( n) f ( d 1 ) f
d1 m d 1>1

-1

(m / d1 ) - (- f (d 1 ) f
d1 m d 1>1

-1

( m / d 1 ))

109

(- f (d 2 ) f
d2 n d 2 >1

-1

(n / d 2 )) =
-1

= f

-1

( m) f

-1

( n) + f

( m) f

-1

( n) - f

-1

( m) f

-1

( n) = f

-1

( m) f

-1

(n)

$i

totul

este clar. & Observa#ie Cum func#ia zeta x este multiplicativ , inversa sa care este func#ia lui Mbius m este multiplicativ . Astfel : 1 dac n=1 m(n)= (-1)t dac n este produs de t primi distinc #i 0 n rest Avem n felul acesta o alt definire a func#iei lui Mbius. PROPOZI'IA 2.3. Dac$ f, gM atunci f*gM. Demonstra#ie (f*g)(1)=f(1)g(1)=1 iar dac (m, n)=1, atunci : ( f * g )(mn) = f (d ) g (mn / d ) = f (d1 ) f (d 2 ) g (m / d1 ) g (n / d 2 ) =
d mn
d1 m d2 n

= ( f (d1 ) g (m / d1 )) ( f (d 2 ) g (n / d 2 )) = [( f * g )( m)][( f * g )(n)]. &


d1 m d2 n

Observa#ii1. Deoarece x k este multiplicativ $i sk = x k * x deducem c $i sk este multiplicativ . Astfel dac k1, p este num r prim iar a1 atunci

s k ( p a ) = p jk =
j =0

p (a +1) k - 1 p -1
k

a1 iar dac n = p1 ... p ta t atunci


t

s k ( n) =
i =1

p i(ai +1) k - 1 p ik - 1

n particular, s (n) =
i =1

pia i +1 - 1 . pi - 1

Deoarece t (p a) = a+1 , t(n) =

(ai+1).
i =1

2 Cum func#ia lui Euler 2 este multiplicativ $i 2=x1*m atunci pentru 1 orice n : j (n) = n (1 - ) . p pn

p prim

3. Func#ia 2 a lui Euler este o func#ie calculabil (adic pentru orice n, 2(n) este cardinalul unei mul#imi $i anume a mul#imii {x: 1 x n $i (x, n)=1}). 110

Func#iile calculabile pot fi cte o dat evaluate #innd cont de principiul includerii 'i excluderii: Dac$ A1,,At sunt submul imi ale unei mul imi finite S, atunci | S (A1 At) | = |S| +
j (- 1) j =1 t 1 i1... i j t

Ai1 ... Ai j

3.Func ia Jordan Jk Func#ia Jordan J K reprezint o generalizare a func#iei Euler 2 $i se define$te astfel: DEFINI'IA 3.1. Pentru n , Jk(n)=num$rul k-uplurilor ordonate de numere naturale (x1,,xk) a.. 1 xi n, 1 i k !i ( x1, x2,,xk, n) =1. Observa#ie Evident J1=j.
a1 Fie n = p1 ... p ta t descompunerea n factori primi a lui n, S=mul#imea k-uplurilor (x1,, xk) a.. 1xi n, 1 i k, iar Ai =mul#imea acelor k-upluri din S pentru care pi | (x1,, xk), 1 i t, atunci: J k (n) =| S (A1 At) | iar

Ai1 ... Ai j = (n / pi1 ... pi j ) .Astfel:


n J K (n) = n + (-1) j =1 1 i1 < i2 <...< i j <t p i1 ... p i j
k t j k n k = m (d ) = d m (n / d ) d n d dn k

Deducem astfel c Jk=xk*m $i astfel rezult c Jk este func#ie multiplicativ . Dac p este prim $i a1, atunci 1 1 a ak (a -1) k = p ak (1 - k ) , astfel c J k (n) = n k (1 - k ) . JK (p ) = p - p p p pn 4.Func ia von Sterneck H k Iat acum o alt generalizare a func#iei lui Euler (numit func#ia von Sterneck ). DEFINI)IA 4.1. Pentru n, k" definim ( n ) = *(e1).*(ek), suma f$cndu-se dup $ toate k-uplurile (e1,,ek) Hk
[ e1,....,e k ]= n

de numere naturale a.. 1 ei n, 1 i k 'i [e1, , ek] =n. Observa#ie n mod evident 2=H 1 iar H k(1)=1. 111

Presupunem acum c (m, n)=1 $i c [e1,, ek]=mn. Pentru i=1,, k, ei poate fi descompus n mod unic sub forma ei=cidi, unde ci|m $i di|n, iar [c1,,ck]=m $i [d1,, dk]=n. Astfel: H k (mn) = j (e1 )...j (e k ) = j (c1 )...j (c k )j (d 1 )...j (d k ) =
[

e1,...,ek ]= mn
1 k

[ [ 1

=
[

c1,...,c k ]= m

j (c )...j (c

)
[

d 1,...,d k ]= n

j (d )...j (d

c1,...,c k ]= m d 1,...,d k ]= n
k

) = H k ( m ) H k ( n)

adic Hk este func#ie multiplicativ . PROPOZI'IA 4.2. Pentru orice k, H k=Jk. Demonstra#ie Facem induc#ie matematic dup k. Am v zut mai nainte c H1= 2=J1. Fie k>1 $i presupunem c Hk-1=Jk-1. Cum Hk $i Jk sunt func#ii multiplicative, a demonstra c Hk=Jk este suficient s demonstr m c Hk(pa)=Jk(pa) unde p este prim, iar a1. Conform ipotezei de induc#ie avem c Hk-1(pa)=Jk-1(pa) iar

H
=

( pa ) =
max(

j ( p b )...j ( p b ) = b b a
1 i 1 ,..., i ) =

max(

bj ( p b a
1 ,..., i ) =

b1

)...j ( p b i -1 )j ( p b i ) + .
max(

j ( p b )...j ( p b b b a
1 1 ,..., i )

i -1

)j ( pa ) =

= H k -1 p a =p
a -1

( ) j ( d ) + j (d )
d pa

k -1

j ( pa ) =

pa

H k -1 ( p ) + p

a ( k -1)

j ( pa ) =
1 p
k -1

= p a -1 J k -1 ( p a ) + p a ( k -1)j ( p a ) = p a -1 p a ( k -1) (1 = p ak [ 1 1 1 1 (1 - k -1 ) + (1 - )] = p ak (1 - k ) = p p p p

) + p a ( k -1) p a (1 -

1 )= p

a Jk (p ). &

112

5.Func ii complet multiplicative DEFINI'IA 5.1. O func ie fA se zice complet multiplicativ! dac$ exist$ n a.. f(n)0 iar f(mn)=f(m)f(n), pentru orice m, n . (dac$ not$m prin Mc clasa acestor func ii, atunci n mod evident M c M A) PROPOZI'IA 5.2. Dac$ fM, atunci fMc f-1=mf. Demonstra#ie Dac fMc, atunci pentru orice n : f (1) = 1 daca n = 1 ( mf * f ) = m ( d ) f ( d ) f ( n / d ) = f ( n ) m ( d ) = , daca n 1 dn dn 0 -1 adic mf *f =d f =mf. Invers, s presupunem c f-1=mf. Pentru a proba c fMc este suficient s prob m c dac p este prim $i a 1, atunci f(pa)=(f(p))a. Acest lucru l vom face prin induc#ie matematic dup a ; evident, pentru a=1 totul este clar. S presupunem c a2 $i c f(pa-1)=(f(p))a-1. Deoarece pentru oricare b2, f-1(pb)=m(pb)f(pb)=0, deducem c : -1 0=(f *f)(pa) = f(pa) + f-1(p) f(pa-1) = f(pa) + f-1(p) (f(p)) -1. Deoarece f-1(p)=-f(b) f(pa)=(f(p))a. 3 COROLAR 5.3. Dac$ fM, atunci fMcf-1(pa)=0, pentru orice p prim !i a2. PROPOZI'IA 5.4. Fie fM. Atunci fMcf(g*h)=(fg)*(fh), pentru orice g, hA. Demonstra#ie Dac fMc, atunci pentru orice g, hA avem ( f ( g * h))(n) = f (n) g (d )h(n / d ) = f (d ) g (d ) f (n / d )h(n / d ) = ( fg * fh)(n) .
dn dn

Invers, s presupunem c f(g*h)=(fg)*(fh), pentru orice g, h A. n particular, pentru g=x $i h=m avem d=fd=f(x*m)=fx*fm=f*fm, adic f-1=mf, adic fMc (conform Propozi #iei 5.4.).& PROPOZI'IA 5.5. Dac$ fM, atunci exist$ g, hMc a.. f=g*h f (p )=0, pentru orice p prim !i orice a3. Demonstra#ie S presupunem c f=g*h cu g, hMc $i fie p prim iar a3. Atunci
-1 a

113

(p ) = (g * h )(p ) = )= g = g ( p ) h ( p
f
-1

-1

-1

-1

-1

a- j

-1

j =0

(1) h -1 ( p a ) + g -1 ( p ) h -1 ( p a -1 )

(c ci gMc) =0 (c ci hMc $i a 3). Invers, fie fM a.. f -1(pa)=0 pentru orice p prim $i a3. Alegem gMc a.. pentru orice p prim, g(p) este o r d cin a ecua#iei : X 2+f-1(p)X+f-1(p2)=0. -1 Dac alegem h=g *f, atunci hM $i pentru orice p prim $i a2, avem: h-1(pa)=(g*f -1)(pa)+g(pa -1)f -1(p)+g(pa - 2)f -1(p2)=g(pa - 2)[(g(p))2+f -1(p)g(p)+ +f -1(p2) ] =0. Conform Propozi #iei 5.4., hMc si astfel f=g*h. & TEOREMA 5.6. Pentru fM, urm$toarele condi ii sunt echivalente: (1) Exist$ g, hMc a.. f=g*h; (2) Exist$ FM a.. pentru orice m, n : f (mn) = f (m / d ) f (n / d ) F (d ) .
d ( m ,n )

(3) Exist$ BM a.. pentru orice m, n: f (m) f (n) =

d (m,n )

f (mn / d 2 ) B (d ) .

(4) Pentru orice p prim !i a1: f(pa+1)=f(p)f(pa)+f(pa-1)[f(p2)-(f(p))2]. Demonstra#ie Vom demonsra c (1)(4), (4)(1), (2)(4), (4)(2), adic (1)(2)(4), iar apoi (2) (3) $i (3)(4) (1)(4). Presupunem c f=g*h cu g, hMc. Dac g(p)=M $i h(p)=N, atunci f(p)=M+N $i f(p2)=M2+MN+N2. Dac a1 atunci partea dreapt a egalit #ii din (4) este egal cu : ( M + N ) M i N a -1 - MN M i N a +1-i =
i=0 i =0

a -1

i =0

M i +1 N e -i + M i N a +1-i - M i +1 N a -1 =
i =0 i =0

a -1

=M

a +1

+ M N
i i =0

a +1- i

a +1
i =0

M N a +1-i = f ( p a +1 )
i

(4)(1). Pentru fiecare p prim, fie M $i N solu#iile ecua#iei: X 2-f(p)X+(f(p))2-f(p2)=0 (evident M $i N sunt func#ii de p) c Fie g, hM a.. pentru orice p prim g(p)=M $i h(p)=N. Atunci f(p)=M+N=(g*h)(p) iar pentru a2:

114

( g * h)( p a ) =

i =0

M i N a -1 = ( M + N ) M i N a -1-i - MN M i N a - 2-i =
i =0 i =0

a -1

a -2

)[ f ( p ) - ( f ( p )) ] = f ( p ). = f ( p) f ( p ) + f ( p Cum fM deducem c f=g*h. (2)(4). Fie p un num r prim $i a1. Punem n ecua#ia din (2) m=p a $i n=p. Atunci f(pa+1)=f(p)f(pa)+f(pa-1)F(p). Dac particulariz m a=1 ob#inem F(p)=f(p2)-f(p))2. (4)(2). Dac (mn, m2n2)=1 atunci ((m, n), (m2, n2))=1 $i (mm2, nn2) = (m, n)(m2, n2). Astfel, pentru a proba (2) este suficient s ar t m c exist fM a.. pentru orice p prim $i a, b1, f ( p a + b ) =
min( a , b ) i =0

a -1

a -2

f ( p a -i ) f ( p b -i ) F ( p i ) (de

fapt este cazul n care F=mB2 cu B2Mc a.. B2(p)=f(p2)-(f(p))2 pentru orice p prim). F r a reduce din generalitate, s presupunem c ba ]i s facem inductie dup b. Dac b=1 totul este clar. Presupunem c b>1 $i c (2) este adev rat pentru b-1 $i orice ab -1. F(p)=[f(p) f(pa)-f(pa-1) B2(p)] f (pb-1)-f (pa) f (pb-2) B2(p) = f (pa) [ f (p) f (pb-1) - f (pb-2) B2(p)]-f (pa-1) f (pb-1) B2(p)=f (pa) f (pb) + f (pa-1) f (pb-1) F(p) (2)(3). Pentru orice m, n avem: f (mn / d 2 ) B (d ) =
d ( m,n )

Cum F=mB2, F(p2)=F(p3)==0 iar f (pa+b) =f (pa+1+b-1)=f (pa+1) f (pb-2)

= = =

d ( m,n )

D(

f(

m n , ) d d

m/d n/d ) f( ) m ( D ) B ( D ) B (d ) = D D

d ( m,n )e ( m,n ) de

f (m / e) f (n / e) m (e / d ) B (e) =

e ( m,n )

f (m / e) f (n / e) B (e) m (e / d ) = f (m) f (n)


de

Astfel func#ia B2Mc serve$te pe post de B cerut n (3). (3)(4). Dac p este prim $i alegem m=n=p, atunci ob#inem B(p)=(f(p))2-f(p2), adic B=B2. Fie a1. Dac alegem m=pa $i n=p ob#inem (4).3 115

Observa#ie Func#ia fA ce satisface una din condi#iile teoremei de mai sus poart numele de func#ie multiplicat$ special$ (Dup cum am observat nainte sk este o astfel de func#ie. Pentru sk avem c B=xk ; ntr-adev r, dac p este prim, atunci B(p)=( sk(p))2-sk(p2)=(1+pk)2-(1+pk+p2k)=pk=xk(p). Deci pentru orice m, n avem: s k (mn) = s k (m / d )s k (n / d ) m (d )d k [S.Ramanujan pentru
d ( m ,n )

k=0, n 1916] $i s k (m)s k (n) =

d ( m ,n )

d k s k (mn / d 2 ). [Busche-1906].

CAPITOLUL 9: RESTURI PTRATICE 1.Generalit$ i.Simbolul lui Legendre Fie m , m >1 un num r natural fixat. DEFINI'IA 1.1. Un num$r a3 cu (m, a)=1 se zice rest p!tratic modulo m dac$ ecua ia x2 a (m) are solu ie. n caz contrar a se zice non-rest p!tratic modulo m. n mod evident, dac a, b3 $i ab(m), atunci a este rest p tratic modulo m b este rest p tratic modulo m. Datorit acestei observa#ii este mai comod s lucr m n 3p dect n 3, distinc#ia f cndu-se n contextul n care se lucreaz (not m deseori elementele lui 3p prin 0, 1,, p-1). Observa#ii: 1. Fie p un num r prim; dac p=2 $i a3 este impar, a=2k+1 cu k3, atunci ecua#ia x2a (mod 2) are solu#ie pentru x=1 sau x=a. Deci orice num r impar este rest p tratic modulo 2. 2. Dac p este impar (deci p3), atunci a3 este rest p tratic modulo p restul mp #irii lui a la p este din 3* 2 (sau 3p* 2). Aici 3* 2 ={x2 | x3*} $i analog 3p* 2. ntr-adev r, dac a3 este rest p tratic modulo p, atunci exist x3 a.. x a(p) exist c3 a.. a-x2=cp a=cp+x2. Reciproc, dac putem scrie a=cp+r2, cu 0 r2 <p, atunci ecua#ia x2a (p) are solu#ie pe x=r.
2

116

n cele ce urmeaz prin p vom desemna un num r prim impar (p 3). Cum p -1 , func#ia s:3p*3p*, s ( x) = x 2
p -1 2

este morfism de

grupuri multiplicative. Cum s(x)2=xp-1=1 deducem c s(x)=1 (n 3p*) (deci s:3p* {1}) Mai mult : X
(p-1)/2

1. s(x)=-1 pentru un anumit x3*p (c ci n caz contrar polinomul -1 ar avea mai multe r d cini dect gradul s u). 2. Dac x=t23p*2, atunci s(x)=x(p-1)/2=(t2)(p-1)/2=tp-1=1 (reamintim c am

notat 3p*2={a2|a3p*}). Din cele de mai sus deducem c : 3p*2 ker s 3p* $i cum [3p*:ker s]= =|3p*/ ker s|=|Im s|=2 deducem c 2=[3p*:3p*2]=[3p*:ker s][ker s:3p*2], de unde [ker s:3p*2]=1, adic ker s=3p*2. DEFINI'IA 1.2. Numim simbolul lui Legendre morfismul de not * grupuri multiplicative s = p :3p {1}. a ( p -1) / 2 Deci , pentru orice a3p* (evident p?a, c ci a3p*). p = s (a) = a Mai mult : a 1 daca a este rest patratic mod ulo p (1) p = - 1 daca a nu este rest patratic mod ulo p n particular: -1 ab a b ( p -1) / 2 * (2) $i p = (-1) p pentru orice a, b 3p . p = p LEMA 1.3.(Gauss) Fie 3p*=XY, unde , 2 ,..., p - 1 } iar X = {1 2

p +1 Y ={ ,..., p - 1 } (evident XY=). 2

117

a g X={ a x | xX}. Atunci 3p*, fie a Pentru a p = (-1) , unde XY| g = |a Demonstra#ie S observ m la nceput c func#ia ma:3p*3p*, )=a x , pentru x3p*, permut doar elementele lui 3p*. ma( x Astfel, dac not m X X = {x1 ,..., x k }, Y = a X Y = { y1 ,..., y g } ,atunci X2Y2=aX, iar X=a X2Y2=, deci g+k=(p-1)/2.

Fie Z = {x1 ,..., x k , p - y1 ,..., p - y g

} X .

S observ m c elementele

lui Z sunt distincte dou cte dou (ca elemente ale lui 3p). ntr-adev r, dac exist i, j a.. xi=p-yjxi+yj=0 (n 3p). ns xi=ar, yj=as cu 1r, s(p-1)/2, deci a(r+s)=0 $i cum a0 deducem c r+s=0 ceea ce este imposibil deoarece 2 r+s <p-1. Deducem atunci c Z=X (c ci ZX $i |Z|=|X| ), p -1 = x1xk(p-y1)(p-yg)=(-1)g x1xky1yg= deci (n 3p) avem : 1 2 2 p -1 p -1 a (c ci X2 Y2=aX !) =(-1)g a(p-1)/212 , de unde =(-1)g a 2a 2 2 g (p-1)/2 (-1) a =1, de unde : a (-1) g =a (p-1)/2 = p .& COROLAR 1.4. Pentru orice num$r prim p impar (deci p3) avem: 2 ( p 2 -1) / 8 p = -1

( )

Demonstra#ie S observ m la nceput c (p2-1)/8 . ntr-adev r, dac p=8m+r (r=1, 3, 5, 7), atunci : p 2 - 1 (8m + r ) 2 - 1 r 2 -1 = = 2n + 8 8 8 (n ) $i

cum (r2-1)/8 pentru r =1, 3, 5, 7 totul este clar. Pentru 1 x<(p-1)/2, 2x<p-1. Atunci g din lema 1.3. a lui Gauss (pentru a=2) este num rul elementelor de forma 2x, 1 x<(p-1)/2 ce verific condi#ia p -1 p -1 2xY x>(p-1)/4, adic : g = -[ ] .Considernd p=8m+r , (r =1, 3, 2 4 5, 7), avem: 118

r -1 r -1 r -1 r -1 r -1 r -1 - [ 2m + - 2m - [ -[ ], ] = 4m + ] = 2m + 2 4 2 4 2 4 care ne duce la concluzia c g este par pentru r=1, 7 $i impar pentru r=3, 5, adic 2 g ( p 2 -1) / 8 3 g $i (p2-1)/8 au aceea$i paritate, de unde p = (-1) = -1 g = 4m +

( )

2. Legea reciprocit $ ii p$tratice n vederea demonstr rii legii reciprocit #ii p tratice, s nceput urm toarea lem : LEMA 2.1. Dac$ p !i q sunt dou$ numere prime impare (p, q3), distincte, atunci : ( p -1) / 2 jq ( q -1) / 2 jp p -1 q -1 + = 2 2 j =1 p j =1 q Demonstra#ie Notnd s ( p, q ) =
( p -1) / 2

stabilim la

, egalitatea din enun# j =1 p


p - 1 jq , este 2 p

jq

devine : s(p, q)+s(q, p)=(p-1)(q-1)/4. Este u$or de observat c pentru orice j=1, 2,,

num rul de numere naturale din intervalul (0, jq/p). Deci pentru fiecare j ca mai sus, [jq/p] este num rul acelor puncte laticiale din plan situate pe dreapta x=j (delimitate strict superior de dreapta y=qx/p $i inferior de y=0 ) Astfel s(p, q) reprezint num rul punctelor laticiale din interiorul dreptunghiului OABC (deci nesituate pe conturul lui OABC !) situate sub dreapta de ecua #ie y=(q/p)x. (vezi Fig.1)

119

y B(0,

q -1 ) 2

C(

p -1 q -1 , ) 2 2 q y= x p
x

x=j 0 A( Fig. 1 Analog, s(q, p) reprezint num rul punctelor laticiale din interiorul dreptunghiului OABC situate deasupra dreptei de ecuatie y=(q/p)x astfel c s(p, q)+s(q, p)=num rul de puncte laticiale din interiorul dreptunghiului OABC p -1 q -1 = $i astfel lema este probat . & 2 2 TEOREMA 2.2. (Legea reciprocit $ ii p$tratice) Dac$ p !i q sunt dou$ numere prime impare distincte, atunci : p -1 q -1 p q = 2 2 1 q p Demonstra#ie Revenim la nota#iile din Lema 1.3. a lui Gauss (numai c de data aceasta elementele xi $i yi vor fi privite ca numere ntregi, deci nu ca

p -1 ,0) 2

( )

elemente din 3p). Fie a = Avem

x j, b = y j
j =1 j =1

p -1 p2 -1 = . 2 8 xX Analog ca n demonstra#ia lemei lui Gauss vom avea :

x = 1 + 2 + ... +

120

zZ

z = x j + ( p - y j) = a - b + p g
j =1 j =1

$i

cum

X=Z

deducem

p2 -1 =a - b + pq 8 Acum, pentru j=1, 2,, (p-1)/2, fie t j restul mp r#irii prin p a lui jq. Evident ctul este [jq/p], deci jq =[jq/p]+t j. F cnd j=1, 2,, (p-1)/2 $i sumnd ob#inem :
( p -1) / 2 q ( p 2 - 1) = p s ( p, q ) + t j = p s ( p, q ) + 8 j =1 2

x j + y j sau
j =1 j =1

q ( p - 1) = p s ( p, q ) + a + b . 8 p2 -1 = a - b + p q deducem c Cum 8 (q - 1)( p 2 - 1) = p[ s ( p, q ) - g ] + 2 b 8 Deoarece p si q sunt primi impari $i (p2-1)/8 , deducem c q g s ( p,q) . Schimbnd rolul lui p cu s(p, q)-g0(mod 2), astfel c p = -1 = -1

( ) ( )

q deducem c

p q =

(-1)s (q, p) , de unde

p -1 q -1 p q s ( p ,q ) + s ( q , p ) = = 2 2 .3 1 1 q p Aplica#ie. Fie p=1009 $i a=45=325. Avem: 2 1009 -1 5 -1 5 5 1009 45 1009 9 3 = = = -1 2 2 = = = 1, 1009 1009 1009 1009 5 5 5

( )

( )

( )

deci 45 este rest p tratic modulo 1009 (adic 45 este p trat n 3*1009).

121

3.Alte cazuri particulare ale teoremei lui Dirichlet


p -1 2 Dup cum am v zut, pentru orice num r prim p, p = -1 8 (conform Corolarului 1.4.) De aici deducem c 2 este rest p tratic modulo p pentru p de forma

( )

8k1 $i non-rest p tratic pentru p de forma 8k3 (cu k *). PROPOZI'IA 3.1. Exist$ o infinitate de numere prime de forma 8n-1, n *. Demonstra#ie Fie n , n2; atunci num rul N=2(n!)2-1>1 are cel putin un divizor prim p impar care nu este de forma 8k+1 (c ci N este de forma 8k-1 iar dac to#i divizorii primi impari ai lui N ar fi de forma 8k+1, atunci $i N ar trebui s fie de aceea$i form ) 2(n!) 2 Atunci p|N 2(n!)2 12(p), de unde deducem c p =1. 2(n!) 2 2 n! 2 2 ns p = p p = p , deci p = 1, adic p trebuie s fie de forma 8k1. Cum p nu este de forma 8k+1 r mne doar c p prim trebuie s fie de forma 8k-1. Cum p | N deducem c p >n. Am probat deci c pentru orice n , n>1, exist un prim p>n de forma 8k-1. S presupunem acum c avem un num r finit de numere prime de forma 8k-1 $i anume q1, q2,, qt. Considernd num rul n=8q1qt-1 conform celor de mai nainte exist un num r prim de forma 8k-1 (adic un qi ) a.. qi >n, ceea ce este absurd. & PROPOZI'IA 3.2. Exist$ o infinitate de numere prime de forma 8n+3, n *. Demonstra#ie Fie n>1 $i a=p2 p3pn (unde pn este al n-lea num r prim). Cum a este impar, a2 va fi de forma 8t+1 iar N=a2 +2 va fi de forma 8t+3. Dac orice divizor prim al lui N este de forma 8t 1, N nsu$i este de aceast form absurd !. Deci N are cel pu #in un divizor prim impar p ce nu este de forma 8t+3 sau 8t+5. 122
2

-2 Cum p|N=a2 +2 deducem c a2 -2 (p) $i deci p = 1. - 2 - 1 2 ns , p = = p p

(-1) 2 (-1) 8

p -1

p 2 -1

-2 Dac p=8t+5 atunci p = -1 absurd, de unde concluzia c p este de forma 8t+3. ns din p|a2 +2 deducem p> p n $i astfel avem o infinitate de numere prime de forma 8t+3. 3 PROPOZI'IA 3.3. Exist$ o infinitate de numere prime de forma 8n+5, cu n *. Demonstra#ie Fie n>1 natural $i a=p2 p3pn. Cum a este impar, N=a2 +4 este de forma 8t+5. Dac to#i divizorii lui N ar fi de forma 8t1, atunci $i N ar fi de aceea$i form ceea ce este imposibil. Atunci N ar trebui s aib un divizor prim p de forma 8t+3 sau 8t+5. -4 Dac p=8t+3 atunci din p|N=a2 +4 a2-4(p), deci p = 1 $i astfel :
p -1 - 4 - 1 2 -4 p = p p = -1 2 $i cum p=8t+3 p = -1 -contradic#ie. Deci p este de forma 8t+5 $i astfel din p|a2 +4 $i a=p2 p3pn deducem c p>pn, de unde rezult imediat c avem o infinitate de numere prime de forma 8n+5.3 Observa#ie: Din legea reciprocit #ii p tratice deducem : 2

( )

COROLAR 3.4. Exist$ o infinitate de numere prime de forma 5n-1, cu n


*

Demonstra#ie Fie n *, n>1 iar N=5(n!)2-1. Cum N>1 $i este impar, atunci N, cum nu este de forma 5t+1, va avea cel pu#in un divizor prim p (p5) ce este de forma 5t+1. Evident p>n. 5 p Cum p|N 5(n!)2 1(mod p), adic p = 1 . Atunci $i 5 = 1 . Avem c p5 poate s fie de forma 5k 1 sau 5k2.

123

Dac

p=5k2,

atunci

p 5 1 2 = = 5 5 5 p

$i

cum

2 1 p = -1 , deducem c = -1 -contradic#ie. = 1, 5 5 p Cum am v zut c p nu poate fi de forma 5k+1 deducem c p trebuie s fie de form 5k-1. De aici corolarul rezult imediat. & Observa#ie Din demonstra#ia Corolarului 3.3. deducem c num rul prim p este de forma p=5k-1 (k ). Evident k=2t, deci p=10t-1. De aici rezult : COROLAR 3.5. Exist$ o infinitate de numere prime de forma 10n-1, n
*

.4

CAPITOLUL 10: FRAC'II CONTINUE 1.Mul imea numerelor ira ionale I Complementara n 9 a lui 7 o vom numi mul#imea numerelor

ira#ionale $i o vom nota cu I (deci I=9\7). (vezi Defini#ia 2.6. de la Capitolul 4). S demonstr m de exemplu c 2 I. Dac presupunem prin absurd c 2 7, atunci putem scrie 2 =m/n, cu m, n * $i (m, n)=1. Deducem imediat c 2n2=m2, adic m2 este num r par, deci m este par, adic m=2m1, cu m1 *. nlocuind deducem n2 =2m12, adic n=2n1, cu n1 n aceea c 2|m $i 2|n, contrar presupunerii c (m, n)=1. n2 $i xd , pentru orice d 7, atunci
n *

.Contradic#ia const
*

Observa#ie Mai general se demonstreaz , analog, c dac x7, n


n

x I. Deci

2 / 3 I,

2 I.

S mai demonstr m de exemplu c log23I. ntr-adev r, dac prin absurd log237, atunci exist m, n * a.. (m, n)=1 $i log23=m/n 2m/n =3 2m =3n, ceea ce este absurd deoarece (2, 3)=1. (mai general deducem c dac m, n
*

, (m, n)=1, atunci log mnI ). 124

Demonstra#ie Am v zut c (7, +, ) este corp.

LEMA 1.1. Dac$ x7, yI, atunci x+yI, iar dac$ x0 atunci xyI.

Notnd z=x+y, dac prin absurd z 7, am deduce c y=z-x7, ceea ce este absurd. Analog pentru partea a doua. 3 De exemplu, 1 2 I, 1 5 I. 2

LEMA 1.2. Opera iile de adunare !i nmul ire nu sunt opera ii interne pe I. Demonstra#ie Fie x=1+ 2 $i y=1- 2 . Cum 17 iar 2 I, conform lemei precedente deducem c x, yI. Cum x+y=2 iar xy=-1, deducem c x+y, xy7. 3 Observa#ie Cu toate acestea este posibil ca pentru x, yI s avem x+yI sau xyI (chiar simultan !). De exemplu, dac x= 2 , y= 3 , atunci x, yI $i xy= 6 I. S 6 =(z2 5)/27, absurd ! . demonstr m c $i x+yI. Fie pentru aceasta z=x+y= 2 + 3 . Dac prin absurd z7, atunci z2=5+2 6 , de unde am deduce c

S prezent m acum cteva rezultate importante legate de numerele ira#ionale. /tim c 1 1 1 1 e = lim (1 + ) n = lim (1 + + + ... + ) . n n n 1! 2! n!

TEOREMA 1.3. eI . Demonstra#ie S presupunem prin absurd c e7, adic e=a/b, cu a, . Pentru orice k , kb, cum b | k! deducem c num rul : a 1 1 1 c= k!( - 1 - - - ... - ) 3. b 1! 2! k! ns 1 1 1 1 1 1 1 0< c = + + ... < + + ... = = <1 1 k + 1 (k + 1)(k + 2) k + 1 (k + 1) 2 k +1 k 1k +1 Contradic#ia provine din aceea c c(0, 1), iar mai nainte am dedus c c3. n concluzie eI. & 125 b
*

Pentru a demonstra c $i alte numere importante sunt ira#ionale se utilizeaz un mic ,, truc , considernd o anumit func#ie (de obicei polinomial ). Pentru n nm2n. Pentru 0<x<1 avem 0<f(x)< 1 . n !
*

, fie f (x ) =

def

x n (1 - x) n 1 2n = c m x m , unde cm3, pentru n! n! m =n

Este clar c f(0)=0 $i c f(m)(0)=0 dac m < n sau m >2n, iar dac m! n m 2n avem f(m)(0)= cm3. n! Deducem ca o concluzie c f, ca $i toate derivatele sale iau valori ntregi n x=0 $i cum f(1-x)=f(x) aceea$i concluzie este valabil $i n x=1. Ca un corolar la acest mic truc s demonstr m: TEOREMA 1.4. Dac$ y7*, ey I.

Demonstra#ie Fie y=h/k7* $i s presupunem prin absurd c ey7. Atunci eh =eky 7 $i s punem eh =a/b, cu a, b *. Consider m (pentru n suficient de mare dup cum se va vedea n final)

func#ia : f ( x) =
def

x n (1 - x) n $i n!
( 2 n -1)

F ( x) = h 2 n f ( x) - h 2 n -1 f ( x) + ... - hf

( x) + f

(2n)

( x) , pentru orice x9.

)innd cont de cele de mai sus deducem c F(0), F(1) 3. De asemenea e hx F ( x) = e hx [hF ( x) + F ( x)] = h 2 n +1e hx f ( x) , oricare

ar fi x9.

1 1 Deducem c : b h 2 n +1e hx f ( x) dx = be hx F ( x) = aF (1) - bF (0) 3. 0 0 Cum ns 0<f(x)<1/n!, deducem c

0 < b h 2 n +1e hx f ( x) dx <


0

bh 2 n e h <1 n!

pentru

suficient

de

mare

(c ci

b ( h 2 e) n 0 pentru n) ceea ce e contradictoriu !. n! Deci eyI. & 126

Considernd o func#ie f asem n toare celei pentru care am demonstrat c e I pentru y7* putem demonstra:
y

TEOREMA 1.5. 3I. Demonstra#ie S demonstr m la nceput c dac n , g3[X], atunci f(x)=xn g(x) are toate derivatele sale n 0 ntregi divizibili prin n!. ntr-adev r, orice termen al lui g(x) este de forma cxj cu c, j3, c0, j0 iar termenul corespunz tor n f(x) este cxj+n. Astfel dac vom demonstra lucrul acesta pentru un singur termen al lui f, atunci el va rezulta n general pentru f. Pentru x=0 este usor de v zut c cxn+j $i derivatele sale sunt zero, cu o singur excep#ie $i anume la derivata sa de ordin j+n care este egal cu c[(j+n)!] $i cum j0 deducem c n! | c[(j+n)!]. S revenim acum $i s demonstr m c 4I. Presupunem prin absurd c 4 =a/b7 (cu a, b polinomul f ( x) =
*

) $i s consider m

x n (a - bx ) n b n x n (p - x) n = (n va fi pus n eviden# ceva mai trziu). n! n! n Consider m g(x)=(a-bx) ; conform celor remarcate la nceput putem trage concluzia c xn (a-bx)n $i toate derivatele sale calculate n 0 sunt ntregi divizibili prin n!. Prin urmare, mp r#ind prin n! deducem c f(x) $i toate derivatele sale calculate n x=0 sunt ntregi, deci f ( j ) (0)3, pentru orice j=1, 2, (cu f (0) =f ). Cum f(4-x)=f(x) deducem c (-1)j f ( j ) (4-x) = f ( j ) (x), pentru orice j1. Considernd x=0 deducem c f ( j ) (4)3, pentru orice j=1, 2,. Fie F(x) = f(x) - f(2)(x) + f(4)(x) - f(6)(x) ++ (-1)n f(2n)(x). Deducem c F(2)(x) = f(2)(x) - f(4)(x) + f(6)(x)- f(8)(x) ++ (-1)n-1 f(2n)(x) (2n+2) (c ci f (x) =0, f fiind polinom de grad 2n). Deducem c F(x) + F(2)(x) =f(x) iar de aici c F(0), F(4)3. Cum (F2(x) sin x-F(x) cos x)2=F22(x) sin x + F(x) sin x = f(x) sin x, deducem c :
p p
0

f ( x) sin x = ( F ( x) sin x - F ( x) cos x) 0 = F (p ) - F (0) 3.


c pentru n suficient de mare avem

Vom demonstra ns 0<


0

f ( x) sin x dx < 1 $i atunci contradic#ia va fi clar .

127

Cum pentru x[0, 4], f ( x) < astfel 0 <


p

p nan
n!

deducem c

f ( x) sin x <

p nan
n!

$i

f ( x ) sin x dx < n! p < 1 pentru orice n>n0 c ci 0

p nan

(pa ) n p 0 n!

pentru n. & n leg tur cu felul n care func#iile trigonometrice genereaz numere ira#ionale prezent m: TEOREMA 1.6. Fie q un multiplu ra ional de 3 (adic$ q=r3, cu r7). Atunci sin q, cos q, tg qI, cu excep ia cazurilor cnd tg q nu este definit iar cos q, sin q{0, 1/2, 1}, tg q{0, 1}. Demonstra#ie Pentru orice num r natural n vom proba prin induc#ie matematic existen#a unui polinom fn3[X] de grad n cu coeficientul dominant 1 a.. 2 cos nq =fn(2 cos q), pentru orice q9. Cum 2 cos 2q =(2 cos q)2-2 deducem c f1(x)=x $i f2(x)=x2 2. Cum 2cos(n+1)q=(2cos q)(2 cos nq)-2cos (n-1)q deducem c! fn+1(x)=xfn(x)-fn-1(x) si astfel prin induc#ie matematic existen#a polinomului fn este asigurat . Fie acum n * a.. nr3. Dac q=r 4 rezult c fn(2cos q)=2cos nq = =2cos nrq=2 (,,+ dac nr este par $i ,,- dac nr este impar). Astfel 2cos q este solu#ie a ecua#iei fn(x) 2=0. Eliminnd cazul cos q=0, cum 2cos q este r d cina unei ecua#ii de forma fn(x) 2=0 cu coeficientul dominant 1, dac 2 cos q7, cu necesitate 2cosq3*. Cum 1cos q 1 deducem c 2 cos q =1, 2, adic cos q{1,1/2}. Astfel, n cazul lui cos q teorema este demonstrat . n cazul lui sin q, dac q este multiplu ra#ional de 4, la fel este $i 4/2-q $i din identitatea sin q=cos (4/2-q) deducem concluzia teoremei pentru sin q. n final din identitatea cos 2q=(1-tg2q)/(1+tg2q) deducem c dac tgq7 atunci $i cos 2q7. )innd cont de cele stabilite n cazul lui cos q deducem c cos 2q=0, 1/2, 1. Dac cos 2q = 0, atunci tg q = 1; dac cos 2q = 1 atunci tg q = 0 iar dac cos 2q =-1, atunci tg q nu este definit .

128

Dac cos 2q = 1/2 atunci tg q{ 3 , 1 / 3 } $i cu aceasta teorema este demonstrat . 3 TEOREMA 1.7. e nu este ira ional p$tratic. Demonstra#ie Presupunem prin absurd c exist a, b, c3, nu toate nule, a.. ae +be+c=0 (vezi Defini#ia 3.10.). Cum eI avem a0 $i c0. S presupunem de exemplu c a>0. Atunci ae + b +ce -1 = 0, a>0, c0. Reamintim c S not m consider m $i bn = n! (-1) n -k -1 1 1 1 = + .... k! n + 1 (n + 1)(n + 2) (n + 1)(n + 2)(n + 3) k n +1 1 1 1 < bn < . Avem c 0 < n + 1 (n + 1)(n + 2) n +1 1 (-1) n = . e k 0 k!
2

Bn

= n!

(-1) k ; avem c Bn3, n=1, 2, $i s mai k =0 k!


n

Astfel: n!(ae + b + ce-1)=(aAn + bn! +cBn)+(aan + (-1)n+1cbn)=Cn+cn=0 (C) unde Cn=(aAn + bn! + cBn)3 $i cn=aan+(-1)n+1cbn , n1 . Alegem acum n a.. n 2a+|c| $i (-1)n+1c >0. 2a + c Cum a>0 avem c 0<cn=aan+(-1)n+1cbn< <1, ceea ce contrazice n +1 (C). 3 2.Numere algebrice !i numere transcendente DEFINI'IA 2.1. Un num$r a< se zice algebric dac$ exist$ f7[X], f0 a.. f(a)=0. Un num$r ce nu este algebric se zice transcendent. Evident, orice num r ra#ional este algebric . De asemenea, i= - 1 este algebric (fiind solu#ie a ecua#iei algebrice X2+1=0).

129

Dac un num r algebric a este r d cina unui polinom nenul f7[X] de grad minim, vom spune c a este de grad n (astfel un num r ra#ional este algebric de grad 1). TEOREMA 2.2. Mul imea numerelor algebrice este num $rabil$. Demonstra#ie Cunoa$tem c orice num r algebric a este solu#ie a unei ecua#ii: anXn +an-1Xn-1 + +a0 =0 cu ai3, 0in, nu to#i nuli. Dac not m N=n+|a0|++|an|, atunci cu necesitate N1. Pentru fiecare n fixat exist numai un num r finit de ecua#ii algebrice de forma celei de mai sus $i fiecare dintre acestea au numai un num r finit de solu#ii. n concluzie, num rul numerelor algebrice corespunz toare lui N este finit; fie EN mul#imea acestora. Cum mul#imea E a numerelor algebrice este o submul#ime a mul#imii U E N (care este num rabil ), deducem c E este
N 2

num rabil .3 Ca un corolar deducem imediat: TEOREMA 2.3. Att n < ct !i n 9, mul imea numerelor transcendente este num$rabil$. TEOREMA 2.4. (Criteriul lui Liouville ) Fie f!3[X] ireductibil de gradul r + 2, -!9 o r$d$cin$ a lui f !i p, q!3 cu q! *. Atunci exist$ un p c num$r real c > 0 ce nu depinde de p !i q a.. a - > r . q q Demonstra#ie Fie f =a0+a1X++arX r !3[X] polinomul minimal al lui + . Putem presupune c a p < 1 (c ci n caz contrar putem lua c=1). q

Atunci fie +=+ 1, + 2, , + r toate r d cinile lui f (conform Teoremei 2.3. acestea sunt n <). Avem : p p f q = ar a - q
i =2 r p p r p a a a (a + 1 + a i ) = c a i r

i=2

unde c' >0 este o constant ce nu depinde de p $i q. 130

p 1 Pe de alt parte, f q r $i astfel teorema este demonstrat . & q Observa#ie Criteriul precedent exprim faptul c , ntr-un anumit mod, numerele algebrice nu pot fi suficient de bine aproximate prin numere ra #ionale. COROLARUL 2.5. Num$rul - = nu este algebric). Demonstra#ie S ar t m la nceput c +"7. Dac prin absurd p += !7 cu p, q! *, atunci considernd un num r ntreg k.1 $i nmul#ind q rela#ia += p cu 3 k ! q ob#inem o rela#ie de forma a=b+q q n
n k +1 k +1 3

n 13

1
n!

este transcendent (adic$

1
n !- k !

cu

a, b !3. Este suficient s ar t m c num rul d = q suficient de mare. Un astfel de k exist

3 - n !+ k ! "3 pentru k

deoarece d este restul unei serii

convergente ; deci +"3. S presupunem acum c + ar fi algebric. Atunci polinomul minimal al s u ar avea gradul r .2. Fie c constanta din Criteriul lui Liouville de mai sus asociat lui +. Consider m k.1 ntreg $i s=
n =1

3 - n ! . Atunci avem
ob#inem inegalitatea

a-s =
3r
k!

n k +1

-n !

Lund

suficient

de

mare,

n k +1

3 - n! < c ceea ce contrazice Criteriul lui Liouville, absurd!. &

n continuare vom mai prezenta si alte exemple. TEOREMA 2.6. (Hermite) Num$rul e este transcendent. Demonstra#ie Fie I (t ) = et - x f ( x) dx , definit
0 t

pentru t0, unde

f9[X] este un polinom de grad m.

131

Integrnd prin p r#i ob#inem: I (t ) = e t f


j =1

( j)

(0 ) -

f ( j ) (t ) .
j =0

Se observ c dac prin f desemn m polinomul ce se ob#ine nlocuind coeficien#ii lui f cu valorile lor absolute, atunci : I (t ) e t - x f ( x)
0 t

dx te t f (t ) .

S presupunem acum prin absurd c e este algebric. Atunci exist a0,a1,,an3 cu a0 0 a.. a0+a1e ++anen =0. Fie f(x)=xp-1 (x-1)p(x-n)p unde p este un num r prim (care va fi convenabil ales). Atunci gradul lui f este (n+1)p-1. Fie J=a0I(0) + a1I(1) ++anI(n). Avem J = - a k f
j = 0k = 0 m n ( j)

(k ) .
( j )

ns pentru 1 k n avem f ( j ) (k) = 0 pentru j<p $i f j =Cp p! g( j - p )(k) pentru jp, unde g(x) = f(x)/(x-k)p.

(k) =

Atunci pentru orice j, f ( j ) (k) este un ntreg divizibil prin p!. j Mai mult, avem c f ( j ) (0)=0, pentru j<p-1 $i f ( j ) (0) = C p -1 (p-1)! h(j-p+1)(0) pentru j p-1, unde h(x)=f(x)/xp-1. Evident h(j)(0) este un num r ntreg divizibil prin p pentru j>0 $i h(0)=(-1)np (n!)p. Atunci pentru jp-1, f ( j ) (0) este un ntreg divizibil prin p! $i f ( p-1) (0) este ntreg divizibil prin (p-1)!, ns nu prin p pentru p>n. Rezult c J este un ntreg nenul divizibil prin (p-1)!, deci | J | / (p-1)!. Pe de alt parte, #innd cont de faptul c f (k).(2n)m $i m2np

deducem c | J | |a1| e f (1) ++|an| n en f (n) ep pentru un anumit c ce nu depinde de p. Alegnd p prim suficient de mare (a.. (p-1)!>c p) ajungem la o contradic#ie evident , de unde rezult c presupunerea c e este algebric este fals , rezultnd deci c e nu este algebric, adic este transcendent. & COROLAR 2.7. eI. Observa#ie De$i ira#ionalitatea lui e rezult din aceea c e este transcendent trebuie re#inut $i demonstra#ia precedent pentru faptul c e este ira#ional, fie $i numai pentru frumuse#ea metodei folosite.

132

TEOREMA 2.8. (Lindemann) 3 este transcendent. Demonstra#ie S stabilim la nceput a$a- zisa ,,identitate a lui Hermite: Fie f9[X] de grad n $i F(x)=f(x)+f 2(x)++f(n)(x). Atunci e x f (t )e -t dt = F (0)e x - F ( x) , pentru orice x9.
0 x

ntr-adev r, integrnd prin p r#i ob#inem rela#ia:


0 -t -x -t f (t )e dt = f (0) - f ( x)e + f (t )e dt 0 x x

Repet nd de n+1 ori integrarea prin p r#i ob#inem egalitatea:


0 -t -x f (t )e dt = F (0) - F ( x)e x

din care rezult acum identitatea lui Hermite. S revenim acum la demonstrarea transcenden#ei lui 4. Pe lng identitatea lui Hermite vom mai folosi $i ecuatia e4i +1=0. S presupunem prin absurd c 4 este algebric. Atunci g=4i este de asemenea algebric; fie n=gradul lui g $i g=g1, g2gn conjuga#ii lui g. Cum eg +1=0, avem
n

g (1 + e i ) = 0 de unde deducem c :
i =1 1 1

g e g +...+e g (1 + e i ) = ... e 1 1 k k =0
i =1
e 1= 0 e n =0

(1)

Presupunem c n rela#ia de mai sus sunt exact m exponen#i nenuli $i a=2n-m care sunt zero (a 1). Atunci, dac a1, , am sunt exponen#ii nenuli putem pune rela#ia (1) de mai sus sub forma a + e a1 + ... + e a m = 0 , a1. (2) Vom ar ta c numerele a1,, am formeaz mul#imea r d cinilor unui polinom y3[X] de grad m. Pentru aceasta s observ m c polinomul:

j ( x) = ... [ x - (e1g 1 + ... + e ng n )]


e1 =0 e n =0

considerat ca polinom n g1,, gn cu coeficien#i n 3[X] este simetric n g1,, gn, deci j(x)7[X]. Atunci r d cinile polinomului j(x) (de grad 2n ) sunt a1,am $i 0 (cu multiplicitate a). Deci polinomul x-a j(x)7[X] (de grad m) are drept r d cini pe a1,,am. 133

Dac r
a

este c.m.m.m.c al numitorilor coeficien#ilor acestui polinom

atunci y(x)=(g/x )j(x)=bmxm+ +b1x+b03[X] (bm>0, b00) are exact r d cinile a1,, am. n identitatea lui Hermite vom considera succesiv x=a1,, am. Dac adun m $i #inem cont de (2) ob#inem : - aF (0) - F (a k ) =
k =1 m k =1

e a k f (t )e -t dt
0

ak

(3)

De aici demonstra #ia transcenden#ei lui 4 merge ca $i n cazul transcenden#ei lui e. Pentru aceasta n (3) vom considera: 1 1 mn -1 n -1 n ( m +1) n -1 n -1 f ( x) = bm x y ( x) = bm x ( x - a 1 ) n ...( x - a m ) n (4) (n - 1)! (n - 1)! unde n este un num r natural ce va fi ales suficient de mare. Vom demonstra c alegnd pe f ca mai sus, din (3) vom ajunge la o contradic #ie. Ob#inem imediat rela#iile: f(l)(0) = 0, l = 0, 1,, n-2 mn -1 n f(n-1)(0)= bm b0 (5) F(0)=
( m +1) n -1 l = n -1

(l )

mn -1 n b0 + nA (A3) (0 ) = b m

Cum ak este o r d cin a lui f(x) de multiplicitate n ob#inem c f (l) (ak)= 0 , l =0, 1,,n-1, k=1, 2,, m . (6) Analog ca n cazul lui e derivata de ordin l a lui xn-1yn(x) are coeficien#i divizibili prin n!. Deci pentru l > n coeficien#ii lui f(l)(x) sunt ntregi $i divizibili mn -1 prin bm n. Atunci din (6) deducem c F (a k ) =
( m +1) n -1 l = n -1

(l )

mn -1 (a k ) = nbm F (a k ) (7), k=1,,m cu ;(z)3[z].

Numerele bk=bmak, k=1,,m sunt ntregi algebrici ce formeaz

mul#imea

r d cinilor unui polinom de grad m din 3[X] cu coeficientul dominant 1. mn -1 Mai mult, bm ;(ak) = H(bk), H3[X]. Atunci
k =1 mn -1 F (a k ) = H ( b k ) = B , B3. bm k =1 m m

(8)

134

Din (5), (7), (8) deducem c : aF (0) +


k =1 mn -1 + n(aA + B) F (a k ) = ab0 bm m

(9)

Fie acum n * a.. (n, b0bm)=1 $i n>1. Membrul drept al lui (9) este un ntreg nedivizibil cu n $i deci nenul, de unde : aF (0) + F (a k ) 1 .
k =1 m

(10)

S c ut m acum o majorare a membrului drept din (3). S presupunem c toate punnctele a1,,am sunt con#inute n cercul |x| R $i s not m
m max bm y ( x) = c , cu c nedepinznd de n.

x R Atunci max f ( x) x R Exist inegalitatea:


m

R n -1c n . (n - 1)! avem

deci n0 a.. pentru orice nn0 ce satisface (10) s


m ak

-x < c n dx me R e a k f ( x)e - x dx f ( x) e a k dx ( n 1 )! (n - 1)! k =1 k =1 0 k =1 0 0

ak

R n -1e R

m ak

( Rc) n

<1 (11) Din (10), (11) $i (3) deducem c 1<1 -absurd! . & 3. Frac ii continue Vrnd s construiasc un planetariu cu ro#i din#ate, Cristian Huyghens (matematician, fizician $i astronom, 1629-1695 ) a avut de rezolvat problema : care raport ntre num rul de din#i a doua ro#i care se angreneaz (egal cu raportul duratelor lor de rota #ie ) este mai apropiat de raportul a dintre duratele de rota #ie ale planetelor respective. Din motive tehnice, num rul de din#i de pe o roat nu putea s fie prea mare. O problem similar a ap rut la alc tuirea calendarului: Ce numar p de ani bisec#i (de 366 zile ) trebuie pus ntr-un ciclu de q ani pentru ca durata medie

135

q 365 + p p = (365 + ) zile, s fie ct mai aproape q q de durata real A=365,24219878..zile ? Calendarul iulian a ales q=4, p=1. Calendarul gregorian, dup care tr im introdus la sfr$itul secolului XVI, l-a aproximat mai bine pe A, alegnd q=400 $i p=97 ; anii bisec#i sunt acei multipli de 4 care nu sunt multipli de 100, excep#ie facnd multiplii de 400. Anul nostru calendaristic dureaz deci 365+97/400= =365,2425 zile. Alte alegeri, ca p=8, q=33, sau p=31, q=128, ar fi dus la 365,24(24) sau 365,24218, dar nu era comod s avem un ciclu de 33 sau 128 de ani. Asemenea probleme de aproximare cu numere ra #ionale apar n numeroase domenii. O solu#ie este dat de frac#iile continue. Dup cum vom vedea, frac#iile continue pot fi folosite cu succes $i la rezolvarea unor probleme care, cel putin aparent, nu au legatur cu aproximarea numerelor. p Fie a7. Atunci putem scrie a = , cu p3 $i q *. q a anului calendaristic, Ac = F cnd mai multe mp r#iri cu rest g sim c pentru un anumit k avem: p=a0q + q1 ,0 <q1 <q q=a1q1+ q2 ,0 <q2 <q1 q1=a2q2+ q3 ,0 <q3<q2 -------------------------qk-2=ak-1qk-1+ qk ,0 <qk<qk-1 qk-1=akqk , unde a03 iar a1,,ak *. Conform algoritmului lui Euclid, ultimul rest nenul qkeste cel mai mare divizor comun al lui p $i q. S observ m c numerele a0, a1,depind numai de a, nu $i de q 1 reprezentarea p/q: a 0=[a], a1= = , etc. q1 a - a 0 Cunoscnd cturile a0, a1,, an putem scrie: 1 a = a0 + 1 a1 + O. 1 ak Convenim s scriem asemenea frac#ii etajate sub forma : 1 1 1 a = a0 + + +---+ a1 a 2 ak + 136

(1)

n frac#ia de mai sus, a03 iar a1,, ak *. Scrierea lui a sub forma (1) nu mai este a$a de simpl dac a este ira#ional ; Procednd analog ca mai 1 a1= >1 $i din nou dac a1=[a1] a - a0 Putem scrie c sus
*

a0=[a]3. 1 atunci a2= >1. a 1 - a1 ob#inem

Evident

a = a0 +
1 a1

1 a1 1 a2

1 a2

. Continund procedeul ob#inem

scrieri intermediare de forma :

a = a0 +

+ ... +

1 an

a n +1

(2)

S observ m c procesul de scriere a lui a sub forma (2) poate continua atta timp ct an+1 . Dup cum am v zut, dac a7, pentru un anumit k , ak . Dac ns a7, acest proces se poate continua orict de mult, deoarece fiecare ak7. Se ob#ine astfel o frac#ie etajat infinit : 1 1 1 a = a0 + + + .... + + ... a1 a 2 an (3)

Semnul egal de mai sus este pus conven#ional : nu $tim deocamdat ce reprezint membrul drept. S comprim m $i mai mult scrierea frac#iilor etajate (1), (2), (3), notndu-le [a 0;a1, a2,,an] pentru (1), [a 0;a1, a2,,an,an+1] pentru (2), $i [a0;a1, a2,] pentru (3). Vom prezenta n continuare cteva propriet #i ale frac#iilor continue. Pentru o frac#ie continu [a0;a1, a2,,an,] (unde a03 iar an pentru n1) s not m : p p n = n =[a0;a1, a2,,an] qn Numerele 4n sunt, evident, ra#ionale $i se numesc redusele frac#iei continue.
*

Observa#ie Frac#ia continu [a0;a1, a2,,am,1] se poate scrie mai scurt [a0;a1, a2,,am+1]. Cu conven#ia ak2, scrierea [a0;a1, a2,,ak] a numerelor ra#ionale nentregi este unic .

137

p p =[a1;a2, a3,,an]. Se vede c leg tura =[a0;a1, a2,,an] $i q q p p p dintre cele dou numere este = a0 + . Dac este o frac#ie ireductibil , q q q p a 0 + q atunci $i este ireductibil , deci putem afirma c , dac $i p/q este o p Fie frac#ie ireductibil , atunci p = p a 0 + q $i q = p . (4) Aceast observa#ie arat c maniera natural de a calcula valoarea unei frac#ii continue finite este exact inversul algoritmului de dezvoltare n frac#ie continu . ntr-adev r, dac a=[a0;a1, a2,,an], atunci an=an/1 este o frac#ie ireductibil , deci formulele (4) permit calculul lui an-1=[an-1;an], apoi al lui an-2 = =[an-2;an-1], etc. Aceast modalitate de calcul poate deveni laborioas pentru n destul de mare $i nu sugereaz nimic despre calculul ,,valorii unei frac#ii continue infinite. PROPOZI'IA 3.1. Num$r$torii !i numitorii reduselor verific$ rela ile : p0 = a0, p1 = a0a1 + 1,, pn+1 = an+1pn+ pn-1 (n=1, 2, ) (5) q0 = 1, q1 = a1,, qn+1 = an+1qn + qn-1 (n=1, 2, ) a a a +1 p p 1 Demonstra#ie Avem : 0 = a 0 = 0 ; 1 = a 0 + = 1 0 $i 1 q1 a1 a1 q0 p2 a2 a (a a + 1) + a 0 a 2 p1 + p 0 = [a 0 ; a1 , a 2 ] = a 0 + = 2 1 0 = , deci q2 a 2 a1 + 1 a 2 a1 + 1 a 2 q1 + q 0 rela#iile (5) se verific pentru n=1. Presupunem c ele sunt adev rate pentru n=k-1 $i ar t m c sunt adev rate $i pentru n=k. Avem: p k +1 = [a 0 ; a1 ,......, a k +1 ] = [a 0 ;a 1 ] , unde a 1 = [a1 ; a 2 ......, a k +1 ] . q k +1 p p p Fie 0 , 1 ,..., k = a 1 redusele frac#iei a1 . Conform ipotezei de induc #ie, q 0 q1 qk p k = a k +1 p k -1 + p k -2 q = a q + q k k +1 k -1 k -2 Pe de alt parte, din (4), avem , q k +1 = p p k +1 = p k a0 + qk k p k = p k -1a 0 + q k -1 , q k = p k -1 p k -1 = p k -2 a0 + q k - 2 , q k -1 = p k -2 138

$i deci,

q k +1 = p k = a k +1 p k -1 + p k - 2 = a k +1 q k + q k -1 . + qk = a 0 (a k +1 p k -1 + p k - 2 ) + a k +1 q k -1 + q k -2 = p k +1 = a 0 p k = a k +1 (a 0 p k -1 + q k -1 ) + a 0 p k - 2 + q k - 2 = a k +1 p k + p k -1

Folosind principiul induc#iei complete, propozi #ia este demonstrat .& n demonstra#ie nu am folosit faptul c an+1 este natural, prin urmare, aplicnd rela#iile (5) cu an+1 n loc de an+1, ob#inem : PROPOZI'IA 3.2. Dac$ a=[a0 ; a1,,an , an+1] atunci + p n -1 p a a= n n +1 . q na n +1 + q n -1

(6)

Rela#iile de recuren# (5) permit calculul u$or al $irului reduselor unei frac#ii continue. Este comod s punem p-1=1 $i q-1=0 ; rela#iile (5) sunt valabile atunci $i pentru n=0. Redusele se ob#in completnd de la stnga la dreapta tabelul:

a p q 1 0

a0 p0=a0 q0=1

a1 p1 q1

a2 . . .

an+1

p . . . an+1pn+pn-1 q2 . . . an+1qn+qn-1

5 +1 . Avem a0=1, 2 5 -1 2 2( 5 + 1) 5 +1 a - a0 = , a1 = = = = a , deci a1=a0 $i a1=a. 2 4 2 5 -1 Este u$or de v zut c an=a $i an=a0=1, pentru fiecare n natural. Frac#ia continu ata$at este, deci [1;1,1,1]. S calcul m cteva reduse: Exemplu Fie a= a p q 1 1 1 1 2 1 1 3 2 1 5 3 139 1 8 5 1 13 8 1 21 13 . . . . . . . . .

1 0

PROPOZI'IA 3.3. Au loc rela iile : q n p n -1 - p n q n -1 = (-1) n , n 0, q n p n - 2 - p n q n - 2 = (-1) n -1 a n , n 1, (7 ) (8) (9) (10)

p n -1 - p n = p n-2 - p n =

(-1) n , n 1 q n q n -1 (-1) n -1 a n , n 2, q n q n-2

Demonstra#ie Deoarece q 0=1, p0=a0, q-1=0, p-1=1 avem q0 p-1 p0 q -1= =(-1)0, deci rela#ia (7) este adev rat pentru n=0. Presupunem c pentru un n avem qn pn-1 pn qn-1=(-1)n. Folosind (5), avem qn+1 pn- pn+1 qn=(an+1qn+qn-1)pn (an+1pn + pn-1)qn= - (qnpn-1 pnqn-1)=(-1)n+1, Deci am demonstrat prin induc#ie rela#ia (7). Folosind nti (5), apoi (7), avem : qn pn-2- pn qn-2=(anqn-1+qn-2)pn-2 (anpn-1 + pn-2)qn-2= an(qn-1pn-2 pn-1qn-2)=(-1)n-1an adic rela#iile (8). Rela#iile (9) $i (10) sunt simple transcrieri ale lui (7) $i (8) $i astfel propozi#ia este demonstrat .& O consecin# imediat a rela#iilor (9) $i (10) o constituie: PROPOZI'IA 3.4. Au loc inegalit$ ile : p 0 < p 2 < p 4 < ... < p 5 < p 3 < p 1 Fie a=[a0;a1,,an, an+1] un num r real oarecare. Folosind (6), avem q p - p n q n -1 p a + p n -1 p n (-1) n a - p n = n n +1 = n n -1 = q na n +1 + q n -1 q n q n (q na n +1 + q n -1 ) q n (q na n +1 + q n -1 ) Egalitatea ob#inut arat c redusele de ordin par sunt mai mici dect a, iar cele de ordin impar sunt mai mari dect a. ntruct an+1 an+1 avem $i 1 1 1 a -pn = = q n (q na n +1 + q n -1 ) q n (q n a n +1 + q n -1 ) q n q n +1 Egalitatea din mijloc este posibil numai dac an+1=an+1, deci dac a este ra#ional $i a=4n+1. Pe de alt parte an+1+1>an+1, deci: 1 1 1 a -pn = > = q n (q na n +1 + q n -1 ) q n [q n + (q n a n+1 + q n-1 )] q n (q n + q n+1 ) 140

Rezumnd cele de mai sus, am demonstrat : PROPOZITIA 3.5. Dac$ a=[a0 ; a1, , an, an+1] , atunci p 1 1 < a- n , q n (q n + q n +1 ) qn q n q n +1 egalitatea din dreapta avnd loc numai dac $ a=

(11)

p n +1 . q n +1 Suntem n m sur s d m sens egalit #ii din (3). Din (5) este u$or de dedus c , pentru frac#ii continue infinite, qn+1>qn , ncepnd cu n=1 $i deci qnn. Pornind de la un num r ira#ional a, $irul ( 4n)n1 aproximeaz din ce n ce mai bine num rul a. n limbajul analizei matematice, lim p n = a . Dac pornim de la
n

o frac#ie continu infinit , Propozi#ia 3.4., mpreun cu (9), garanteaz c $irul (4 n)n1 converge. L s m n seama cititorului s arate c frac#ia continu ata$at acestui num r este tocmai frac#ia continu de la care am plecat. Ideea demonstra#iei este urm toarea : Dac [a0;a1,, a2n]<b=[b0; b1]< [a0;a1,, a2n,a2n+1], atunci: b0=a0 $i [a1;a2,, a2n+1]<b1=[b1; b2]< [a1;a2,, a2n] S mai demonstr m o proprietate a reduselor: p PROPOZI'IA 3.6. Fie a01, n -1 =[a0;a1,, an-1] q n -1 !i pn =[a0;a1,, an]. qn q pn !i [an;an-1,, a1]= n . p n -1 q n -1 Demonstra#ie Proced m prin induc#ie dup n. a a + 1 p1 a0 p Pentru n=1, [a 0 ; a1 ] = 0 1 = , = 0 a1 q1 1 q0 Atunci [an;an-1,, a0]= a 0 a1 + 1 p1 q = , a1 = 1 a0 p0 q1 Presupunem afirma#ia adev rat pentru n. Atunci : q a q + q n-1 1 [a n+1 ; a n ,..., a1 ] = a n +1 + = a n+1 + n -1 = n +1 n [a n ; a n -1 ,..., a1 ] qn qn Tot cu ajutorul lui (5), avem $i Avem [a1 ; a 0 ] = 141

[a n +1 ; a n ,..., a 0 ] = a n +1 + ceea ce trebuia demonstrat. &

1 [a n ; a n -1 ,..., a 0 ]

= a n +1 +

p n -1 p = n +1 pn pn

Vom prezenta n continuare cteva chestiuni legate de aproximarea numerelor reale. Fie a un num r real. Problema aproxim rii lui cu numere ra#ionale are urm toarea interpretare geometric . n planul xOy consider m dreapta (d) de ecua#ie y=ax $i re#eua de puncte ,,laticiale din semiplanul drept, adic mul#imea punctelor de coordonate ntregi (q, p) cu q>0 (vezi Fig. 2). C ut m puncte P(q, p) pentru care p/q este aproape de a, adic puncte P(q, p) situate ,,aproape de dreapta (d). Aceast apropiere o putem m sura prin abaterea a pantele dreptelor (d) $i OP (de ecua#ie y= p dintre q

p x ), fie prin distan#a de la P la q

dreapta (d) sau, ceea ce este echivalent, prin lungimea |qa-p| a segmentului PQ, unde Q este punctul de pe dreapta (d) care are abscis cu P.

y (d) Q(q,+q) y=+x P(q,p) 0 Fig. 2 x

142

Vom spune c

p este o ,, cea mai bun$ aproximare de spe #a nti a q


p p p , cu 0< q q avem a <a. q q q

lui a dac pentru orice alt frac#ie Num rul dac

p se nume$te o ,,cea mai bun$ aproximare de spe #a a doua a lui a q qa - p < q a - p , pentru orice (q , p ) (q, p ) pentru care q q . Se

vede imediat c orice ,,cea mai bun aproximare de spe#a a doua este $i o ,,cea mai bun aproximare de spe #a nti . Ne ocup m aici numai de cele mai bune aproxim ri de spe#a a doua $i le vom numi pe scurt cele mai bune aproxim ri. PROPOZI'IA 3.7. Orice cea mai bun$ aproximare a lui a este o redus$ a frac iei continue a lui a. p o cea mai bun aproximare a lui Demonstra#ie Fie q a=[a0;a1,,an,.]. p p p Dac <a0 (=40) , atunci : 1 a - a 0 = a - n qa - p , deci q q qn n-ar fi o cea mai bun aproximare. Dac p p1 > (=41) atunci : q q1

a-

p p p 1 > - 1 , ( c ci avem urm toarea ordonare q q1 qq1 q

p0
deci

p1

q )

1 . q1 1 1 Pe de alt parte, din (11), = 1 a - a0 q1 a1 qa - p >

$i, din nou, p/q n-ar fi

o cea mai bun aproximare. Am stabilit deci c 40 p/q 41 Presupunem c p/q nu coincide cu nici o redus a lui a. Atunci p/q este cuprins ntre dou reduse 4n-1 $i 4n+1, cu rangurile de aceea $i paritate. Avem 143

p p n -1 1 $i q q n -1 qq n -1

p p p p n -1 1 < n - n -1 = q q n -1 q n q n -1 q n q n -1

de unde deducem qn<q . Pe de alt parte, p p p 1 a - n - n -1 , q q n q n -1 q n q n -1 deci qa - p $i din (11), 1 q n +1 q na - p n 1 q n +1

adic qn<q $i q na - p n qa - p , n contradic#ie cu faptul c p/q este o cea mai bun aproximare $i astfel propozi#ia este demonstrat . & Observa#ie. Dac a este ra#ional $i p/q nu este o redus a lui a, atunci p g sim redusa n , cu q na - p n qa - p $i qn<q. qn Este adev rat $i reciproca: PROPOZI'IA 3.8. Orice redus$ este o cea mai bun$ aproximare, cu excep ia eventual$ a redusei

p0=

p0 . q0

a0 nu este o cea mai bun 1 aproximare, c ci |1a-a0|=1/2=|1a-a0-1|. n schimb,41=a este, evident, o cea mai bun aproximare. p Demonstra#ie Examin m numai cazul a[a0 ; 2]. Fie m o redus a lui qm Observa#ie Dac a = [a0 ; 2], atunci 40 = a, cu m1. Consider m numerele | ya-x |, unde y *, y qm, iar x este [ya] sau [ya]+1. Fie | y0a-x0 | cel mai mic dintre ele. Dac minimul este atins de mai multe valori y, am notat cu y0 cea mai mic dintre ele; x0 este atunci unic determinat, deoarece, dac | y0a-x0 |=| y0a-x0-1|, atunci y0a-x0 = x0+1-y0a, deci 2x + 1 a= 0 este ra#ional. 2 y0 Fie a=[a0 ; a1,,an], cu an 2, frac#ia continu a lui a. Avem n 1 $i deoarece cazul [a 0 ; 2] l-am exclus, rezult fie an>2, fie an=2 $i n >1. Avem 144

2y0=qn=anqn-1+qn-2 $i 2x0+1=pn=an pn-1+pn-2 1 1 1 de unde qn-1<y0, dar | qn-1 a-pn-1| = = = | y0 a-p0 |, ceea ce ar qn 2 y0 2 contrazice alegerea lui y0. Num rul x0 este deci o cea mai bun aproximare a y0 x0 p = k . Cum $irul q1, q2, este strict y0 qk

lui a $i, conform teoremei precedente,

cresc tor, avem k m (c ci qk qm). Dac k=m, am terminat, dac , ns , k<m, atunci, folosind (11), avem: 1 1 1 1 qka - pk > = q ma - p m q k + q k +1 q m-1 + q m q m-1 + a m+1q m q m +1 ceea ce ar contrazice defini #ia lui y0. n prima parte a demonstra#iei am ar tat c , exceptnd numerele a=[a0; 2], lund un q * (n locul lui qm ), exist o cea mai bun aproximare x0 (deci o redus a lui a ) cu y0q. n cazul q=1, aceast cea mai bun y0 aproximare este 40 sau a0 + 1 $i deci, 40 este o cea mai bun aproximare a lui a, 1

exceptnd cazul cnd cnd q1=1, deci a=[a0; 1,] . & n continuare ne vom ocupa de dezvoltarea n frac ii continue periodice a numerelor ira ionale p$tratice. DEFINI'IA 3.9. Frac ia continu$ infinit$ [a0;a1,] se zice periodic! dac$ exist$ h * !i k cu an=an+h+1 pentru fiecare nk. Convenim s$ not$m o asemenea frac ie continu$ cu [a0 ; a1 ,..., a k -1, a k , a k +1 ,..., a k + h] . Pentru asemenea frac#ii continue putem calcula valoarea mai simplu dect ca limit a $irului de reduse. Exemplu Fie a= [1; 2] =[1;2,2,2,2].Avem a=[1;a1], unde

a1=[2;2,2,2,2]=[ [2] ].De asemenea a1=[2;a2], unde a2=a1, deci a1=2+1/a1, adic a12-2a1-1=0, de unde a1=1+ 2 . Revenind la a, ob#inem a=1+1/a1= 2 .

145

general,

dac

[a 0 ; a1 ,..., a k -1 , a k ,..., a k + h ] ,

atunci

ak= [a k ; a k +1 ..., a k + h ] =ak+h+1 $i, conform lui (6),

a=

p k -1a n + p k -2 p k + ha k + p k + h -1 = q k -1a n + q k - 2 q k + ha k + q k + h -1

Din a doua egalitate urmeaz c ak este r d cina unei ecua#ii de gradul doi cu coeficien#i ntregi : Aak2 +Bak + C = 0 iar prima egalitate ne d -q a + pk -2 a k = k -2 q k -1a - q k - 2 de unde : A(p k-2 - a qk-2)2 + B(pk-2 -a qk-2)(a qk-1 pk-1) +C(a qk-1 pk-1)2 = 0 deci $i a este r d cina a unei ecua#ii de gradul doi cu coeficien #i ntregi. DEFINI'IA 3.10. Numerele ira ionale, r$d$cini ale unei ecua ii de gradul doi cu coeficien i ntregi (nu to i nuli), se numesc ira ionale p!tratice. n anul 1770, Joseph Louis de Lagrange (1736-1813) a demonstrat urm$torul rezultat: PROPOZI'IA 3.11.(Lagrange) Un num$r ira ional este p$tratic dac$ !i numai dac$ frac ia sa continu$ este periodic$. Demonstra#ie Am ar tat deja c orice frac#ie continu periodic este un ira#ional p tratic. S presupunem acum c a este r d cin a ecua#iei cu coeficien#i ntregi Ax2+Bx+C=0, unde A0 $i 0<B2-4AC nu este p trat perfect. Fie a=[a0;a1,,an-1,an] . Cu rela#ia(6), avem p a + p n-2 a = n -1 n q n -1a n + q n - 2 $i, deci, A(pn-1 an+ pn-2)2 + B(qn-1an+ qn-2)(pn-1 an+ pn-1) +C( qn-1an + qn-2)2 = 0 adic an este r d cina ecua#iei Anx2 + Bnx + Cn=0, unde: 2 2 An = Ap n (12) -1 + Bp n -1 q n -1 + Cq n -1 Bn = 2 Ap n -1 p n - 2 + B( p n -1 q n -2 + q n -1 p n - 2 ) + 2Cq n -1 q n - 2
2 Ap n -2 2 Bp n - 2 q n - 2 + Cq n -2

(13)

Cn = + (14) S observ m nti c Cn=An-1. Din (7) deducem c pn-1qn-2 + qn-1pn-2 este impar $i deci B $i Bn au aceea$i paritate. Prin calcul direct se verific $i c 146

Bn2-4AnCn=(B2-4AC)( pn-1qn-2 + qn-1pn-2)2=B2-4AC. Folosind ns faptul c Aa2+Ba+C=0, rela#ia (12) se scrie: 2 2 2 2 An = Ap n -1 + Bp n -1 q n -1 + Cq n -1 - q n -1 ( Aa + Ba + C ) =


2 2 = A( p n -1 - q n -1 ) + B ( p n -1 - a q n -1 ) q n -1 =

(15)

= ( p n -1 - a q n -1 )( A( p n -1 + a q n -1 ) + Bq n -1 ) Cu ajutorul lui (11), vom avea 1 1 An A( p n -1 + a q n -1 ) + Bq n -1 A( p n -1 + a q n -1 ) + Bq n -1 qn q n -1 p p n -1 + a + B A n -1 - a + 2 a + B A (1 + 2 a ) + B q n -1 q n -1 Vedem de aici c $irul de ntregi An ia un num r finit de valori $i deci Cn(=An-1) ia un num r finit de valori; n fine, din cauza lui (15), an ia un num r finit de valori. Rezult c pentru anumi#i k, h, vom avea ak=ak+h+1. A Este u$or de dedus de aici c ak= ak+h+1, ak+1= ak+1+h+1 $i prin induc#ie, an=an+h+1 pentru n k, deci frac#ia continu a lui a este periodic . &

Cele mai simple frac#ii continue periodice sunt cele pur periodice.(adic cele pentru care a0=an+1 ). Fie deci a= [a 0 ; a1 ,.., a n ] o frac#ie continu pur periodic . Avem a0=an+11 $i a=[a0;a1,,an,a], deci, folosind (6), p na + p n -1 a= , adic : q na + q n -1 q n a2 + (qn-1- pn)a- pn-1 = 0. Pentru trinomul f(x) = qnx2 +(qn-1-pn)x-pn-1, avem f(-1) = qn - qn-1 + pn - pn-1 > 0, f(0) = - pn-1 < 0. Cum, evident, a > a0 0, deducem c cealalt r d cin a trinomului este cuprins ntre 1 $i 0. Evident a este de forma P+ D , iar cealalt r d cin este Q ~= a P- D $i Q

P- D P+ D . Pentru un ira#ional p tratic a= , vom nota Q Q ~ conjugatul lui a.. l vom numi pe a

DEFINI'IE 3.12. Num$rul ira ional p$tratic a se nume!te redus ~ (-1, 0). dac$ a > 1, iar a 147

Teorema care urmeaz a fost demonstrat n 1828 de Evariste Galois (1811-1832), pe atunci elev. PROPOZI'IA 3.13. (E. Galois) Frac ia continu$ a lui a este pur periodic$ dac$ !i numai dac$ este un ira ional p$tratic redus. Demonstra#ie Am v zut mai sus c orice frac#ie continu pur periodic este un ira#ional p tratic redus (vom prescurta n continuare i.p.r). Fie a un i.p.r. 1 ~ = 1 (-1,0), c ci a 1. Prin induc#ie, rezult >1 $i a Avem a1 = 0 1 ~-a a - a0 a 0 c an este i.p.r. pentru fiecare n. /tim c frac#ia continu a lui a este periodic . Dac nu este pur periodic , atunci a = [a 0 ; a1 ,..., a k -1 , a k ,..., a k + h ] , unde ak-1ak+h. Am v zut ns c ak-1 =[ak-1; ak] este i.p.r. $i la fel este ak+1=[ak+h; ak+h+1]=[ak+h; ak]. ~ = a k -1 + Avem deci a k -1

1 1 = a k -1 + ~ (-1, 0), a ak k
1

~ =a a k +1 k + h + ~ (-1, 0). ak Deducem de aici c 1 1 a k -1 ~ ,- a ~ -1- a k k $i

1 1 1 ak +h ~ ]. ~ ,- a ~ -1- a deci ak-1 = ak+h = [- a k k k Am ajuns la o contradic#ie, deci a= [a 0 ; a1 ,..., a h ] . & Ce se ntmpl dac ,,r sturn m perioada unui i.p.r.? PROPOZI'IA 3.14. Fie a = [a 0 ; a1 ,..., a n ] !i b = [a n ; a n -1 ,..., a 0 ] . 1 Atunci a = - ~ .

Demonstra#ie ntruct a = [a 0 ; a1 ,..., a n , a ] , folosind (6), avem, dup cum am mai v zut, qna2 + (qn-1-pn)a-pn-1=0. (16) 148

Cum analog, de unde

b = [a n ; a n -1 ,..., a 0 , b ] ,
~2

cu ajutorul propozi#iei 6 se deduce,

pn-1b2 + (qn-1-pn)b-qn = 0, ~ p n -1 b + (q n-1 - p n ) b - q n = 0 , ~ 1 2 1 - b 2 ~ ) + (q n -1 - p n )(- ~ ) - p n -1 =0 q n (- b b

1 $i, deoarece ecua #ia (16) are o singur r d cin pozitiv , a = - ~ . &

Cele mai simple ira#ionale p tratice sunt cele de forma

D , unde

D 7. Frac#iile lor continue, n cazul D>1, au propriet #i D7+ $i remarcabile: PROPOZI'IA 3.15. Fie D7, D>1, D 7. Atunci

D = [a 0 ; a1 ,..., a n ,2a 0 ] n plus, partea a1, a2,,an a perioadei este simetric$, adic$ ak=an+1-k, pentru 1kn. ~ =a - D (-1,0), Demonstra#ie Avem a =[ D ] deci a=a + D >1 $i a
0 0 0

deci a este i.p.r. $i [a]=2a0, deci a= [2a 0 ; a1 ,..., a n ] . Deducem de aici c : D = [a 0 ; a1 ,..., a n ,2a 0 ] $i, nc , -a0+ D = [0; a1 ,..., a n ,2a 0 ] , de unde = [a1 ; a 2 ,..., a n ,2a 0 ] - a0 + D Folosind propozi #ia 11, vom avea: 1 - ~ = [2a 0 ; a n ,..., a1 ] =a0+ D =a= [2a 0 ; a1 ,..., a n ]
not

b =

de unde rezult an+1-k=ak. Putem demonstra $i reciproca: Dac a= [a 0 ; a1 ,..., a n ,2a 0 ] ,

(a01),

unde

ak=an+1-k,

atunci

1 a+a0= [2a 0 ; a1 ,..., a n ] $i = [a1 ; a 2 ,..., a n ,2a 0 ] = [a n ; a n -1 ,..., a1 ,2a 0 ] $i, a - a0 ~ , adic n scrierea din Propozi#ia 11, vom avea a+a0=(-a+a0)~, deci a = -a

a=

P+ D P+ D -P+ D , avem = , de unde P=0, deci a = Q Q Q 149

D Q2

&

Pe noi ne intereseaz informa#ia pe care ne-o d Propozi#ia 3.15 despre frac#ia continu a lui Avem deci 5 =[2;4]. 2. S g sim frac#ia continu a lui a0=2, a1=1/( 7 -2)=( 7. 7 +2)/3 D n cazul D , cu D 7 . Exemple 1. S dezvolt m n frac#ie continu num rul a= 5 . a0=2, a1=1/ 5 -2= 5 +2, a 1=4, a2=1/a1-a1=1/ 5 -2= 5 +2=a1,

a1=4, a2=3/( 7 -1)=3( 7 +1)/6=( 7 +1)/2 a2=1, a3=2/( 7 -1)=2( 7 +1)/6=( 7 +1)/3 a3=1, a4=3/( 7 -2)=3( 7 +2)/3= 7 +2 a4=4, a5=1/( 7 -2)=a1, deci 7 = [2;1,1,1,4] . Acest $ir poate fi destul de lung:

991 = [31; 2,12,10,2,2,2,1,1,2,6,1,1,1,1,3,1,8,4,1,2,1,2,3,1,4,1,20,6,4,31,4,6,20,1,4,1,3,2,1,4,8, 1,3,1,1,1,1,2,1,1,2,2,2,10,12,2,62 ]. n continuare vom pune n eviden# un algoritm de dezvoltare a lui a= D n frac#ie continu (cu D Avem a1= 1 D - a0 = a0=[ D ], D + a0 D2 a0
*

a.. a7). deci D =a0+

a1

deci

D + b1 2 , unde b1=a0 $i c1=D-a 0 >0 (deoarece c1 1

a0=[ D ]). Avem D-b 0 =c1. Continund ob#inem: a1=[a1] $i a1=a1+ a2= 1
2

a1

,deci

a 1 - a1

150

1 D + b1 - a1 c1

c1 D + b1 - a1c1 D + b2 c2
2

c1 ( D + a1c1 - b1) D - (a1c1 - b1 )


2

c1( D + a1c1 -b1)


2 2 2 - a1 D - b1 c1 + 2a1b1c1

D + a1c1 - b1
2 1 - a1 c1 + 2a1b1

unde b2=a1c1-b1 $i c2=1-a 1 c1+2a1b1. Pentru n , n2, fie bn+1=ancn-b1 $i cn+1=cn-1-a 1 cn+2anbn $i s ar t m c pentru n2 : (1) D- b 2 n =cn-1cn. Vom proba (1) prin induc #ie matematic relativ la n2. 2 2 2 2 Pentru n=2 avem D-b 2 2 =D-(a1c1-b1) =D-b 1 -a 1 c 1 +2a1b1c1=
2 2 2 =c1-a 1 c 1 +2a1b1c1=c1(1-a 1 c1+2a1b1)=c1c2. 2

S presupunem c pentru n2 avem D-b 2 n =cn-1cn. Atunci:


2 2 2 2 2 2 2 D-b 2 n +1 =D-(ancn-bn) =D-b n -a n c n +2anbncn=cn-1cn-a n c n +2anbncn=cn(cn-1-a n cn + +2anbn)=cncn+1 $i astfel (1) este adev rat pentru orice n2. S ar t m acum c pentru orice n1: D + bn (2) an= cn Dup calculele de la nceput avem c (2) se verific pentru n=1, 2. Dac presupunem c (2) este verificat pentru n, atunci: cn c ( D + a n c n - bn ) 1 1 a n +1 = = = = = n a n - an D - (a n c n - bn ) 2 D + bn D + bn - a n c n - an cn

c n ( D + bn +1 ) = c n c n +1

D + bn +1 c n +1 $i astfel

(am #inut cont $i de (1) ), astfel c (2) este adev rat pentru orice n . n mod evident c1 . Atunci b1=a0= [ D ] < D Astfel 0 < ( D -b1)/c1 < 1 < ( D +b1)/c1. 151 0< D -b1 <1, deci 0< ( D -b1)/c1 <1. Cum a1>1 deducem c ( D +b1)/c1 >1.

S ar t m acum c pentru orice n : (3) 0 < ( D -bn)/cn < 1 < ( D +bn)/cn. (pentru n=1 (3) este adev rat datorit celor stabilite mai sus). S presupunem c (3) este adev rat pentru un anumit n $i s o prob m pentru n+1. D + bn +1 Conform cu (2) avem = a n +1 > 1 astfel c : c n +1
2 D - bn +1 D - bn +1 = = c n +1 c n +1 ( D + bn +1 )

cn D + bn +1

cn D + a n c n - bn

1 D - bn + an cn

de unde deducem c 0 <

D - bn +1 <1. (#innd cont $i de ipoteza de induc #ie ) c n +1 D -bn<0

Astfel (3) este adev rat pentru orice n . Dac cn<0 pentru un anumit n , atunci din (3) deducem c $i D +bn<0, deci 2 D <0 absurd!. Deci cn >0 pentru orice n n consecin# pentru orice n
* *

. D -bn< D +bn $i astfel bn>0

D -bn<cn< D +bn, deci

Din (3) deducem c bn< D $i astfel cn< D +bn<2 D . Din observa#ia de mai nainte deducem c num rul perechilor (b n, cn) este mai mic dect 2D. D + bn numai un num r finit Astfel, printre termenii $irului an= cn dintre ei sunt diferi#i, fiecare dintre ace$tia fiind mai mici dect 2D. Astfel cel pu#in doi termeni ai $irului (an)n1 sunt egali.

a n +1

Deci exist k, s a.. k, s<2D $i (4) ak=ak+s. Deoarece 1 = pentru n1, din (4) deducem c ak+1=ak+s+1 $i mai general, a n - [a n ]

an=an+s pentru nk . Astfel $irurile (an)n1 $i (an)n1 sunt periodice (c ci an=[an] pentru n1). D - bn = Fie (5) a n pentru n1; #innd cont de (1) deducem imediat cn 1 c an = [ ] pentru orice n1. x n +1 152

a k -1

= an + k $i deci pentru k>1 avem Mai mult, cum ak=ak+s deducem c a n 1 1 1 ] = a k + s -1 . )innd cont de rela#iile an=an+ $i ak=ak+s =[ ]=[ +s a n +1 xk xk

deducem c ak-1=ak+s-1. Repetnd ra#ionamentul anterior pentru k>2 ob#inem c ak-2=ak+s-2. Astfel an+s=an $i an+s=an pentru orice n *. Deducem imediat formulele: 1 1 1 1 1 1 1 = as + + ... + + a 1 = a1 + + ... + + $i . a s -1 a1 a2 a s a1 a1 1 ( ) x1 Deoarece a1>1 $i 1 a1 > 1 aceste ultime relatii ne dau : $irul a1, a2, , as-1 este

as=2a0=2[ D ], a1=as-1, a2=as-2, , as-1=a1. (adic simetric)

)innd cont c dac x9 $i k *, atunci [x/k]=[[x]/k] avem (conform D + bn [ D ] + bn a + bn ], adic cu rela#iile (1)) : an=[an]=[ ]=[ ]=[ 0 cn cn cn an=[ a 0 + bn ] pentru orice n1. cn Rezumnd cele expuse mai nainte ob#inem urm torul algoritm de D (cu D
*

dezvoltare a lui

a..

D 7) n frac#ie continu .

Alegem a0=[ D ], b0=1, c0=1 $i apoi construim sirurile (an)n0, (bn)n0 $i (cn)n0 cu ajutorul recuren#elor : a + bn -1 a n=[ 0 ] c n -1 (6) b n=an-1cn-1-bn-1 pentru n1 2 D - bn c n= c n -1 Construim apoi $irul (b2, c2), (b3, c3) $i g sim cel mai mic indice s D = [a 0 ; a1 ,..., a s ] .

pentru care bs+1=b1 $i cs+1=c1. Atunci

Observa#ie Conform unei teoreme a lui T. Muir (vezi O. Perron: Die Lehre von den Kettenbrchen 1, Stuttgart 1954) , dac num rul s de termeni ai perioadei este par, atunci k=s/2 este cel mai mic indice pentru care b k+1=bk, pe 153

cnd dac s este impar atunci k=(s-1)/2 este cel mai mic indice pentru care ck+1=ck. Practic se procedeaz astfel: Pentru a= D (cu D * a.. D 7) alegem a0=[ D ], b0=0, c0=1 !i apoi construim prin recuren $ !irurile (an)n0, (bn)n0 !i (cn)n0 cu ajutorul formulelor: 2 a + bn -1 D - bn (7) bn=an-1cn-1-bn-1, cn= , an-1= [ 0 ], pentru n1. c n -1 c n -1 Calculele se continu$ pn$ cnd bn+1=bn sau pn$ cnd cn+1=cn. Dac$ bn+1=bn, atunci D =[a0 ; a1,, an-1, an, an-1, ,a1, 2a0] (adic$ lungimea perioadei minime este par $ ). D =[ a 0 ; a1 ,..., a n , a n ,..., a1 ,2a 0 ] Dac$ cn+1=cn, atunci lungimea perioadei minime este impar $ ). D + bn Numerele bn, cn sunt cele din scrierea lui an= . cn Exemple 1. Fie D=1009 $i a= 1009 . Avem a0=[ D ]=[ 1009 ]=31, b0=0, c0=1. Conform recuren#elor (6) sau (7) avem: 1009 - b12 (adic$

b1=a0c0-b0=a0=31, c1=

c0

1009 - 312 1009 - 961 =48, = 1 1

a + b1 31 + 31 a1= 0 = =1. Apoi: c1 48 b2=a1c1-b1=17, c2=


2 a + b2 31 + 17 1009 - b2 =15, a2= 0 = =3 c2 c 2 15

Aplicnd din nou recuren#ele (6) $i (7) g sim 2 1009 - b3 b3=a2c2-b2=28, c3= =1=c2. c2 Conform algoritmului descris mai nainte avem iar a 3 = 28 + 1009 . 15 154 1009 = [31;1,3,3,1,62] ,

2. Fie a , a3, D=a2-2 $i a= D = a 2 - 2 . Cum (a-1)2 = a2-2a+1 < a2-2 < a2, deducem c a0=[ a 2 - 2 ]=a-1. Deci, b1=a0=a-1, c1=D-a 0 =a2-2-(a-1)2=2a-3, a + b 2a - 2 1 a1= 0 1 = = 1 + =1 c1 2a - 3 2a - 3 Continu m b2=a1c1-b1=2a-3-(a-1)=a-2, 2 D - b2 a 2 - 2 - ( a - 2) 2 4 a - 6 = = =2 c2= c1 2a - 3 2a - 3 a + b2 a - 1 + a - 2 3 a2= 0 = a - = a-2 = 2 2 c2 Apoi b3=a2c2-b2=(a-2)2-(a-2)=a-2; 2 D - b3 a 2 - 2 - ( a - 2) 2 4 a - 6 = = = 2a-3 c3= c2 2 2 a + b 3 a -1 + a - 2 a3= 0 = =1 c 3 2a - 3 b4=a3c3-b3=2a-3-(a-2)=a-1, 2 D - b4 a 2 - 2 - (a - 1) 2 = =1 c4= 2a - 3 c3 a + b4 a - 1 + a - 1 a4= 0 = = 2a-2. 1 c4 n sfr$it, 2 D - b5 a 2 - 2 - (a - 1) 2 b5=a4c4-b4=2a-2-(a-1)=a-1=b1, c5= = 2a-3=c1 = c4 1 Din cele expuse mai nainte avem s=4 astfel c : a 2 - 2 = a - 1;1, a - 2,1,2a - 2 . Analog se ob#ine a . 155 a2
2

] + 1 = [a; 2a ] $i

a 2 + 2 = a; a,2a pentru orice

Observa#ie. Acest paragraf a fost redactat n cea mai mare parte dup lucrarea [10]. CAPITOLUL 11: TEOREME DE REPREZENTARE PENTRU NUMERE NTREGI 1 Reprezentarea unui num$r natural ca sum$ de dou$ p$trate de numere ntregi. Pentru un num r natural n, prin d(n) vom nota num rul divizorilor lui n iar prin da(n) num rul divizorilor d ai lui n cu proprietatea c d<a (4). Astfel, d1(n) reprezint num rul divizorilor de forma 4k+1 ai lui n iar d 3(n) num rul divizorilor de forma 4k+3 ai lui n (k ! ). Conform teoremei fundamentale a aritmeticii pe n l putem scrie sub forma n = 2 k n1 n 2 cu k! , n1 = p r iar n 2 = q s .
p prim p 1 (4 ) q prim q 3 (4 )

n cadrul acestui paragraf vom da r spuns la urm toarele chestiuni : P1. Pentru care numere naturale n exist x, y!3 a.. n=x2+y2 (C). P2. n caz c pentru n fixat ecua#ia (C) are cel pu#in o solu#ie atunci s se determine num rul tuturor solu#iilor sale. Observa#ie Dac ecua#ia (C) are o solu#ie (x, y) n (C) va avea solu#iile (x, y). Astfel : i) Dac x=y=0 atunci cu necesitate n=0 $i ecua#ia (C) are o unic solu#ie: (0, 0). ii) Dac x60 $i y=0 atunci solu#ia (x, 0) din solu#ii n 3 3 $i anume: (x, 0), (0, x), (-x, 0) $i (0, -x). iii) Dac x=0 $i y60 atunci solu#ia (0, y) din iv) Dac x60, y60 $i x6y atunci solu#ia (x, y) din genereaz de genereaz opt asemenea patru solu#ii n 3 3 $i anume: (0, y), (y, 0), (0, -y), (-y, 0). solu#ii n 3 3 $i anume: (x, y), (y, x), (-x, y), (y, -x), (x, -y), (-y, x), (-x, -y), $i (y, -x). 156 genereaz patru , atunci n 3 3 ecua#ia

v) Dac x60, y60 $i x=y atunci solu#ia (x, x) din

genereaz

patru solu#ii n 3 3 $i anume: (x, x), (-x, x), (x, -x) $i (x, -x). Aceast observa#ie ne arat c atunci cnd vorbim despre num rul de solu#ii pentru ecua#ia (C), trebuie s specific m neap rat urm toarele: a) Dac este vorba de num rul de solu#ii din sau din 3 3. b) Ce n#elegem prin solu#ii distincte ? (altfel spus, dac solu#iile (x, y) $i (y, x) pentru x6y sunt considerate distincte sau nu) . Pentru a nu crea confuzii n cadrul acestei lucr ri vom #ine cont de ordinea termenilor n cadrul solu #iei (x, y) (pentru x6y) urmnd ca atunci cnd nu #inem cont de lucrul acesta s -l men#ion m expres. Exemple 1. Ecua#ia x2+y2=1 are dou solu#ii n : (1, 0) $i (0, 1) pe cnd n 3 3 are patru solu#ii: (1, 0), (0, 1), (-1, 0) $i (0, -1). Dac nu #inem cont de ordinea termenilor concluzion m c ecua#ia x2+y2=1 are o unic solu#ie n (pe (1, 0) $i (-1, 0)). (pe (1, 0)) pe cnd n 3 3 are dou solu#ii o solu#ie unic $i anume pe (1, 1), pe

2. Ecua#ia x2+y2=2 are n

cnd n 3 3 are patru solu#ii $i anume : (1, 1), (1, -1), (-1, 1) $i (-1, -1). Dac nu #inem cont de ordinea termenilor concluzion m c ecua#ia x2+y2=2 are n 3 3 trei solu#ii $i anume : (1, 1), (-1, 1) $i (-1, -1). 3. Ecua#ia x2+y2=5 are n dou solu#ii: (1, 2) $i (2, 1) pe cnd n 3 3 are opt solu#ii: (1, 2), (1, -2), (-1, 2), (-1, -2), (2, 1), (-2, 1), (2, -1), (-2, -1) Dac nu #inem cont de ordinea termenilor concluzion m c ecua#ia x2+y2=5 are o unic solu#ie n (pe (1, 2)) pe cnd n 3 3 are patru solu#ii: (1, 2), (-1, 2), (1, -2), $i (1, -2). LEMA 1.1. Dac$ p este un num$r prim de forma 4k+1, atunci p - 1 ! + 1 0 ( p ) . 2 Demonstra#ie Scriind c p -1 p + 1 ( p - 1) != .... ( p - 1) = 1 2 .... 2 2 p - 1 p -1 = ! ( p - 1) ( p - 2 ) ... p 2 2 deducem imediat egalit #ile modulo p : 157
2

p -1 p - 1 p -1 p -1 ( p - 1) != ! (- 1) 2 != ! . 2 2 2 Conform teoremei lui Wilson ( p - 1) ! +1 0 ( p ) astfel

p - 1 ! + 1 0 ( p ) . & 2 LEMA 1.2. Dac$ p! este un num$r prim iar a!3 a.. p?a, atunci p a.. la o alegere convenabil $ a exist$ numerele naturale nenule x, y < semnelor + sau s$ avem ax y 2 0 (p) . Demonstra#ie Dac m=[ p ], atunci (m+1)2>p $i consider m mul#imea X={ax-y D0*x, y*m}. Cum DXD=(m+1)2>p, rezult c exist dou perechi diferite (x1, y1), (x2, y2)!X cu x1.x2 $i pD(ax1-y1)-(ax2-y2)= =a(x1-x2)-(y1-y2). Egalitatea x1=x2 este imposibil , c ci n caz contrar ar rezulta c pDy1-y2 (lucru imposibil c ci 0 * y1, y2 * m * p <p). De asemenea, egalitatea y1=y2 este imposibil , c ci n caz contrar ar rezulta pDa(x1-x2), deci pDx1-x2 imposibil (c ci 0*x1, x2 * m * p <p). Deci x=x1-x2! * (dac Cum x=x1-x2*x1 *m< x<0, atunci not m x=x2-x1) $i cum y1-y2 !3*, exist o alegere convenabil a semnelor + sau a.. y =(y1-y2)! *. p , deducem c 0<x, y < p $i astfel num rul a semnelor + $i este egal cu axb (care la o alegere convenabil a(x1-x2)-(y1-y2)) se divide prin p. & TEOREMA 1.3. (Fermat) Orice num$r prim p de forma 4k+1 se poate scrie ca suma p $tratelor a dou $ numere naturale. p -1 Demonstra#ie Consider m a = ! . Evident, a! * $i (a, p)=1. 2 Conform Lemei 1.2., exist o alegere convenabil a semnelor + $i a.. axy<0(p). Atunci a2x2-y2=(ax+y)(ax-y)<0(p) $i conform Lemei 1.1. a2+1<0(p), de unde deducem c a2x2+x2<0(p) iar de aici c (a2x2+x2)-(a2x2-y2)=x2+y2<0(p), adic putem scrie x2+y2=kp cu k! *. 158

Cum x, y < p deducem c x2+y2 < 2p, adic kp <2p, deci k <2, adic k=1 (c ci x, y! *). Deducem c p=x2+y2 $i astfel Teorema lui Fermat este complet demonstrat .& COROLAR 1.4. Dac$ n! * con ine n descompunerea sa n factori primi numai numere prime de forma 4k+1, atunci n se poate scrie sub forma n=x2+y2 cu x, y! . Demonstra#ie Totul rezult din Teorema 1.3. $i din aceea c un produs finit de expresii de forma x2+y2 este de aceia$i form (conform identit #ii (x2+y2)(z2+t2)=(xz+yt)2+(xt-yz)2 ). & Vom demonstra acum c scrierea unui num r natural ca sum de dou p trate de numere naturale este unic , dac nu #inem cont de ordinea termenilor. n fapt, vom demonstra o propozi #ie mai general : PROPOZI'IA 1.5. Fie a, b! . Dac$ un num$r natural prim p se scrie sub forma p=ax2+by2 cu x, y! atunci aceast$ scriere este unic$ (cu conven ia ca n cazul n care a=b=1 s$ nu inem cont de ordinea termenilor). Demonstra#ie S presupunem c p are dou descompuneri: 2 2 2 2 p=ax +by = ax1 + by1 cu x, y, x1, y1! . Atunci cum p2=(axx1+byy1)2+ab(xy1-yx1)2=(axx1-byy1)2+ab(xy1+yx1)2
2 2

(axx1+byy1)(xy1+yx1)=(ax +by )x1y1+

(ax

2 1

+ by ) xy=p(x1y1+xy)
2 1

$i

deducem c pDaxx1+byy1 sau pDxy1+yx1. Dac pDaxx1+byy1, atunci din prima reprezentare a lui p deducem c xy1-yx1=0 $i deci xy1=yx1 , p=axx1+byy1 , px=(ax2+by2)x1=px1, de unde x=x1 $i atunci y=y1. Dac pDxy1+yx1, atunci din a doua reprezentare a lui p deducem c axx1-byy1=0 $i p2=ab(xy1+yx1)2, de unde a=b=1. Vom avea deci p=xy1+yx1 $i xx1-yy1=0, de unde px=(x2+y2)y1=py1, 2 2 adic x=y1 $i din p=x2+y2= x1 + y1 , deducem c y=x1 (astfel c n acest caz descompunerile se pot deosebi doar prin ordinea termenilor). & Observa#ii 1. Din propozi #ia de mai nainte deducem c dac num rul natural n poate fi reprezentat n cel pu#in dou moduri diferite ca sum de dou 159

p trate de numere naturale (cu condi #ia s nu consider m diferite descompunerile ce se deosebesc numai prin ordinea termenilor), atunci cu necesitate n nu este prim. De exemplu, din egalit #ile 2501=12+502=102+492 deducem c num rul 2501 nu este prim. 2. Dac num rul n are doar o singur descompunere ntr-o sum de dou p trate de numere naturale, nu rezult cu necesitate c n este prim. De exemplu, se demonstreaz cu u$urin# c numerele 10, 18 $i 45 au descompuneri unice sub forma 10=12+32, 18=32+32, 45=32+62 $i totu$i ele nu sunt numere prime ( se subn #elege c nu am #inut cont de ordinea termenilor). Putem acum r spunde la chestiunea P1 formulat la nceputul paragrafului : TEOREMA 1.6. (Fermat-Euler) Un num$r natural n (scris sub forma n=2kn1n2 ca la nceputul paragrafului) se poate scrie sub forma n=x2+y2 cu x, y! dac$ !i numai dac$ to i exponen ii s din scrierea lui n2 sunt numere pare. Demonstra#ie Revenim la scrierea lui n sub forma n=2kn1n2 cu k! , n1 = p r $i n 2 = q s .
p prim p 1 (4 ) q prim q 3 (4 )

Cum 2=12+12 iar conform Teoremei 1.3. fiecare factor prim p <1(4) din scrierea lui n1 se scrie sub forma x2+y2 cu x, y! deducem imediat c n1 se poate scrie sub aceia$i form $i aceia$i proprietate o va avea $i 2kn1 (adic 2k n1=z2+t2 cu z, t! ). Dac presupunem c fiecare exponent s din scrierea lui n2 este par, atunci n mod evident n2=m2 cu m! $i atunci n=2kn1n2= =(z2+t2)m2=(zm)2+(tm)2. Reciproc, fie n! ce se poate scrie sub forma n =x2+y2 cu x, y! $i s demonstr m c dac qs este cea mai mare putere a unui num r prim q<3(4) ce intr n descompunerea n factori primi a lui n (de fapt a lui n2) atunci cu necesitate s este par. Presupunem prin absurd c s este impar. Dac d=(x, y), x y n 2 2 $i y1 = , n1 = 2 ob#inem c n1 = x1 + y1 atunci d2Dn $i dac not m x1 = d d d cu (x1, y1)=1.

160

Conform presupunerii s este impar iar d 2 (prin care am mp r#it egalitatea n=x2+y2 ) con#ine eventual o putere par a lui q, deducem c q|n1 $i c q nu divide simultan pe x1 $i y1 (s zicem c q?y1). 2 2 2 2 Privind acum egalitatea n1 = x1 + y1 n 3q deducem c 0 = x1 + y1 $i cum am presupus c q?y1 deducem c -1 x1 unde q = q

-1 2 y1

2 -1 y1 0 = x1

( )

-1 + 1 x1 y1

= -1 de

=1.
q -1 -1 2 $i cum q = (- 1)

ns n cadrul Capitolului 9 am stabilit c qA3(4) deducem c q -1 este impar, astfel c 2

-1 q = -1 , absurd. Deci s este par. Ra#ionnd inductiv deducem c to#i exponen#ii s din

descompunerea lui n2 sunt pari $i cu aceasta teorema este demonstrat . & Pentru a r spunde la chestiunea P2 de la nceputul paragrafului avem nevoie s reamintim anumite chestiuni legate de aritmetica ntregilor lui Gauss. 3[i]={a+bi | a, b!3}. Se cunoa$te faptul c ( 3[i], +, ) este un inel comutativ n care U( 3[i], +, ) ={1, i}, precum $i faptul c elementele prime din 3[i] sunt (pn la o multiplicare cu 1 sau i ) urm toarele : 1) 1i 2) Numerele prime p din 3 cu pA3(4) 3) Numerele de forma a+bi cu a, b ! * $i a2+b2=p, unde p este un num r natural prim $i pA1(4). Reamintim c descompunerea numerelor din 3[i] n factori primi este unic (n ipoteza c nu se #ine seama de multiplic rile cu 1, i, $i de ordinea factorilor). Pentru z=a+bi!3[i] definim norma lui z prin N(z)=a2+b2. Evident, dac N(z)=p cu p prim, p A1(4), atunci a6b (c ci n caz contrar p=2a 2A0(2) ).

161

Fie acum n! n1 = p r iar n 2 =


p prim p 1 (4 )

q prim q 3 (4 )

pe care l scriem sub forma n=2kn1n2 cu k! , q s . Atunci descompunerea lui n n factori primi n

3[i] va fi : n = [(1 + i )(1 - i )]k

a +b = p p prim p 1 (4 )

[(a + bi )(a - bi )]r q s (unde r $i s variaz 2


q prim q 3 (4 )

o dat cu p $i q). )innd cont de unicitatea descompunerii lui n de mai nainte deducem c fiec rei reprezent ri a lui n sub forma n=u2+v2=(u+iv)(u-iv) (cu u, v!3) i corespund pentru u+iv $i u-iv descompuneri de forma : ( C) u + iv = i t (1 + i )k1 (1 - i )k2 (a + bi )r1 (a - bi )r2 q s1

( CC) u - iv = i -t (1 + i )k2 (1 - i )k1 (a + bi )r2 (a - bi )r1 q s2 cu t!{0, 1, 2, 3}, k 1+ k2=k, r1+r2=r $i s1+s2=s. Observ m c factorii primi asocia#i lui u+iv determin n mod unic factorii primi ai lui u-iv ($i reciproc). De asemenea, fiecare pereche de numere complex conjugate (u+iv, u-iv) cu u, v!3 dat de rela#iile (C) $i (CC) de mai sus verific egalitatea n=u2+v2. Observ m de asemenea c schimbarea i--i nu afecteaz factorii reali q astfel c s1=s2 iar s=2s1 (#innd cont de Teorema 1.6.). Pentru alegerea lui t avem 4 posibilit #i (c ci t!{0, 1, 2, 3}). Pentru k 1 avem k+1 posibilit #i de alegere (c ci k1!{0, 1, ,k}) iar pentru k1 ales, k2 se determin din k2=k-k1. Analog, pentru r1 avem r+1 posibilit #i de alegere (c ci r1!{0, 1, ..,r}) iar r2=r-r1. Astfel, avem un num r total de 4(k+1) (r + 1) posibilit #i de a asocia lui u+iv factorii primi Gauss din descompunerea lui n n factori primi (n 3[i]) (unde produsul (r + 1) se face dup to#i primii p<1(4) a.. prDn). S vedem cte dintre aceste asocieri sunt diferite.

(1 + i ) (1 - i )
k1

)innd cont de egalitatea 1+i=i(1-i), dac


k2

avem un factor

atunci acesta devine

162

i k1 (1 - i )k1 (1 - i )k2 = i k1 (1 - i )k1 + k2 = i k1 (1 - i )k fapt 4


p prim pr n

astfel c num rul c utat este de

(1 + r ) = d (n1 ) (c ci n1 = p ).
r p prim p 1 (4 )

Din cele de mai nainte deducem c num rul total de solu#ii ntregi ale ecua#iei x2+y2=n este 4d(n1). S ar t m acum c d(n1)=d1(n)-d3(n). Pentru aceasta s observ m c num rul divizorilor impari ai lui n este egal cu num rul termenilor sumei
0 mi ri 0 k j s j

p1 1 p 2

... p n

q1 1 ... q t t = ( CCC)

p n p prim p 1 (4 )

(1 + p + .. + p r ) s (1 + q + ... + q s )
q n q prim q 3 (4 )

k j este par, n caz contrar avnd d <3(4).


j =1

Dac dDn, atunci este clar c avem d<1(4) dac $i numai dac n (CCC)

Dac nlocuim pe q cu 1 atunci produsul


s

(1 + q + ... + q s ) este zero

q n q prim q 3 (4 )

chiar dac un singur exponent s este impar ; dac to#i ace$ti exponen#i s sunt pari atunci 1 + q + ... + q s = 1 $i astfel membrul drept din (CCC) devine

p n p 1 (4 )

(1 + p + ... + p r ) astfel c termenii dezvolt rii acestui produs sunt exact to#i

qs n q prim q 3 (4 )

divizorii lui n1. Pentru a ob#ine d(n1) fiecare termen trebuie s fie num rat ca 1. Acest lucru este u$or de realizat dac n (CCC) nlocuim n partea dreapt $i pe p cu 1, ob#innd (1 + r ) . Dac privim acum membrul stng al egalit #ii
pr n p prim p 1 (4 )

(CCC) dup ce n partea dreapt am nlocuit fiecare p cu 1 $i fiecare q cu 1 163

este clar c fiecare dDn, d<1(4) este num rat ca +1 $i fiecare dDn, d<3(4) este num rat ca 1. Astfel membrul stng din (CCC) devine d1(n)-d3(n) iar membrul drept d(n1), de unde egalitatea d(n1)=d1(n)-d3(n). Sumnd cele expuse pn aici ob#inem urm torul rezultat ce include $i Teorema 1.6. (Fermat Euler) : TEOREMA 1.7. Fie n! * iar n=2kn1n2 (cu k! , n1 = p r !i
p n p prim p 1 (4 )

n2 =

q s ) descompunerea lui n n factori primi.


q n q prim q 3 (4 )

Atunci ecua ia x2+y2=n are solu ie n 3 dac$ !i numai dac$ to i exponen ii s din descompunerea lui n2 sunt pari. Num$rul solu iilor din 33 ale ecua iei x2+y2=n este egal cu 4(d1(n)-d3(n)) unde da(n) este num$rul divizorilor d ai lui n cu proprietatea c$ d2a(4), a=1, 3. Exemple 1. Dac n=1, atunci d1(1)=1 $i d2(1)=0, astfel c n 33 ecua#ia x2+y2=1 va avea 4(1-0)=4 solu #ii. 2. Dac
2 2

n=2, atunci d1(2)=1 $i d2(2)=0, astfel c n=5, atunci d1(5)=2 $i d2(5)=0, astfel c

n 33 ecua#ia n 33 ecua#ia

x +y =2 va avea 4(1-0)=4 solu #ii. 3. Dac


2 2

x +y =5 va avea 4(2-0)=8 solu#ii. (Se confirm astfel cele stabilite la exemplele 1)-3) de la nceputul paragrafului 1 ). 4. Am v zut mai nainte (Teorema 1.3.) c dac p este un num r prim de forma 4k+1, atunci exist x, y! * a.. p=x2+y2.( cum d1(p)=2 iar d2(p)=0, conform teoremei 1.7. ecua#ia x2+y2=p va avea n 33 4(2-0)=8 solu#ii. Se reconfirm concluzia de la observa #ia de la nceputul paragrafului 1, cazul iv)). n continuare vom prezenta o metod de g sire a numerelor x, y atunci cnd se d p (metod dat de Lagrange n anul 1808, dup ce, tot el demonstrase n 1785 c lungimea perioadei pentru func#ia continu a lui pentru numerele prime p de forma 4k+1). p este impar

164

Pentru aceasta s ne reamintim c la capitolul de frac#ii continue a fost prezentat urm torul algoritm de dezvoltare n frac#ie continu a unui ira#ional p tratic += D : Punem a 0 =

[ D ], b =0, c =1 $i apoi construim prin recuren#


0 0

2 a + bn +1 D - bn +1 c = . a n +1 = 0 , b = a c b $ i n +1 n +1 n n n c c n n +1 Calculul se continu pn cnd bn+1=bn sau cn+1=cn .

D = a 0 ; a1 ,..., a n -1 , a n , a n -1 ,..., a1 ,2a 0 i) Dac bn+1=bn , atunci lungimea perioadei minime este par ). ii) Dac cn+1=cn , atunci D = a 0 ; a1 ,..., a n , a n ,..., a1 ,2a 0 perioadei minime este impar ).

(adic lungimea

(adic

bn + D . cn S trecem acum la rezolvarea ecua #iei x2+y2=p, cu p un num r prim de Numerele bn $i cn de mai sus sunt cele din scrierea lui a n = forma 4k+1 ( de exemplu n ). Dup cum am amintit mai sus, lungimea perioadei minime pentru frac#ia continu a lui Deci p = a 0 ; a1 ,..., a n , a n ,..., a1 ,2a 0 .

perioada simetric , deci ~ = -1 #innd cont de Propozi#ia 3.14. de la Capitolul 10 - deducem c a n +1 a n +1 (nota#iile sunt cele de la Capitolul 10). Pe de alt parte, ob#inem b n +1 + c n +1 p bn +1 - p 2 2 = -1 b n +1 + c n +1 = p c n +1 a ecua#iei x2+y2=p (evident

Num rul a n +1 = a n ; a n -1 ,..., a1 ,2a 0 , a1 ,..., a n

p este impar .

] ] are

a n +1 =

bn +1 + c n +1

~ = bn +1 - p , a n +1 c n +1

astfel c

$i astfel (bn+1 , cn+1) este singura solu#ie din dac nu #inem cont de ordinea termenilor).

Exemplu S se rezolve ecua#ia x2+y2=1009 n . Evident, num rul p =1009 este prim de forma 4k+1.

165

Avem c1 = 1009 - b12 c0

a0=31,

b0=0,

c0=1

$i

apoi

b1 = a 0 c0 - b0 = 31 ,

31 + 31 =48, a1 = = 1, 48
2 1009 - b2 31 + 17 = 15, a 2 = = 3, c1 15

b2 = a1c1 - b1 = 17, c 2 =

b3 = a 2 c 2 - b2 = 28, c3 =

1009 - 28 2 = 15 = c 2 . 15

Prin urmare suntem n cazul ii) astfel c

1009 = 31;1,3,3,1,6,2 $i

a3 =

28 + 1009 a$a nct 282+152=1009, deci n acest caz solu #ia ecua#iei 15 este (15, 28) (dac nu #inem cont de ordinea termenilor).

x2+y2=1009 din

2 Reprezentarea numerelor naturale ca sum $ de patru p$trate de numere ntregi. Scopul acestui paragraf este acela de a demonstra c orice num r natural poate fi scris ca sum a patru p trate de numere ntregi. )innd cont de identitea lui Euler, potrivit c reia dac x1, x2, x3, x4, y1, y2, y3, y4!3, atunci

(x

2 1

2 2 2 2 2 2 2 y1 + x2 + x3 + x4 + y2 + y3 + y4 = (x1 y1 + x 2 y 2 + x 3 y 3 + x 4 y 4 )2 +

)(

+ (x1 y 2 - x 2 y1 + x 3 y 4 - x 4 y 3 )2 + ( x1 y 3 - x 3 y1 + x 4 y 2 - x 2 y 4 )2 + + (x1 y 4 - x 4 y1 + x 2 y 3 - x 3 y 2 )2 pentru a demonstra c un num r natural se scrie ca sum de patru p trate de numere naturale, este suficient s prob m lucrul acesta pentru numere prime. TEOREMA 2.1. (Lagrange) Fie p este un num$r prim; atunci: 2 2 2 2 (1) Exist$ m !i x1, x2, x3, x4! a.. m p = x1 + x2 + x3 + x4 (1)m<p) (2) Dac$ m este cel mai mic num$r natural ce verific $ (1), atunci m=1. Demonstra#ie Pentru a proba (1), s consider m mul#imile : p - 1 X = x 2 x = 0, 1, 2,..., $i 2 166

p - 1 Y = - x 2 - 1 x = 0, 1, 2,..., . 2 S observ m c elementele lui X $i Y nu sunt congruente dou cte dou modulo p (separat). p - 1 2 2 ntr-adev r, dac exist x1 , x 2 0, 1,..., a.. x1 x 2 (mod p ) 2 cu x1>x2 $p|(x1-x2)(x1+x2) ceea ce este imposibil deoarece 1 * x1+x2 *p-1. Analog se arat c elementele lui Y nu sunt congruente dou cte dou modulo p. Dac not m prin | X | num rul de elemente ale lui X modulo p, atunci p +1 p +1 cum |X|+|Y|= + = p +1 > p , deducem c exist 2 2 p - 1 2 2 a.. x, y! 1, 2,..., a.. x <-y -1 (mod p), altfel zis exist m! 2 mp=x2+y2+1. Clar 2 p -1 p -1 1 p -1 1 1 2 1 p -1 x + y 2 + 1 2 + < + < p. 1 m = + 1 = p p p p p 2 2 2 Pentru a proba (2) s observ m c dac m este par, atunci sau toate xiurile sunt impare sau dou . Dac toate xi-urile sunt impare, atunci egalitatea de la (1) se mai scrie

m x + x 2 x1 - x 2 x3 + x 4 x3 - x 4 sub forma: 1 = p iar cum + + + 2 2 2 2 2 x1x2 $i x3x4 sunt numere pare se contrazice minimalitatea lui m. Dac numai x1 $i x2 sunt pare iar x3 $i x4 sunt impare, din nou se contrazice minimalitatea lui m (c ci din nou x1x2 $i x3x4 sunt numere pare). Analog dac xi-urile sunt pare. Deci m trebuie s fie impar. Dac m=1 nu avem ce demonstra. S presupunem deci c 3 * m<p. Alegem y1, y2, y3, y4 a.. xi<yi (m), n mod evident m -1 m -1 yi , i=1, 2, 3, 4 $i 2 2 2 2 2 2 2 2 2 2 y1 + y2 + y3 + y4 0 (m ) , deci mn = y1 + y2 + y3 + y4 4 m -1 <m. m 2 167
2

pentru un anumit n. Mai mult, 0 n

Evident, n ' 0 (c ci n caz contrar ar rezulta yj= 0 pentru orice j=1, 2, 3, 4, ceea ce ar implica x j<0(m), j=1, 2, 3, 4, $i deci 2 2 2 2 mp = x1 + x2 + x3 + x4 = 0 m 2 , de unde p<0(m), ceea ce este imposibil

( )

deoarece 3*m<p). Deci n.1 $i deducem imediat c 2 2 2 2 2 2 2 2 2 2 2 2 2 m np = x1 + x2 + x3 + x4 y1 + y2 + y3 + y4 = z1 + z2 + z3 + z4 , unde

)(

z1 = x1 y1 + x 2 y 2 + x3 y3 + x 4 y 4 , z 2 = x1 y 2 - x 2 y1 + x3 y 4 - x 4 y3 z 3 = x1 y3 - x3 y1 + x 4 y 2 - x 2 y 4 , z 4 = x1 y 4 - x 4 y1 + x 2 y3 - x3 y 2 m -1 m -1 yi Cum xi<yi (m), , i=1, 2, 3, 4, deducem c m|zj , 2 2 j=2, 3, 4 $i din egalitatea de mai sus rezult c m|z1. z z z z Avem deci c np = 1 + 2 + 3 + 4 , ceea ce din nou m m m m contrazice minimalitatea lui m.(c ci n<m). n concluzie m=1 $i totul este acum clar. & 3. Alte teoreme de reprezentare a numerelor ntregi TEOREMA 3.1. (Erds-Suranyi) Orice num$r k!3 se poate scrie ntr-o infinitate de moduri sub forma k=1222m2 cu m! . Demonstra#ie Facem induc#ie matematic observnd c este suficient s presupunem k! . Observ m c 0=1 2+22- 32+42- 52- 62+72 1=12 2= -1 2- 22- 32+42 3= -1 2+22 4= -1 2 -22+32
2 2 2 2

S presupunem acum c pentru un k! avem k= 12 22 m2. Cum (m+1)2 - (m+2)2 - (m+3)2 +(m+4)2 = 4, avem 2 k+4= 1 22 m2 + (m+1)2 - (m+2)2 - (m+3)2 + (m+4)2 $i astfel teorema este demonstrat . Infinitatea descompunerii rezult din identitatea

168

(m+1)2 - (m+2)2 - (m+3)2 + (m+4)2 - (m+5)2 + (m+6)2 + (m+7)2 - (m+8)2 = 0 $i astfel n descompunerea lui k nlocuim pe m cu m+8 $.a.m.d. & n leg tur cu alte tipuri de reprezent ri ale numerelor ntregi recomand m cititorului lucrarea lui Emil Grosswald : ,,Representations of Integers as Sums of Squares, Springer-Verlag, 1985. Printre alte rezultate, n cartea respectiv se prezint $i urm toarele: TEOREMA 3.2. Un num$r natural n se poate scrie sub forma n=x +y +z2, cu x, y, z!3 dac$ !i numai dac$ n nu este de forma 4 k (8m+7)
2 2

cu k, m! . TEOREMA 3.3. Num$rul solu iilor ntregi (x, y, z) ale ecua iei 16 x2+y2+z2=n este dat de n L(1, c ) q (n ) P(n ) , unde n=4an1 , (4?n1),

0 daca n1 7 (8 ) q (n ) = 2 - a daca n1 3 (8 ) - a -1 daca n1 1, 2, 5 sau 6 (8) 3 2


-1 n b -1 p 2b 1 P(n ) = 1 + p - j + p -b 1 - p p j =1 p prim3 2b p n

(P(n)=1 dac$ n nu con ine p$trate), iar

L(s, c ) =

m =1

c (m ) m - s , cu

c (m ) =

- 4n (simbolul lui Jacobi !). m Demonstra#iile acestor teoreme fiind destul de laborioase am renun#at la

prezentarea lor n detaliu limitndu-ne doar la a le semnala. (cititorul interesat le poate g si n cartea citat mai sus). TEOREMA 3.4. (H.E.Richert) Orice num$r natural n>6 se poate scrie ca sum$ de diferite numere prime. 169

Demonstra#ie Pentru a demonstra teorema lui Richert avem nevoie de dou rezultate preliminare: Lema 1. Fie m1, m2, un !ir infinit cresc$tor de numere naturale a.. pentru un k! , (1) mi+1.2mi pentru orice i>k. Presupunem c$ exist$ a! !i r, sr-1! a.. sr-1/mk+r a.. fiecare dintre numerele : (2) a+1, a+2, , a+s r-1 este suma diferitelor numere din !irul m1, m2, , mk+r-1. Atunci fiecare dintre numerele: (3) a+1, a+2, , a+s r este suma diferitelor numere din !irul m1, m2, , mk+r !i mai mult, sr/mk+r+1. ntr-adev r, fie n un num r din $irul (3). Dac n.a+sr-1 nu mai avem ce demonstra deoarece conform ipotezei n este sum de diferi#i termeni ai $irului m1, m2, , mk+r-1. S presupunem c n>a+sr-1.Cum sr-1/mk+r, avem n/a+1+mk+r , deci n-mk+r/a+1, adic num rul n-mk+r este un termen al $irului (2) $i n consecin# se va scrie ca sum de termeni din $irul m1, m2, , mk+r-1. Rezult c $i n este atunci sum de diferi#i termeni din $irul m1, m2, , mk+r. Mai mult, #innd cont de (1) deducem c mr+k+1.2mk+r $i astfel sr=sr-1+mk+r/2mk+1/mk+r+1. Astfel Lema 1 este probat . Lema 2. Fie m1, m2, un !ir infinit de numere naturale a.. (1) are loc pentru un num$r natural k, !i exist$ s0, a! a.. s0/mk+1 a.. fiecare dintre numerele (4) a+1, a+2, , a+s 0 este sum$ de diferi i termeni din !irul m1, m2, , mk. Atunci orice num$r natural >a se scrie ca sum$ de termeni ai !irului m1, m2, ntr-adev r, conform Lemei 1 (cu r=1, 2, , t, t ! ) fiecare dintre numerele (5) a+1, a+2, , a+s t se scrie ca sum de termeni din $irul m1, m2, , mk+t. Cum ns sr>sr-1, r=1, 2, t, observ m c pentru orice num r natural n exist un num r natural t a.. n.a+st. n consecin# orice num r natural n>a este unul dintre termenii $irului (5) cu t convenabil ales $i astfel va fi sum de diferi#i termeni din $irul m1, m2, Cu aceasta Lema 2 este $i ea probat . S revenim acum la demonstra#ia teoremei. Fie mi=pi cu i=1, 2, (pi-fiind al i-ulea num r prim). Conform Corolarului 3.21. de la Capitolul 7, numerele m i verific condi#iile Lemei 2 (cu a=6, s0=13, k=5). Aceasta deoarece 13=p 6 $i fiecare dintre numerele 7, 8, , 19 170

se scriu ca sum de diferite numere prime, .p5=11 dup cum urmeaz : 7=2+5, 8=3+5, 9=2+7, 10=3+7, 11=11, 12=5+7, 13=2+11, 14=3+11, 15=3+5+7, 16=5+11, 17=2+3+5+7, 18=7+11, 19=3+5+11. Teorema rezult acum ca o consecin# imediat a Lemei 2. & COROLARUL 3. 5. Orice num$r natural n/10 se poate scrie ca sum$ de diferite numere prime impare. Demonstra#ie ntr-adev r, dac alegem mi=pi+1 atunci condi#iile Lemei 2 de la demonstra#ia Teoremei 3.4. sunt satisf cute (cu a=9, s0=19, k=6), deoarece 19=p8=m7 , deci s0=m6+1 $i mai mult, fiecare dintre numerele 10, 11, , 28 se scriu ca sum de diferite numere prime impare, .m6=19 dup cum urmeaz : 10=3+7, 11=11, 12=5+7, 13=13, 14=3+11, 15=3+5+7, 16=5+11, 17=17, 18=5+13, 19=3+5+11, 20=7+13, 21=3+5+13, 22=5+17, 23=3+7+13, 24=11+13, 25=5+7+13, 26=3+5+7+11, 28=3+5+7+13. Observa#ie: n lucrarea A. Macowski, Partitions into unequal primes din Bull. Acad. Sci. Sr. Sci. Math. Astr. Phys., 8(1960), pp. 125-126 se demonstreaz urm toarele rezultate : TEOREMA 3.6. Orice num$r natural n>55 se poate scrie ca sum$ de diferite numere prime de forma 4k-1. TEOREMA 3.7. Orice num$r natural n>121 se poate scrie ca sum$ de numere prime de forma 4k+1. TEOREMA 3.8. Orice num$r natural n>161 se poate scrie ca sum$ de numere prime de forma 6k-1. TEOREMA 3.9. Orice num$r natural n>205 se poate scrie ca sum$ de numere prime de forma 6k+1. S mai amintim $i un rezultat al lui L. Schnirelman: TEOREMA 3.10. (Schnirelman) Exist$ un num$r natural s a.. orice num$r natural mai mare sau egal cu 2 se scrie ca suma a cel mult s numere prime (nu neap$rat distincte). Cititorul poate g si demonstra#ia acestei teoreme n lucrarea [21, pp.107] (preluat dup articolul original al lui Schnirelman: Uber additive Eigeenschaften von Zahlen din Math. Ann. 107, 1933, pp. 649-690). n lucrarea lui Vinogradov: Representation of an odd number as a sum of three primes din Comptes Rendus (Doklady) de lAcademie de Sciences de l URSS , nr 15, 1937, pp. 191-294, se demonstreaz (din p cate neelementar ):

171

TEOREMA3.11. (Vinogradov) Orice num r natural impar suficient de mare se scrie ca sum a cel mult trei numere prime.

Din Teoremele lui Schnirelman $i Vinogradov deducem imediat: COROLARUL 3.12. Exist $ n0 ! , n0/2, a.. orice num$r natural n, n/n0 se scrie ca suma a cel mult patru numere prime. Observa ii 1. Shapiro !i Warga n lucrarea: On representation of large integers as sums of primes din Comm. Pure Appl. Math, 3, 1950, p. 153 demonstreaz$ elementar un rezultat mai slab: orice num r natural suficient de mare se scrie ca suma a cel mult 20 de numere prime. 2. Rafinnd procedeul lui Schnirelman, Yin Wen-Lin, n lucrarea Note on the representation of large integers as sums of primes din Bull. Acad. Polon. Sci. cl III, 4, 1956, pp. 793-795 demonstreaz elementar c orice num$r natural suficient de mare se scrie ca suma a cel mult 18 de numere prime. 3. S reamintim aici $i o conjectur a lui Goldbach: orice num$r natural par mai mare sau egal cu 4 se scrie ca suma a dou $ numere prime. Dac aceast conjectur ar fi adev rat (lucru neprobat pn acum) atunci ar rezulta c orice num$r natural mai mare sau egal cu 2 se scrie ca suma a cel mult 3 numere prime.

172

CAPITOLUL 12: ECUA'II DIO PHANTICE n cele ce urmeaz prin ecua#ie diophantic $ n#elegem o ecua#ie de forma f(x1,,xn)=0 cu f3[X1,,Xn]. A rezolva o astfel de ecua#ie diophantic revine la a g si toate n-uplurile (a1,,an)3n pentru care f(a1,,an)=0. Observa#ie Denumirea de ecua#ii diophantice provine de la numele matematicianului grec Diophante (aprox. secolul III era noastr ). 1.Ecua ia ax+by+c=0, a, b, c 3 (1) LEMA 1.1. Ecua ia (1) are solu ie n 3 dac$ !i numai dac$ d=(a, b) | c. Demonstra#ie n mod evident, dac x, y3 a.. ax+by+c=0, atunci cum c = -ax-by deducem c d | c c=dt cu t!3. Reciproc, s presupunem c d|c. Atunci din algoritmul lui Euclid deducem c exist x1, y13 a.. d=ax1+by1. Atunci c = dt =(ax1 + by1)t = a(x1t) + b(y1t) a(x1t) + b(y1t)-c = 0 a(-x1t)+b(-y1t)+c=0, adic (-x1t, -y1t) este solu#ie a ecua#iei ax + by + c = 0. 3 LEMA 1.2. Dac$ (a, b) =1 iar (x0, y0) este solu ie particular$ a ecua iei (1), atunci solu ia general$ din 3 a acestei ecua ii este dat$ de x=x0-kb !i y=y0+ka , cu k3. Demonstra#ie Dac particular a lui (1) =ax0+by0+c-abk+abk=0. $i x=x0-kb $i y=y0+ka (cu (x0, y0)32 solu#ie k3), atunci ax+by+c=a(x0-kb)+b(y0+ka)+c=

Fie acum (x, y)32 a.. ax+by+c=0. Atunci ax0+by0=ax+by a(x0-x)=b(y-y0). Cum (a, b)=1 deducem c a|y-y0, adic y-y0=ka (cu k3) y=y0+ka. Deducem imediat c a(x0-x)=bka, de unde x=x0-kb. & COROLAR 1.3. Fie a, b, c3 a.. d=(a, b)|c, a=da2, b=db2, c=dc2. Dac$ (x0, y0)32 este o solu ie particular$ a ecua iei a2x+b2y+c2=0, atunci solu ia general$ a ecua iei (1) este dat$ de x=x0-kb2, y=y0+ka2 cu k3. 173

Observa#ie )innd cont de Lema 1.2. $i Corolarul 1.3. deducem c atunci cnd suntem pu$i n situa#ia de a rezolva o ecua#ie diophantic de forma (1) (n cazul n care d=(a, b)6c ) este recomandabil s mp r#im ambii membrii ai ecua#iei prin d, transformnd-o astfel n ecua#ia echivalent a2x+b2y+c2=0 (cu a2=a/d, b2=b/d, c2=c/d ). Cum (a2, b2)=1, forma general a solu#iilor ecua#iei a2x+b2y+c2=0 este dat de Lema 1.2. S prezent m acum un procedeu de a g si o solu#ie particular (x0, y0) a ecua#iei (1) (cu a, b, c 3, (a, b)=1). Pentru aceasta vom dezvolta num rul ra#ional a=a/b n frac#ie continu . P strnd nota#iile de la Capitolul 10 (de Frac #ii continue), observ m c ultima redus pn/qn a lui a este chiar pn/qn=a/b=a. )innd cont de Propozi #ia 3.3. de la Capitolul 10 putem scrie: p n p n -1 (-1) n -1 a p (-1) n -1 = - n-1 = aq n -1 - bp n -1 + (-1) n -1 = 0 q n q n-1 q n q n -1 b q n -1 q n q n -1 de unde (prin nmul#ire a ambilor membrii ai ultimei egalit #i cu (-1)n c ) a[(-1)n c qn-1] + b[(-1)n+1 c pn-1] + c = 0. Deducem c x0=(-1)n c qn-1 $i y0=(-1)n+1 c pn-1 este o solu#ie particular a ecua#iei (1). Conform Lemei 1.2. solu#ia general a ecua#iei (1) va fi atunci x=(-1)n c qn-1-bk $i y=(-1)n+1 c pn-1 +ak cu k3. Exemplu S se rezolve ecua#ia diofantic (*) 317 x + 182 y + 94 = 0 Avem a=317, b=182, c=94 $i se observ c (a, b)=1, astfel c ecua#ia (*) are solu#ie n 32 (conform Lemei 1.2.). Pentru a g si solu#ia general
2

a ecua#iei (*) s

g sim o solu#ie

particular (x0, y0)3 a ecua#iei (*). Prin calcul direct g sim urm toarea dezvoltare n frac #ie continu a lui 317 317 : =[1;1, 2, 1, 6, 1, 5]. a= 182 182 317 Redusele a = se ob#in completnd de la stnga la dreapta tabelul: 182

174

a p q

1 (a 0) 1 1 0 1

1 (a1) 2 1

2 (a2) 5 3

1 (a3) 7 4

6 (a4) 47 27

1 (a5) 54 31

5 (a6) 317 182

Deducem c a =
n+1

p6 317 = , adic n=6. q6 182

O solu#ie particular va fi x0 = (-1)n c qn-1 = (-1)6 94q5 = 9431 = 2914, y0 = (-1) c pn-1 = (-1)794p5 = -9454 = -5076 Astfel, solu#ia general a ecua#iei (*) va fi x=2914-182k, y=-5076+317k, cu k3. 2.Ecua ia x2 + y2 = z2 (2) n primul rnd trebuie observat c $ dac$ tripletul (x, y, z) de numere ntregi verific$ ecua ia (2), atunci aceea !i ecua ie va fi satisf$cut$ de orice triplet de forma ( lx, ly, lz), unde l3 !i reciproc. De aceea, pentru a g si toate solu#iile ecua#iei (2) (constnd din numere diferite de zero) este suficient s g sim (solu#iile (x, y, z) pentru care numerele x, y, z sunt relativ prime (adic nu au nici un divizor prim diferit de 1)). Este clar c dac ntr-o solu#ie (x, y, z) a ecua#iei (2) dou dintre numerele x, y, z au un divizor comun l = 1, atunci $i cel de al treilea num r se divide cu l. De aceea ne putem restrnge la solu#iile ce constau din numere relativ prime dou cte dou , pe care le vom numi solu#ii primitive. Dac (x, y, z) este o solu#ie a lui (2), atunci n mod evident $i (y, x, z) este solu#ie. Pe de alt parte, dac (x, y, z) este solu#ie, atunci x sau y este par (c ci dac x $i y ar fi impare atunci x2+y2 ar fi de forma 4k+2, pe cnd p tratul unui num r ntreg nu poate fi dect de forma 4k sau 4k+1). n plus, este evident c dac (x, y, z) este solu#ie, atunci $i (x, y, z) vor fi solu#ii.

175

LEMA 2.1. Orice solu ie particular$ (x, y, z) de numere naturale (cu n par) a ecua iei (2) este de forma x=2mn, y=m2-n2, z=m2+n2 cu m, n !i n<m, (n, m)=1 iar m, n au parit $ i diferite . Demonstra#ie Identitatea (2mn)2+(m2-n2)2=(m2+n2)2 arat c numerele de forma din enun# sunt solu#ii ale ecua#iei (2) cu x par. Dac x, y, z au un divizor comun l 2, atunci l divide $i numerele 2m2=(m2+n2)2+(m2-n2)2 $i 2n2=(m2+n2)2- (m2-n2)2 . Rezult c l=2 (c ci (m, n)=1). ns atunci m2 $i n2 sunt simultan pare sau impare, ceea ce este imposibil c ci prin ipotez m $i n au parit ti diferite. Deci solu#ia din enun# este primitiv . Reciproc, fie (x, y, z) o solu #ie primitiv a lui (2) cu x, y, z iar x=2a. Atunci y $i z sunt impare, deci numerele z+y $i z-y sunt pare (fie z+y=2b, z-y=2c). Orice divizor comun al lui b $i c divide pe z=b+c $i pe y=b-c, de aceea l=1, astfel c (b, c)=1. Pe de alt parte 4a2 = x2 = z2 - y2 = 4bc, de unde a2 = bc, adic b = m2 $i c= n2 (m, n ) iar de aici a2=m2n2a=mn, deci x=2a=2mn, y=b-c=m2-n2 iar z=b+c=m2+n2 (se observ c n<m). & COROLAR 2.2. Solu ia general$ a ecua iei (2) este x= 2rmn, y=r (m -n2), z=r (m2+n2) cu r, m, n3.
2

3. Ecua ia x4+y4=z4 (3). n cadrul acestui paragraf vom demonstra un rezultat ceva mai general %i anume: LEMA 3.1. Ecua ia x4+y4=z2 ( 4 ) nu are solu ii n 3*. Demonstra#ie S presupunem c ar exista o solu#ie n 3* a ecua#iei (4). Putem presupune n mod evident c aceast solu#ie const din numere din *. Cum orice mul#ime nevid de numere naturale are un cel mai mic element, atunci printre solu#iile ecua#iei (4) exist una (x, y, z) cu z minim.(vezi Teorema 4.5. de la Capitolul 1). Analog ca n cazul ecua#iei (2) se arat c x sau y trebuie s fie par ; s presupunem c x este par. Cum (x2)2+(y2)2=z2 iar x2, y2 $i z sunt naturale ( $i pot fi presupuse relativ prime ), atunci conform celor stabilite la 2. exist numerele naturale m, n, m>n, relativ prime $i de paritate diferit a.. x2=2mn, y2=m2-n2 $i z2=m2+n2.

176

Dac m=2k $i n=2k+1y2=4(k2-l2-l-1)+3, ceea ce nu se poate (c ci y2 trebuie s fie de forma 4k sau 4k+1). Rezult c m este impar iar n este par. Fie n=2q; atunci x2=4mn a$a c mq=(x/2)2. Cum (m, n)=1 deducem c m=z12,q=t2 cu (z1,t)=1 $i naturale. n particular, observ m c y2 = (z12)2-(2t2)2 ( 2t2)2 + y2 = (z12)2. Aplicnd din nou cele stabilite la 2 deducem c exist a, b *, a>b, (a,b)=1 $i de parit #i diferite a.. 2t2 =2ab t2 =ab y2 =a2-b2 z12 =a2+b2. Cum (a, b)=1 iar t 2=ab deducem c a=x12, b=y12 $i atunci x14+y14=z12. Deducem c (x1, y1, z1) este o solu#ie a lui (4) $i conform alegerii lui z ar trebui ca z1 z z12 z m m2 + n2, ceea ce este absurd. & COROLARUL 3.2. Ecua ia (3) nu poate avea solu ii (x, y, z) cu x, y, Observa#ie Ecua#ia (3) este legat de ceea ce n teoria numerelor a fost cunoscut sub numele de Marea teorem a lui Fermat (de$i corect ar fi fost s fie numit ,,Marea Conjectur a lui Fermat! ): (n sensul c$ xyz 0). (Evident, este suficient s presupunem c n este prim). Pentru n=4 am v zut mai sus c ecua#ia lui Fermat x4+y4=z4 nu are Dac$ n 3 atunci ecuatia xn+yn=zn nu poate avea solu ie nenul$ n 3 z3*.

solu#ii n 3* (Corolarul 3.2.). Printre hrtiile lui Fermat a fost g sit demonstra#ia teoremei numai pentru cazul n=4 (interesant este c aceasta este singura demonstra#ie a unui rezultat de teoria numerelor care s-a p strat de la Fermat ! ). n ce prive$te cazul general, n>4, Fermat a notat ( pe marginea unei pagini din ,,Aritmetica lui Diophant ) c a g sit ,,o demonstra#ie cu adev rat minunat a acestui fapt, dar ,,aceast margine este prea ngust pentru a o cuprinde. Cu toate eforturile multor matematicieni, aceast demonstra #ie nu a fost g sit $i este ndoielnic c ea ar fi existat n general!. Mai mult, numai pentru n=4 s-a reu$it s se dea o solu#ie elementar . Astfel se explic de ce speciali$tii n teoria numerelor au fost convin$i de imposibilitatea demonstr rii Marii teoreme a lui Fermat prin procedee elementare.

177

Paradoxul const totu$i n aceea c n toate cazurile n care Fermat a afirmat categoric c a demonstrat o afirma#ie sau alta, ulterior s-a reu$it a se demonstra aceast afirma#ie. Cel care a reu$it s demonstreze conjectura lui Fermat este matematicianul englez Andrew Wiles de la Universitatea din Princeton (S.U.A). De fapt acesta a demonstrat o alt conjectur (a$a zisa conjectur a lui TaniyamaWeil) din care conjectura lui Fermat rezult ca un corolar. Din p cate demonstra#ia lui Wiles este destul de dificil , ea neavnd un caracter elementar, limitnd astfel accesul la n#elegerea ei pentru un foarte mare num r de matematicieni. Celor care posed cuno$tin#e solide de aritmetica geometriei algebrice le recomand m lucrarea lui A.Wiles din care rezult conjectura lui Fermat (dac x, y, z3, p 3 este num r prim a.. xp+yp=zp, atunci xyz=0):

A.Wiles: Modular Elliptic Curves and Fermats Last Theorem, Annals of Math, vol. 141, pp. 443-551, 1995. 4 Ecua ii de tip Pell: x2-Dy2=1 (D ) (5) Ca $i n paragrafele precedente, pentru a rezolva ecua #ia (5) n 3 este suficient s g sim solu#iile sale x, y Dac D=n cu n
2 * *

, atunci (x-ny )(x+ny)=1 $i se arat imediat c


*

aceast ecua#ie nu are solu#ii x, y

R mne deci s ne ocup m doar de cazul D *$i D I. n Capitolul 10 (Propozi#ia 3.15) am v zut c frac#ia continu a lui D este de forma : D = [a 0 ; a1 ,..., a n , 2a 0 ] , D=

D = [a 0 ; a1 ,..., a n , a 0 + D ] , de unde

( q (a
n

p n a 0 + D + p n -1
0

) D )+ q

adic iar de aici,

n -1

Dq n + D (q n a 0 + q n -1 ) = ( p n a 0 + p n -1 ) + p n D . Cum = (-1)n+1 , adic D I, deducem c Dqn=pna0 + pn-1 $i pn=qna0 + qn-1.


2 2 pn - Dq n = (-1) n +1 . 2 2 Atunci p n - Dq n = (qna0 + qn-1)pn- (pna0 + pn-1)qn = -(qnpn-1 - pnqn-1) =

178

Aceast ultim egalitate ne sugereaz : LEMA 4.1. Toate solu iile ecua iei (5) sunt date de reduse ale lui D.
2 2 Demonstra#ie Egalitatea p n - Dq n = (-1) n +1 r mne adev rat $i dac n locul lui n punem k(n+1)-1 (deoarece nu este nevoie s consider m cea mai scurt perioad ) : 2 2 k ( n +1) ( C) p k , ceea ce ne arat c o infinitate ( n +1) -1 - Dq k ( n +1) -1 = ( -1)

de reduse ale lui redus a lui D.

D ne dau solu#ii pentru ecua#ia x2-Dy2=1.


*

Fie acum p, q

a.. |p2-Dq2|=1. Vrem s demonstr m c p/q este o

S presupunem prin absurd c p/q nu este o redus a lui D . Atunci conform observa#iei de la Propozi#ia 3.7. de la Capitolul 10, exist o redus pk /qk a lui Avem 0< | absurd. ale lui D cu : |qk D -pk|<|q D -p| $i qk<q. |qk D +pk|2qk D +|pk-Dqk|2(q-1) D +|q D -p|=2q D 2 2 p k - D q k |=|qk D -pk||qk D +pk|<|q2D-p2|=1 , ceea ce este

-(2 D -|q D -p|)<2q D -|q D -p||q D +p|, de unde rezult c :

Rezult deci c toate solu#iile ecua#iei x2-Dy2=1 sunt date de reduse D.


2 2 Fie acum p k - Dq k = 1 o astfel de solu#ie.

Avem D =[a0;a1,, ak, ak+1]. /tim c ak+1 este un ira#ional p tratic redus care satisface ecua #ia: 2 2 Ak+1x2 + Bk+1x + Ck+1=0, unde Ak+1= p k - Dq k = 1 . n plus, Rezult

Bk2+1 -4Ak+1Ck+1=4D $i Bk+1 este par.


ak+1= B k +1 + D $i cum ak+1 este ira#ional p tratic redus 2 D , deci toate

avem ak+1=a0+ D $i deci [2a 0 ; a1 ,..., a k ] este o perioad a lui

solu#iile ecua#iei (5) sunt de forma ( C).& 2 2 Observa#ie Este de re#inut algoritmul de g sire a solu#iei x0 - Dy 0 =1, cu cele mai mici x0 $i y0 naturale nenule: 179

[a0;a1,,an] dac perioada minim are lungimea par

x0 = y0
[a 0;a1,,an,2a0,a1,,an] dac n este par (adic perioada este impar ). S remarc m $i faptul c dac lungimea perioadei lui D este par , atunci ecua#ia x2-Dy2=-1 nu are solu#ii. Exemple : a) Ecua#ia x2-7y2=1. p Avem: 7 = [2;1,1,1,4], 3 =[2;1,1,1]=8/3, deci x0=8 $i y0=3. q3 b) Ecua #ia x2-13y2=-1. p4 Avem: 13 = [3;1,1,1,1,6], =18/5, deci x0=18 $i y0=5. q4 S mai not m faptul c ecua#iile de forma x2-Dy2=m cu D, m3 sunt cunoscute sub numele de ecua#ii de tip Pell (de$i Pell nu s-a ocupat de studiul unor astfel de ecua#ii, aceast gre$eal de denumire datorndu-se lui Euler ).

5. Ecua ii de tipul ax2 + by2 + cz2=0, cu a, b, c3.(6) n cadrul acestui paragraf ne vom ocupa de rezolvarea ecua #iei diophantice (6), unde a, b, c 3 sunt libere de p trate (adic nu con#in n descompunerea lor factori de forma d 2 cu d prim), iar (a, b)=(b, c)=(c, a)=1. n mod evident, dac a, b, c 0 sau a, b, c 0 atunci ecua#ia (6) are solu#ie trivial x=y=z=0. Prin urmare vom presupune c a, b, c nu sunt simultan negative sau pozitive. Dac m, n3, vom scrie m R n dac exist x3 a.. x2m(n), (adic m este rest p tratic modulo n). TEOREMA 5.1. (Legendre ) Fie a, b, c3, libere de p$trate, oricare dou$ relativ prime, neavnd toate acela !i semn. n aceste condi ii ecua ia ax2+by2 =z2 (7) are o solu ie netrivial$ dac$ !i numai dac$ urm$toarele condi ii sunt ndeplinite: (i) -ab R c (ii) ac R b 180

(iii)

bc R a

Este preferabil s demonstr m teorema lui Lagrange sub urm toarea form echivalent : TEOREMA 5.2. Fie a, b numere naturale libere de p $trate. Atunci ecua ia ax2+by2+cz2=0 are o solu ie netrivial$ ntreag$ dac$ !i numai dac$ urm$toarele condi ii sunt ndeplinite : (i) aRb (ii) bRa (iii) (ab/d2) R d , unde d=(a, b) ntr-adev r, s presupunem c Teorema 5.2. este adev rat $i s consider m ecua#ia ax2+by2+cz2=0 cu a, b, c ca n enun #ul Teoremei 5.1. (s presupunem c a, b>0, iar c<0 ). Atunci acx 2-bcy2-z2=0 satisface condi#iile din Teorema 5.2. Dac (x, y, z) este o solu#ie netrivial , atunci, deoarece c este liber de p trate, c/z. Punnd z=cz2 $i simplificnd ajungem la o solu#ie netrivial pentru (5). L s m ca exerci#iu probarea faptului c Teorema 5.1. implic Teorema 5.2.. S trecem acum la demonstrarea Teoremei 5.2.. Dac a=1 totul este clar. S presupunem c a>b (c ci dac b>a schimb m pe x cu y, iar dac a=b atunci 1 este p trat modulo b, $i se verific imediat c exist r, s3 a.. b=r2 +s2; n aceste condi#ii o solu#ie a ecua#iei (6) va fi x = r, y = s, z = r2+s2). S construim acum o nou form Ax2+by2=z2 satisf cnd acelea$i condi#ii ca n enun#ul Teoremei 5.2., 0<A<a $i a.. dac forma astfel construit are o solu#ie netrivial , atunci acea solu#ie verific $i forma din enun#ul Teoremei 5.2.. Astfel dup un num r de pa$i, schimbnd de fiecare dat pe A cu b dac A<b ajungem la unul din cazurile a=1 sau a=b care au fost deja discutate. Iat cum ajungem la aceste cazuri . Conform cu (ii), exist T, c3 a.. (8) c2-b=aT=aAm2, cu A, m3, A liber de p trate, iar |c| a. S ar t m c 0<A<a. ntr-adev r, din (8) deducem c 0c2 =aAm2+b <a(Am2+1), adic A0. Cum b este liber de p trate deducem c A>0. Mai mult, din (8) deducem c aAm2<c2a2/4 astfel c A.Am2 <a/4<a. S ar t m acum c A R b. Fie b=b1d, a=a1d, cu (a1, b1)=1 $i s observ m c (a1, d)=(b1, d)=1 deoarece a $i b sunt libere de p trate. Atunci (8) devine: (9) c2-b1d=a1dAm2 $i cum d este liber de p trate deducem c d/c. Punnd c=c1d $i simplificnd ob#inem : 181

2 (10) d c1 -b1=a1Am2. Atunci Aa1m2-b1 (d) sau Aa12m2-a1b1 (d). ns (m, d)=1 deoarece din (10) deducem c n caz contrar un factor comun al lui m $i d ar divide b1 $i d $i astfel b nu ar mai fi liber de p trate. Utiliznd (iii) $i faptul c m este o unitate modulo d deducem c A R d. Mai mult, c2aAm2(b1) iar deoarece a R b avem c a R b1. De asemenea (a, b1)=1 deoarece n caz contrar un factor comun ar divide d $i b1, contrazicnd faptul c b=b1d este liber de p trate. Similar (m, b1)=1, ceea ce arat c A R b1. Atunci A R db1 sau A R b. Vom scrie acum A=rA1, b=rb2, (A1, b2)=1 $i trebuie s demonstr m c A1b2 R r . Din (8) deducem c : c2-rb2=arA1m2 (11). Cum r este liber de p trate deducem c r | c. Dac c=rc1 atunci aA1m2-b2 ( r ). Cum a R b avem a R r. Scriind acum c aA1b2m2 b22 ( r ) $i observnd c (a, r)=(m, r)=1, concluzion m c A1b2 R r. S presupunem acum c AX2+bY2=Z2 are o solu#ie netrivial . Atunci AX2=Z2-bY2 (12). Din (12) $i (6) prin multiplicare ob#inem : A(Axm) 2=(Z2-bY2)(c2-b)=(Zc+bY)2-b(cY+Z)2. Atunci (6) are solu#ia : x=AXm y=cY+Z z=Zc+bY ceea ce completeaz demonstra#ia

(c ci X0 $i m0 deoarece b este liber de p trate). & COROLAR 5.3. Fie a, b, c3 libere de p$trate, cu (a,b)=(a, c)= =(b, c)=1 !i nu au toate acela !i semn. Dac$ pentru un num$r prim p2 congruen a ax2+by2+cz20(pm) are solu ie (x, y, z)33 , pentru orice m * a.. nici o component$ a sa nu se divide prin p, atunci ax2+by2+cz2=0 are solu ie netrivial$ ntreag$ (x, y, z). Demonstra#ie Fie m=2 $i s presupunem c p|a. Atunci dac (x, y, z) este o solu#ie ca n corolar, s ar t m c p?yz. Dac p|y, atunci p|cz2 care implic (deoarece (a, c)=1) c p|z. Atunci p2|ax2 $i cum p?x ob#inem contradic#ia p2|a. Similar p?z. Atunci by2+cz20(p), de unde deducem c bc R p, ceea ce implic bc R a. Similar ab R c !i ac R b iar acum corolarul rezult $ din teorema lui Legendre (pus$ sub prima form$ ).

182

Observa#ii 1. Acest corolar confirm principiul lui Hasse conform c ruia rezolubilitatea local implic rezolubilitatea global (aici rezolubilitatea local nseamn c ecua#ia considerat are solu#ie netrivial modulo pm pentru orice p prim $i m natural nenul, iar rezolubilitatea global nseamn c ecua#ia are o solu#ie ntreag ). 2. Pentru forme p tratice acest principiu func#ioneaz ns este fals dac ecua#ia are grad mai mare. De exemplu : ecua#ia x4-17y4=2z4 are solu#ie netrivial modulo pm pentru orice p prim $i m $i o solu#ie real , ns nu are solu#ie netrivial ntreag [vezi H. Reichardt: Einige im Kleinen berall lsbare, im Grossen unlsbare diophantische Gleichungen, J. Reine Angew und Math., 184(1942) pp. 12-18]. & 6 Rezolvarea n numere ntregi a sistemelor de ecua ii liniare n cadrul acestui paragraf vom prezenta condi #ii necesare $i suficiente ca un sistem de m ecua#ii liniare cu n necunoscute cu coeficien#i din 3 s aib solu#ie ntreag precum $i modul de aflare a solu#iei generale n caz de compatibilitate. DEFINI'IA 6.1. O matrice U!Mn (3) (n/2) se zice unimodular$ dac$ det (U)=1. n mod evident U este unimodular dac $i numai dac U este inversabil n Mn (3). Grupul unit #ilor monoidului (Mn (3) ,) se noteaz prin GLn(3) $i poart numele de grupul general liniar de ordin n al lui #.. Pentru n/1, i, j! , i6j , 1.i, j.n $i E!3 vom nota prin Tij(E)

maticea din Mn (3) ce are 1 pe diagonala principal , E pe pozi#ia (i, j) $i 0 n rest. Reamintim c pentru m, n! , m, n/2, matricea unitate In este matricea din Mn (3) ce are 1 pe diagonala principal $i 0 n rest, iar matricea nul Om, n este matricea din Mm, n (3) ce are 0 pe toate pozi #iile. De asemenea, pentru 1.i.n vom nota prin Di matricea ce difer de matricea unitate In doar pe pozi#ia (i, i), unde Di are 1. n mod evident det (Tij(E))=1 $i det (Di) = -1, de unde deducem c Ti,j , Di!GLn(3).

183

DEFINI'IA 6.2. Matricele de forma Ti j (E) !i Di cu E!3 , 1)i, j)n definite anterior se numesc elementare. nmul irea la stnga sau la dreapta a unei matrici A cu o matrice elementar $ poart$ numele de transformare elementar$. Din felul n care se nmul#esc dou matrice, urm torul rezultat este imediat: TEOREMA 6.3. Fie m, n ! , m, n/2 !i A!Mm, n (3). 1) Dac$ Ti j (E) este o matrice elementar $ din M m(3) atunci matricea Ti j (E)A se ob ine din A adunnd la elementele liniei i pe cele ale coloanei j nmul ite cu E. 2) Dac$ Tij(E) este o matrice elementar $ de ordinul n atunci matricea ATi j (E) se ob ine din A, adunnd la elementele coloanei j pe cele ale coloanei i nmul ite cu E. 3) Dac$ Di este o matrice elementar $ de ordin m atunci matricea Di A se ob ine din A nmul ind elementele liniei i cu 1. 4) Dac$ Di este o matrice elementar $ de ordin n, atunci matricea ADi se ob ine din A nmul ind elementele coloanei i cu 1.

a11 a12 a13 a14 Exemple Fie m=3 $i n=4 $i A = a 21 a 22 a 23 a 24 . a 31 a32 a33 a34 1 0 0 1) Dac T23(E) = 0 1 l !M3(3), atunci 0 0 1 a11 a12 a13 a14 T23(E) A = a 21 + l a 31 a 22 + l a 32 a 23 + l a33 a 24 + l a34 . a31 a32 a33 a34

184

1 0 2) Dac T12(E) = 0 0

l 0 0

1 0 0 !M4(3), atunci 0 1 0 0 0 1

a 11 a12 + l a11 a13 a14 AT12(E) = a 21 a 22 + l a 21 a 23 a 24 . a31 a32 + l a31 a33 a34 1 0 0 3) Dac D2= 0 - 1 0 !M3(3), atunci 0 0 1 a11 a12 a13 a14 D2A= - a 21 - a 22 - a 23 - a 24 . a 31 a 32 a 33 a 34 1 0 4) Dac D4= 0 0 0 1 0 0 0 0 0 0 !M4(3), atunci 1 0 0 - 1

a11 a12 a13 - a14 AD4= a 21 a 22 a 23 - a 24 . a31 a32 a33 - a 34


DEFINI'IA 6.4. Fie n! , n/2 !i 1.i, j.n. Matricea Pi j!Mn(3) ce se ob ine din In punnd pe pozi iile (i, i) !i (j, j) n loc de 1 pe 0 !i care n plus pe pozi iile (i, j) !i (j, i) are 1 poart $ numele de matrice de transpozi ie. 185

1 0 Exemplu Dac n=4, atunci P23= 0 0

0 0 0 0 1 0 . 1 0 0 0 0 1

OBSERVA'IA 6.5. 'innd cont de Lema 6.3. deducem c $: Pentru orice n! , n/2 !i 1.i, j.n avem egalitatea: Pi j=DiTi j(1) Ti j(-1) Tj i(1). n particular, det (P i j)= -1, deci Pi j!GLn(3). De asemenea avem urm torul rezultat: LEMA 6.6. Fie m, n ! , m, n/2 !i A!M m, n (3). 1) Dac$ Pij are ordinul m, atunci matricea P ijA se ob ine din A permutnd linia i cu linia j. 2) Dac$ Pij are ordinul n, atunci matricea AP ij se ob ine din A permutnd coloana i cu coloana j. DEFINI'IA 6.7. Fie m, n ! , m, n/2 !i A, B!M U!GLm(3) !i V!GLn(3) a.. UAV=B.
m, n

(3). Vom

spune c$ A este aritmetic echivalent $ cu B, !i vom scrie A~B, dac$ exist$ Se verific imediat c rela#ia ~ este o echivalen# pe M m, n (3). LEMA 6.8. Oricare ar fi A!M
m, n

d1 d1, , dr! * a. . A~ 0

0 d2 O dr !M m, n (3). 0 O 0 Demonstra#ie Pentru fiecare matrice A= ai j 1i m !M m, n (3) definim:

(3) exist$ 0.r.min{m, n} !i

( )

1 j n

186

0 daca det ( A) = 0 m( A ) = min a ij cu aij 0 daca det ( A) 0.

Vom face induc#ie matematic dup m(A). Lema este n mod evident adev rat dac A=Om, n. S presupunem c A6Om, pentru toate matricele B!Mm, Mm-1,n-1 (3). Exist atunci 1.i0.m $i 1.j0.n a.. m(A)=
n

$i c lema este adev rat

n (3) cu m(B)< m(A) ca $i pentru matricele din

a i0 j0 . Prin diferite

permut ri de linii $i coloane ale lui A putem presupune c i0=j0=1 (adic A~ P 1i0 AP 1 j0 ). Astfel, putem presupune c m(A)=a11 $i chiar mai mult c a11 >0 (c ci dac a11 < 0, atunci n loc de A putem lua D1A). Cazul 1. Presupunem c a11|a1j pentru 2.j.n $i a11|ai1 pentru 2.i.m, adic exist q1j , qi1 !3 a.. a1j=a11=q1j cu 2.j.n $i ai1=a11 =qi1 cu 2.i.m. Adunnd la coloanele 2, 3, , n coloana 1 a lui A nmul #it respectiv cu q12, -q13, ., -q1n $i procednd analog pentru linii, ob #inem: A~

a11 0

0 , cu A2 !Mm-1, n-1(3). A 0 !M m-1 , n-1 (3), 0 O 0

d2 O dr V2!GLn-1(3) a.. U AV = 0
unde di! * pentru 2.i.r.

Aplicnd ipoteza de induc#ie lui A2 deducem c exist U2!GLm-1(3) $i

187

1 0 1 0 , V= , d1=a11 avem U!GLm(3), 0 V 0 U 0 d1 d2 O a11 0 V!GLn(3) $i A~U V= d r 0 A 0 O 0 0


Alegnd U= cu A!Mm, n(3). Cazul 2. S presupunem c exist n prima linie (sau prima coloan ) a lui A un element (s zicem a1 j0 , cu 2.j0.n) ce nu divide pe a11. mp r#ind pe

a1 j0 la a11 putem scrie a1 j0 = a11 q1 j0 + r1 j0 cu 0 < r1 j0 < a11.


Adunnd la coloana j0 a matricii A coloana nti nmul#it cu - q1 j0 se ob#ine o matrice B~A care are n pozi #ia (1, j0) elementul

r1 j0 .

Cum m(B). r1 j0 <a11=m(A), conform ipotezei de induc#ie B este echivalent cu o matrice de forma celei din enun# $i atunci $i A va avea acelea$i proprietate. & OBSERVA'IA 6.9. Analiznd demonstra#ia Lemei 6.8. se observ c matricea diagonal cu care A este echivalent se ob#ine aplicnd asupra lui A un num r finit de transform ri elementare. LEMA6.10. Orice matrice unimodular $ U!GLn(3), este egal$ cu produsul unui num$r finit de matrice elementare. Demonstra#ie . Conform Observa#iei 6.9. exist matricele elementare R1, ., Rs, Q1, , Qt a..

188

d1 R1 . RsUQ1 Qt =D= 0

d2 O dr

0 !M m ( 3 ) . 0 O 0

Cum 1=|det (U)|=det (D), rezult c det (D)60, deci r=n. Din di! *, 1.i.n $i d1dn=1 deducem c d1=d2==dn=1, adic D=In $i atunci U=R1-1 . Rs-1Qt-1 Q1-1. Din

Tij-1 (l ) = Tij (- l ) $i

1 Di-1 = Di rezult c $i matricile Ri-1 $i Q sunt elementare, deci U este j

produs finit de matrici elementare. & LEMA6.11. Pentru orice a, b!3 avem

a 0 (a, b ) 0 b ~ 0

0 . [a, b]

Demonstra#ie Fie d=(a, b) $i a1, b1!3 pentru care a=da1 $i b=db1 . Conform Corolarului 2.7. de la Capitolul 6, exist h, k!3 a.. d=ha+kb, de unde 1=ha1+kb1. Alegnd U=

1 - kb1

1 h - b1 $i V= k a avem c ha1 1

det (U)=det (V)= ha 1+kb1=1, adic U, V!GL2(3) $i cum ab=(a, b) [a, b] ob #inem c :

a 0 a 0 (a, b ) 0 b ~U V= 0 b 0

0 .& [a, b]

n cele ce urmeaz vom prezenta un rezultat important (cunoscut sub numele de Teorema factorilor invarian #i ). 189

TEOREMA 6.12. Fie m, n ! , m, n/2 !i A!M

m, n

(3). Atunci

exist$ f1, ,fr! * cu r.min {m, n} unic determina i a.. f1|f2||fr !i

f1 O A~ 0

fr

0 !M m, n (3). 0 O 0

Demonstra#ie . Conform Lemei 6.10. avem

d1 O dr A~ 0

0 =D cu di! * 1.i.r.min{m, n} 0 O 0

iar D!Mm, n (3) . F cnd la nevoie permut ri de linii sau coloane putem presupune c d1.d2..dr. Dac pentru i<j, di nu divide dj, atunci conform Lemei 6.11. exist matricile unimodulare U=

x y p q , V= s t a.. z w 0 (d i , d j ) V= dj 0

di 0

0 di , d j

Consider m acum matricele U2 de ordin m ce se ob #ine din Im punnd pe pozi#ia (i, i) pe x, pe pozi#ia (j, j) pe w, pe pozi #ia (i, j) pe y iar pe pozi #ia (j, i) 190

pe z $i matricea V2 de ordin n ce se ob#ine din In punnd pe pozi#ia (i, i) pe p, pe pozi#ia (j, j) pe t, pe pozi#ia (i, j) pe q iar pe pozi #ia (j, i) pe s. n mod evident, U2!GLm(3), V2!GLn(3) iar matricea U2DV2 se ob#ine din D nlocuind pe di cu (di , dj ) iar pe dj cu [di , dj ] . Dac d1|dj , 2.j.r atunci se define$te f1=d1. Dac exist j/2 a.. d1?dj atunci d1 se nlocuie$te cu (d1, d2 ), iar dj cu [d1, dj ] $i observ m c n acest caz (d1, d2) < d1 $i (d1, d2) | [d1, d2 ]. Dup un num r finit de pa$i se ajunge la

0 0 O 0 d j cu 2.j.r $i se ia f1= d1 . Dac d 2 d j , 3.j.r atunci vom lua f2= d 2 . cu d 1


n caz contrar, se aplic procedeul de mai nainte $.a.m.d.. Astfel, dup un num r finit de pa$i se ob#ine o matrice de forma celei din enun # echivalent cu A. S ar t m acum unicitatea numerelor r, f 1, f2, , fr . Pentru matricea A prin >i(A) vom nota cel mai mare divizor comun al minorilor de ordin i al matricei A. Atunci dac A~B n mod evident >i (A)=>i (B), i=1, 2, , n iar pentru

d 1 d2 O A~ d r 0

f1 O matricea D= 0

fr

0 cu f1|f2||fr , avem 0 O 0

>1(D)=f1, >2(D)=f1f2, ..,>r(D)=f1f2fr iar >i(D)=0, pentru r .i.min {m, n} . Cu aceasta teorema este complet demonstrat . & 191

DEFINI'IA determinat$

6.13.

Dac$

A!Mm,n(3),

atunci

matricea

unic

f1 O B= 0

fr

0 !M 0 O 0

m, n

(3), cu

f1|f2||fr

a.. A~B se

nume!te forma diagonal canonic $ a lui A. Numerele f1, , fr >1 se zic factorii invarian i ai lui A. Exemplul[14] S g sim forma diagonal canonic a matricei

6 2 - 12 8 A= - 6 0 12 - 6 . 12 2 - 24 14
nmul#ind pe rnd la dreapta matricea A cu matricile P 12, T12(-3), T13(6), T14(-4) de ordin 4 $i apoi la stnga cu matricea T31(-1) de ordin 3, se ob #ine matricea B=T31(-1) A P12

0 0 2 0 - 6 . T12(-3) T13(6) T14(-4)= 0 - 6 12 0 6 - 12 6

nmul#ind la stnga matricea B cu matricea D2 de ordin 3, apoi pe rnd la dreapta cu matricile T23(2), T 24(-1) de ordin 4 $i n sfr$it la stnga cu matricea T32(-1)

2 0 0 0 de ordin 3 se ob #ine matricea D=T32(-1) D2 B T23(2) T24(-1)= 0 6 0 0 0 0 0 0


ce reprezint forma diagonal canonic a matricei A, 2 $i 6 fiind factorii invarian#i ai acesteia.

192

0 0 1 Fie U=T32(-1) D2 T31(-1)= 0 - 1 0 -1 1 1

$i

2 - 1 0 - 1 . 1 0 0 1 2 0 0 0 Avem U!GL3(3), V!GL4(3) $i UAV= 0 6 0 0 . 0 0 0 0 0 1 1 - 3 V=P12 T12(-3) T13(6) T14(-4) T23(2) T24(-1)= 0 0 0 0
Cu ajutorul celor stabilite anterior vom studia n continuare sistemele liniare de m ecua#ii cu n necunoscute:

a11 X 1 + ... + a1n X n = b1 a X + ... + a X = b 21 1 2n n 2 (S) ........................................ a m1 X 1 + ... + a mn X n = bm


cu coeficien#ii aij , bj !3, 1.i.m, 1.j.n. Prin solu#ie ntreag$ a lui (S) n#elegem un n-uplu (E1, En)!3
ij j
n

a..

a l
j =1

= bi pentru orice 1.i.m.

X1 b1 a11 L a1n M , b= M $i X= M atunci Dac not m A= M X b a n m1 L a mn m


sistemul (S) se scrie matricial sub forma AX= b.

193

DEFINI'IA
n

6.14.

Dac$

U!GLn(3),

U=

(u )

ij 1i m 1 j n

atunci

transformarea X i=

u Y
ij j =1

, 1.i.n ( sau matriceal X=UY) se nume !te

Y1 substitu ie ntreag$ unimodular$ , unde Y= M . Y n


PROPOZI'IA 6.15. Fie U!GLn(3), U= -i, .i cu 1.i.n a.. .i=

(u )

ij 1i m 1 j n

!i numerele reale

u a
ij j =1

, 1.i.n.

Atunci .i!3 pentru 1.i.n dac$ !i numai dac$ -j!3 pentru 1.j.n. Mai mult, ( .1, .n ) este solu ie ntreag$ a sistemului (S) dac$ !i numai dac$ ( -1, -n ) este solu ie ntreag$ a sistemului (AU)Y=b. Demonstra#ie O implica#ie este evident . S presupunem acum c ,i!3 pentru 1.i.n $i fie V!GLn(3) a..

VU=UV=In.

Atunci

n v1i b i a1 a1 b1 i =1 de unde M = VU M = V M = n M a a b n n n v ni b i i =1

deducem c +i!3 pentru 1.i.n. Ultima afirma#ie este evident . & LEMA 6.16. Dac$ a1, an!3, atunci exist$ U!GLn(3) a.. ( a1, , an ) U = (d, 0, , 0), unde d=( a 1, a2, an ). Demonstra#ie Facem induc#ie matematic dup n $i s ar t m la nceput c lema este adev rat pentru n=2. $i a 2 = da Dac d=(a1 , a2 ), atunci a1 = da1 2 cu a1 , a 2 !3 iar

, a2 )=1, de unde deducem c exist h, k!3 a.. ( a1


U=

+ ka ha1 2 = 1 $i fie

h - a 2 + ka 2 = 1 deducem c U!GL2(3)). k a . (cum det (U)= ha1 1


194

+ a 2 a1 ) = (d, 0). Avem c (a1, a2 ) U= ( ha1 +ka2 , - a1 a 2 Fie n>2 $i s presupunem c lema este adev rat pentru n-1. Atunci
exist V1!GLn-1(3) a.. ( a2, ,an ) V1 = (d1, 0, 0 ), unde d 1=(a2 , a3, an) astfel c dac not m V=

1 0 !GLn(3) avem 0 V1

( a 1, ,an ) V=(a1, d1, 0, 0). Conform cazului n=2 exist W1!GL2(3) a.. (a1, d1)W1=(d1, 0), unde d=(a1, d1).

W1 Dac alegem W= 0

0 1 !GLn(3) atunci W!GLn(3) $i O 1

(a1, ,an ) U=(d, 0, 0), unde U=VW ( se observ c d=(a1, ,an )).& S consider m acum ecua#ia (C) a1X1+.+anXn=b, cu a1, ,an, b!3. Pentru n=2 am ar tat n 1 de la Capitolul 12, n ce condi #ii aceast ecua#ie are solu#ii ntregi $i felul n care acestea se g sesc. n cele ce urmeaz vom face acela$i lucru cu ecua#ia (C) pentru n/2 (prezentnd deci o generalizare a Lemelor 1.1 $i 1.2 de la Capitolul 12). TEOREMA 6.17. Ecua ia (C) cu coeficien i ntregi admite solu ii ntregi dac$ !i numai dac$ d| (a1, ,an) . Dac$ U!GLn(3), U= u ij este a.. (a1, ,an)U=(d, 0, 0), (conform Lemei 6.16.) atunci

(x

( )

1 i , j n

0 1

0 ,..., x n cu

xi0 = u i1

b , 1.i.n este solu ie ntreag$ particular$ a ecua iei (C). Solu ia d


(x1, xn) cu
n

general$ din 3 a ecua iei (C) va fi de forma

xi = xi0 + u ij t j , tj!3, 1.i.n.


j =2

Demonstra#ie Dac U!GLn(3) ca n enun#, atunci f cnd substitu#ia ntreag unimodular X=UY ob#inem (d, 0, , 0)Y=b. Atunci deducem c aceast ultim ecua#ie are solu#ie ntreag dac $i numai dac d|b iar o solu#ie 195

ntreag particular a acesteia este

b ,0,....0 , solu#ia general fiind de forma d

b , t 2 ,...t n cu tj!3, 2.j.n arbitrare. Conform Propozi #iei 6.15. ob#inem c d b u b d 11 x10 d 0 M = U = M este solu#ia ntreag particular a x0 M u n1b n 0 d
ecua#iei (C) iar dac (x1, xn) este solu#ia ntreag oarecare a lui ( C) , atunci

u11b n b + u t j j 1 d x1 d j = 2 n t2 adic x i = x i0 + u ij t j , 1.j.n. & = = U M M n j =2 x M un1b + u t n nj j t d j =2 n


OBSERVA'II 1. Cnd d|b, descrierea solu #iilor ntregi ale ecua#iei (C) din enun#ul teoremei precedente se face cu ajutorul matricei unimodulare U. Calculul lui U se face folosind de n-1 ori algoritmul lui Euclid extins. ntr-adev r, ntr-o prim etap cu ajutorul acestui algoritm determin m succesiv : d 1=(an-1, an), h1an-1+k1an=d1 d2=(an-2, d1 ), h2an-2+k2d1=d2 .. d=dn-1=(a2, dn-2 ), hn-1an-1+k1dn-2=dn-1=d $i atunci avem

196

1 O 1 U= 0

h1 - a n -1 k1 - a n 0

1 O 1 0

h2 - d 1 k 2 a n -1

0 1

hn -1 - d n -1 0 k a n -1 1 . unde 1 O 0 1 d1 = d 2 d1 , a n - 2 = d 2 a n - 2 , etc.
LEMA 6.18. Fie n/2 !i A= exist$ U!GLn(3) a..

, a n = d1 a n

a n-1 = d1 a n -1 ,

2. Cnd n=2 ob#inem rezultatele de la 1, Capitolul 12.

(a ) !M (3) a.. 5=det (A) > 0. Atunci


ij
n

c11 c12 AU= .... c n1


1.i.n.

0 c 22 .... cn 2

L 0 L 0 unde ci i >0, 1.i.n !i 0.ci 1, ci 2, , ci, i-1< ci i , .... .... L c nn

Demonstra#ie Fie c11=(a11, a12, a1n). Conform Lemei 6.16. exist

U1!GLn(3) a.. (a11, a12, , a1n) U1 =(c11, 0, ,0)

197

c11 a 21 $i deci AU1= .... a n1


lem

0 a 22 .... a n2

L 0 n L a2 !3. Aplicnd din nou aceia$i unde a ij .... .... L a nn 1 0 avem U2!GLn(3) $i se ob#ine 0 V 0 .... 0 0 .... 0 .... a3 n . a 33 .... .... .... .... a nn a n3 c21= a 21 ,
n caz contrar scriem a223, ,a22n) V =(c22, 0, ,0) unde

g sim V!GLn-1(3) a.. (a222,

c22=(a222, a223, ,a22n). Punnd U2=

c11 a 21 AU 1U2= a31 ... a n1


Dac

0 c 22 a 32 .... a n2

= c 22 q 21 + r21 cu 0.r21<c22 . Atunci a 21 c11 0 .... 0 AU1U2T21(-q21)= r21 c 22 .... 0 $i alegem c21=r21. Continund .... .... .... ....
se g se$te matricea U=U1U2..!GLn(3) a.. matricea AU este de forma celei din enun#. & TEOREMA 6.19. Fie un sistem de de n ecua ii liniare cu n
n

0. a 21 <c22

lu m

a matricea A= (a )
necnoscute (S1)
j =1 n

ij

X j = bi , 1.i.n a.. aij, bi!3 !i det (A) > 0 (A fiind


).

ij 1 i , j n

Atunci sistemul (S1) admite solu ie ntreag$ dac$ !i numai dac$ congruen ele (C)

a
j =1

ij

X j bi (m ) , 1.i.n au solu ie ntreag$ pentru

orice m!3 a.. 0<m.5. 198

Demonstra#ie. Implica#ia de la stnga la dreapta este evident . S presupunem acum c ( C ) are solu#ie pentru orice 0<m.>. Scriem pe (C) sub form matricial astfel AXAb (m), 0<m.>. Dac X=UY este o substitu#ie ntreag unimodular , atunci

(AU)YAb (m), 0<m.>. Alegnd U!GLn(3) dat de Lema 6.16. sistemul (S1) c11Y1 = b1 c Y + c 22 Y2 = b 2 devine 21 1 ............................. c n1Y1 + c n 2 Y2 + ... + c nn Yn = b n . Evident >=c11c22cn n , deci 0<c11c22cii .>, 1.i.n. Cum 0<c11.>, congruen#a c11Y1=b1 (c11) are solu#ie, deci exist h, k!3 a.. c11h=b1+kc11 , de unde c11+1=b1 cu +1=h-k. Adunnd ecua#ia c11Y1=b1 (nmul#it cu c21) cu ecua#ia c12Y1+c22Y2= =b2 (nmul#it cu c11) se ob#ine c11c22Y2 =- c21b1+c11b2 . Conform ipotezei, congruen#a c11c22Y2 A c21b1+c11b2 (c11c22) are solu#ie, deci exist h2, k2 !3 a.. c11c22 h2 = - c21c11+1+c11b2 +k2c11c22. Simplificnd cu c1160, ob#inem c21+1+c22+2 =b2, unde +2=h2-k2!3. Analog, din primele trei ecua#ii n Y1, Y2, Y3 ob#inem c11c22 c33Y3 =c11c22b3 - c31 c22b1 - c11c22b1-c11 c32b2 +c21c32b1. nlocuind b1= c11+1 , b2 =c21+1+c22+2 $i pornind de la condi #ia ca aceast ultim ecua#ie s fie solubil modulo c11 c22c33 , g sim +3!3 a.. c 31+1+c32+2 +c33+3=b3 . Continund n acela$i mod g sim o solu#ie ntreag (+1, +2, , +n) a sistemului AUY=b $i atunci (,1, ,2, , Fn), unde este o solu#ie ntreag a sistemului (S1) AX=b. & Observa#ie Cum 3m-urile sunt finite rezult din teorema de mai sus c putem stabili printr-un num r finit de ncerc ri dac sistemul (S1) are sau nu solu#ii ntregi. Teorema urm toare solu#ioneaz cazul sistemelor omogene.

b i = u ija j
j =1

, 1.i.n

199

TEOREMA 6.20. Sistemul de ecua#ii liniare (S2)

a
j =1

ij

X j = 0 , 1.i.m cu aij!3, (m<n) admite o solu ie x j (a1 a 2 ....a m )


1 n-m ,

ntreag$ netrivial$ (x1, xn) ce satisface condi ia 1.j.n, unde

ai = aij , 1.i.m.
j =1

Demonstra#ie Fie Li(X1, Xn)=

a
j =1

ij

X j , 1.i.m, bi= a ij ,
aij > 0

- ci =
atunci

aij < 0

ij

, 1.i.m.

Atunci ai=bi+ci cu 1.i.m $i fie a! . Dac 0.+j.a cu 1.j.n,

- ci a Li (a 1 ,...,a n ) bi a , 1.i.m,
Alegnd

deci Li(+1, +n) ia cel mult ai a +1 valori ntregi.

a = (a1 ...a m )
-1, de unde

m- n

(partea ntreag !) atunci

a > (a1 a 2 ....a m )

n-m

(a + 1)n > (a + 1)m a1 ...a m > (a1a + 1)....(a m a + 1) .


Deducem c exist (+21, ,+2n)6 (+221, ,+22n ) cu 0.+2i, +22i .a a.. Alegnd xi=+2i-+22i , 1.i.n, avem c
1 n- m ,

Li (+21, +2n )= Li (+221, +22n ), 1.i.n.

x j (a1 a 2 ....a m )

Li (x1, xn)=0, 1.i.m $i

1.j.m. Mai mult, (x1, xn)6(0, 0) $i astfel

teorema este demonstrat . & Cu ajutorul formei diagonal canonice a matricelor din Mm, n (3) putem acum solu#iona problema existen#ei $i descrierii solu#iilor ntregi ale unui sistem de m ecua#ii liniare cu n necunoscute cu coeficien#i ntregi.

200

TEOREMA 6.21. Fie sistemul de m ecua ii liniare n n necunoscute cu coeficien i ntregi (S) Dac$ A=

a
j =1

ij

X j = bi , 1.i.n .

(a )

ij 1 i m 1 j n

!M m, n (3) !i U!GLm(3), V!GLn(3), U=(uij),

f1 O V=(vij) sunt a.. UAV= 0


ca

fr

0 (vezi Teorema 0 O 0

6.12.), atunci condi ia necesar$ !i suficient$ ca (S) s$ aib$ solu ii ntregi este

fk

u kj b j , 1.k.r !i
j =1

u b
ij j =1 0 1

= 0 , r<j.min {m, n}.

n aceste condi ii

(x

0 0 ,...x n , unde xi =

k , j =1

r ,m

vik u kj b j fk

, 1.i.n, este

o solu ie ntreag$ a sistemului (S). Mai mult, un sistem (x1, xn) de numere ntregi este solu ie a lui (S) dac$ !i numai dac$

xi = xi0 +
-1

k = r +1

ik k

t , tk!3, 1.i.n.
DY=UL unde

Demonstra#ie Scriem sistemul (S) sub form matricial AX=b. Cum UAVV X=Ub, notnd Y=V-1X avem (S2)

f1 O D= 0

fr

0 . 0 O 0
201

Deducem c

f k Yk = u jk b j , 1.k.r $i 0 = u kj b j , r < k.
j =1

.min {m, n}. Este clar c DY=Ub admite solu#ie ntreag dac $i numai dac

fk

u
j =1

kj

b j , 1.k.r $i o solu#ie particular a sistemului (S2) este:

m u b m u1 j b j rj j ,...., , 0 ,..., 0 fr j =1 j =1 f 1

iar solu#ia general a sistemului (S2) este :


m u b m u1 j b j rj j cu tr+1, , tn arbitrari din 3. t t ,...., , ,..., r + n 1 f f j = j = 1 1 r 1

Cum X=VY deducem c solu#iile sistemului (S) sunt cele din enun#. & Exemplu S consider m sistemul : 6 X 1 + 2 X 2 - 12 X 3 + 8 X 4 = 10 ( C) - 6 X 1 + 12 X 3 - 6 X 4 = 18 12 X 1 + 2 X 2 - 24 X 3 + 14 X 4 = -8. 6 2 - 12 8 Avem A= - 6 0 12 - 6 . 12 2 - 24 14 Dup exemplul de la Teorema 0 1 0 0 1 1 - 3 unde U= 0 - 1 0 , V= 0 0 - 1 1 1 0 0 Avem r=2, f1=2, f2=6. 2 0 1 0 2 0 0 0 6.12. avem c UAV= 0 6 0 0 , 0 0 0 0 - 1 - 1 . 0 1

202

Din

u
j =1 3

1j

b j = 10 $i 2|10 , unde 2=f1 b j = -18 $i 6|-18 , unde 6=f2 bj = 0

u
j =1 3

2j

u
j =1

3j

rezult c sistemul (C) are solu#ie n numere ntregi (conform Teoremei 6.21.). Urmnd algoritmul dat de Teorema 6.21., deducem c o solu#ie particular a lui (C) este

(x

0 1

0 0 0 , x2 , x3 , x4 =( -3, 14, 0, 0) iar solu #ia general este:

x1= -3+2t3 - t4 x2=14 - t4 x3=t3 x4=t4 , cu t3, t4!3 arbitrare. Observa#ie Acest paragraf a fost redactat n cea mai mare parte dup lucrarea [ 14]. CAPITOLUL 13: PUNCTE LATICIALE N PLAN 6I SPA'IU 1.Puncte laticiale n plan S consider m planul euclidian G raportat la un sistem ortogonal de axe de coordonate. DEFINI'IA 1.1. Un punct M de coordonate (a, b) din planul euclidian G se zice punct laticial dac$ a, b3. TEOREMA 1.2. (Steinhaus-Sierpinski) Pentru fiecare num$r n
*

exist$ n planul euclidian G un cerc ce con ine n interiorul s$u exact n puncte laticiale.

203

Demonstra#ie S consider m n G punctul C de coordonate ( 2 , 1/3) $i s demonstr m c dac M(a, b) $i N(c, d) sunt dou puncte laticiale din G ce au aceea$i distan# la punctul C, atunci MN. ntr-adev r, dac CMCN, atunci: (a- 2 )2 + (b1 2 1 ) = (c- 2 )2 + (d - )2 2(c-a) 3 3 2 =c2 + d2 - a2 - b2 + 2 (b-d), 3

de unde a=c $i c2 + d2 - a2 - b2 + d+b-

2 2 (b-d) = 0 (d-b)(d+b- )=0 $i cum b, d 3, 3 3

1 0, ceea ce implic b=d, adic MN. & 3 )innd cont de observa#ia de mai nainte, punctele laticiale din G pot fi

ordonate n func#ie de distan#ele lor la C( 2 , 1/3). Fie deci M1 punctul laticial a c rui distan# d1 la C este cea mai mic , M2 urm torul (adic acel punct pentru care distan#a d2 de la M2 la C este cel mai apropiat num r natural fa# de d1) $.a.m.d. Ob#inem astfel $irul M1, M2, de puncte laticiale cu proprietatea c dac not m prin di distan#a de la Mi la C, i=1, 2, , atunci d 1<d2<d3<. Atunci cercul cu centru n punctul C $i de raz dn+1 con#ine n interiorul s u doar punctele laticiale M1, M2, , Mn ce sunt n num r de n $i astfel teorema este demonstrat . & Observa#ie Exist un rezultat datorat lui Hugo Steinhaus potrivit c ruia pentru fiecare num r natural n s u exact n puncte laticiale. exist un cerc de arie n ce con #ine n interiorul

TEOREMA 1.3. (A. Schinzel) Pentru orice num$r natural n exist$ n G un cerc ce con ine pe circumferin a sa exact n puncte laticiale. Demonstra#ie Dac n este par, adic n=2k cu k , vom demonstra c 1 (k-1)/2 cercul de centru (1/2, 0) $i raz 5 con#ine pe circumferin#a sa exact n 2 puncte laticiale, pe cnd atunci cnd n este impar, adic n=2k+1 cu k , cercul

204

de centru (1/3, 0) $i raz laticiale.

1 k 5 con#ine pe circumferin#a sa exact n puncte 3

Pentru aceasta vom apela la Teorema 1.7. de la Capitolul 11 potrivit c reia num rul total de perechi (x, y) din 33 pentru care x2+y2=n este egal cu 4(d1(n)-d2(n)), unde d1(n) este num rul divizorilor lui n de forma 4t+1 iar d 2(n) este num rul divizorilor primi de forma 4t+3. (atunci cnd num r m perechile (x, y) facem distinc#ie ntre (x, y) $i (y, x) pentru xy). Cazul 1 : n=2k cu k . S consider m ecua#ia (1) x2 + y2 = 5k-1. To#i divizorii lui 5k-1 sunt puteri ale lui 5, deci to #i ace$ti divizori sunt de forma 4t+1. Cum num rul acestor divizori este k deducem c d1(5k-1)=k iar cum d2(5k-1)=0 atunci num rul perechilor (x, y) 33 pentru care x2 + y2=5k-1 este 4(k-0)=4k. Cum 5k-1 este impar trebuie ca x sau y s fie impar. 1 (k-1)/2 Cercul de circumferin# C1(1/2, 0) $i raz 5 are ecua#ia: 2 1 1 (a- )2 + b2 = 5k-1 (2a-1)2 + 4b2 = 5k-1 (2a-1)2 + (2b)2 = 5k-1. (2) 2 4 Deci un punct M( a, b) se afl pe circumferin#a cercului C1 dac $i numai dac coordonatele sale ( a, b) verific (2). Se observ c dac M(a, b) se afl pe cercul C nu rezult c $i M(b, a) se afl pe C1. Astfel, num rul punctelor M( a, b) de pe cercul C 1 cu ( a, b)3 este egal cu num rul perechilor ordonate ( a, b)3 ce verific ecua#ia (2). Se observ c ecua#ia (2) este de tipul (1), astfel c num rul solu#iilor (a, b)3 ale lui (2) este egal cu num rul solu#iilor ordonate (x, y) 3 ce verific (1), adic cu 4k/2=2k=n. Cazul 2: n=2k+1. Analog ca n cazul 1, dac vom considera ecua#ia (3) x2+y2=52k, num rul perechilor (x, y)3 ce verific (3) este egal cu : 4[d 1(52k)-d2(52k)] =4[(2k+1)-0]=8k+4. S observ m acum c punctul M(a, b) se afl pe circumferin#a cercului 1 k 1 1 C2(1/3, 0) $i raz 5 (a- )2 + b2 = 52k (4) (3 a-1)2 + b2 = 52k. 3 3 9 Astfel num rul de puncte laticiale M( a, b) de pe C2 este egal cu num rul solu#iilor ordonate (x, y) 3 ale ecua#iei (3) cu x=3a-1 si y=3b. Pentru a determina num rul acesta, s mp r#im cele 8k+4 solu#ii din 3 ale lui (3) n 8 familii: (x, y), (x,-y), (-x, y), (-x,-y), (y, x), (y, -x) (-y, x), (-y, -x). 205

Dac de exemplu x=0 atunci familia se reduce la 4 solu #ii: (0, y), (0,-y), (y, 0), (-y, 0). De asemenea, dac x=y exist numai 4 solu#ii n familia de mai sus: (x, x), (-x, x), (x, -x), (-x, -x). (cum 52k este impar aceast posibilitate este exclus ). Solu#iile lui (3) cu o component nul sunt: (0, 5 k), (0, -5k), (5k, 0) $i k (-5 ,0). n consecin# , familia celor 8k+4 solu#ii se mparte n k familii de 8 solu#ii $i o familie de 4 solu#ii. Observ m de asemenea c ecua#ia (4) este de tipul (3) (cu x -1(3) $i y 0(3) ) $i c 52k = 25k 1k 1(3). Deoarece p tratul unui num r ntreg este congruent cu 0 sau 1 modulo 3, dac (x, y)3 $i x2+y2=52k atunci trebuie ca unul dintre x2 sau y2 s fie congruent cu 1 iar cel lalt cu 0 modulo 3. Fie x $i x termenii din familia celor 8 solu#ii ce sunt divizibili prin 3. n acest caz y sau y este congruent cu 1 modulo 3. S presupunem c y -1(3). Atunci numai cele 2 solu#ii (y, x) $i (y, -x) au primul termen congruent cu 1 modulo 3 $i pe al doilea congruent cu 0 modulo 3 (observ m c n familia celor 4 solu#ii, (-5k, 0) sau (5 k, 0) este de tipul de mai nainte ). n concluzie, fiecare din cele k familii de 8 solu#ii (x, y) ale lui (3) con#in exact 2 solu#ii ale lui (4) $i o singur familie din cele 4 solu#ii ale lui (3) con#ine o singur solu#ie a lui (4). Ob #inem n total 2k+1=n solu#ii pentru (4), astfel c pe cercul C2 se afl exact 2k+1=n puncte laticiale. & TEOREMA 1.4.(G. Browkin) Pentru orice num$r natural n , exist$ n G un p$trat ce con ine n interiorul s$u exact n puncte laticiale. $ir P1, P2, Pentru aceasta vom utiliza func#ia f : 339+, 1 1 f(x, y)= x + y 3 + x 3-y, pentru orice (x, y)33. 3 3 S ar t m la nceput c dac (a, b), (c, d) 33 $i f(a, b)=f(c, d), atunci (a, b)=(c, d), adic a=c $i b=d. ntr-adev r, egalitatea f(a, b)=f(c, d) este echivalent cu : 1 1 1 1 p(a + b 3 - ) + q (a 3 - b ) = r (c + d 3 - ) + s (c 3 - d ) 3 3 3 3 cu p, q, r, s{1}. Demonstra#ie Vom ncerca s ,,ordon m punctele laticiale din G ntr-un

206

)innd cont c

3 este num r ira#ional $i c o egalitate de forma

x+y 3 =x2+y2 3 cu (x, y), (x2 y2)33 implic x=x2 $i y=y2, din (1) deducem c : r- p =0 $i (2) pa-qb-rc+sd+ 3 q-s rd+sc-pb-qa+ =0 3 r- p q-s , 3, lucru posibil doar Din (2) deducem cu necesitate c 3 3 pentru r = p $i q = s, astfel c (2) cap t forma echivalent : (3) p(a-c)+q(d-b)=0 p(d-b)+q(c-a)=0 Multiplicnd prima egalitate din (3) cu p $i pe a doua cu q $i sc zndu-le, ob#inem egalitatea (a-c)(p2+q2)=0 2(a-c)=0 a=c. Deducem atunci $i c b=d. S vedem ce interpretare geometric are f. Pentru aceasta consider m n G dreptele d $i d de ecua#ii: 1 ( d ) : x + y 3 - =0 3 1 =0 ( d ): x 3 - y 3 Evident, d ^ d $i ( d ) ( d )={(1/3, 0)}.

207

P(x, y) (d) . Q . O

(d$) S x

1 R( ,0) 3 Fig. 3 )innd cont de formula ce d distan#a unui punct P(x, y) la ( d ) $i respectiv ( d ), deducem imediat c f(x, y)=2PQ+2PS, adic f(x, y) este perimetrul dreptunghiului PQRS din figura 3 de mai sus. G sim atunci un punct laticial P1(x1, y1) n apropierea lui R pentru care f(x1, y1) este cea mai mic valoare a lui f(x, y) (cnd x, y3). Conform celor stabilite la nceput punctul P1 este unic. n felul acesta putem ordona punctele laticiale ntr-un $ir P1,P2,. (scriind c Pi(xi, yi) < Pi+1(xi+1, yi+1) +fi(xi, yi) < fi+1(xi+1, yi+1)). Dac Pn(xn, yn) este a n-lea punct laticial n aceast ordonare, s not m 1 1 an=f(xn, yn), iar h(x, y)=x(1+ 3 )+y( 3 -1)- 3 3 1 1 g(x, y)=x(1- 3 )+y(1+ 3 )- + 3 3 S consider m acum cele 4 drepte : h(x, y) = an+1 $i g(x, y) = an+1; se verific imediat c cele 4 drepte formeaz un p trat.

208

g = a n +1 g = - a n +1
P(x, y)

h = a n +1

h = - a n +1
O Fig. 4 Dac avem un punct laticial P(x, y) atunci -an+1<h(x, y)<an+1 |h(x, y)|<an+1, adic P se g se$te ntre dreptele de ecua #ie h(x, y)=an+1 $i h(x, y)= -an+1 $i reciproc. Similar, se deduce c punctul P(x,y) se afl ntre dreptele de ecua#ii g(x, y)=an+1 $i g(x, y)=-an+1 |g(x, y)|<an+1. Astfel punctul P(x, y) se afl n interiorul p tratului din figura 4 |h(x, y)|<an+1 $i |g(x, y)|<an+1. ns se verific imediat c pentru numerele reale a, b, c: a+b a-b |a|<c $i |b|<c | | +| |<c $i astfel : 2 2 |h(x, y)|<an+1 $i h ( x, y ) + g ( x, y ) h ( x , y ) - g ( x , y ) + <an+1 f(x, y)<an+1, 2 2 |g(x, y)|<an+1 adic! punctul laticial P(x, y) se afl ! n interiorul p!tratului din Fig. 4 dac ! %i numai dac! f(x, y)<an+1. Atunci n interiorul p!tratului din figur! sunt exact punctele laticiale P1, P2,,Pn, ce sunt n num r de n. & TEOREMA 1.5. (Pick) Fie P un poligon convex n plan care con ine m puncte laticiale n interiorul s$u, k puncte laticiale pe laturi sau vrfuri !i vrfurile sale sunt puncte laticiale. 209 x

k -1. 2 Demonstra#ie S demonstr m formula la nceput pentru cazul m=0, k=3 (aceasta exprim faptul c P este un triunghi cu vrfurile n nodurile re#elei $i care nu mai con#ine alte noduri pe laturi sau n interior). Atunci S=1/2 (vezi figura 5) Atunci Aria(P)=m+

A B C Fig. 5 S trecem acum la cazul general. Descompunem poligonul P n triunghiuri cu vrfurile n puncte laticiale $i care nu mai con#in puncte laticiale pe laturi sau n interior. Vom calcula num rul n de triunghiuri de mai sus n dou moduri exprimnd n dou moduri suma unghiurilor lor. Pe de alt parte suma unghiurilor lor este 180 o n iar pe de alt parte suma unghiurilor lor este egal cu suma unghiurilor poligonului $i a unghiurilor din jurul punctelor interioare, adic 180 o (k-2)+ 360 o m. Deci 180 o n= 180 o (k-2)+ 360 o m, de unde n=2m+k-2 $i cum S=n/2 k deducem c S= m+ -1. & 2 Observa#ii 1. Teorema lui Pick este valabil $i pentru poligoane oarecare (nu neap rat convexe), ns demonstra#ia ei este diferit de cazul convex. Pentru aceasta vom considera dou poligoane Q1 $i Q2 care au toate vrfurile n puncte laticiale $i care sunt adiacente prin una din laturile comune AB (vezi figura 6).

210

Fig. 6 k -1 este adev rat pentru 2 amndou aceste poligoane ; vom demonstra c n acest caz, formula va fi adev rat $i pentru poligonul mai mare Q, ob #inut prin reuniunea lui Q1 $i Q2. ntr-adev r, fie S1, m1 $i k1 aria, num rul punctelor laticiale din interiorul poligonului $i num rul punctelor laticiale de pe frontiera lui Q 1, iar S2, m2 $i k2-numerele corespunz toare pentru poligonul Q 2. S presupunem cunoscut c formula S= m+ 2 2 Vom nota cu k num rul nodurilor re#elei de p trate situate pe segmentul AB, care con#ine punctele A $i B. Pentru poligonul Q, aria sa S, num rul m de puncte laticiale din interiorul s u $i num rul k de puncte laticiale de pe frontiera sa vor fi exprimate cu ajutorul lui m1, m2, k1, k2 $i k astfel: S=S1+S2, m=m1+m2+( k -2) (la punctele laticiale interioare se vor ad uga toate punctele laticiale situate pe AB cu excep#ia lui A $i B) $i k=(k1- k )+(k2- k )+ 2 ( n ultimul termen +2 figureaz nodurile A $i B ). Deci: k k k + k 2 - 2k + 2 -1=m+ k -1. S=S1+S2= m1+ 1 -1+ m2+ 2 -1=(m1+m2+ k -2)+ 1 2 2 2 2 Formula de demonstrat la modul general se poate stabili acum inductiv. 2. Merit s mai amintim $i un rezultat datorat lui Hermann Minkowski legat de punctele laticiale: Conform ipotezei avem S1=m1+

k1 -1 $i S =m + k 2 -1.
2 2

211

Dac$ un poligon convex simetric fa $ de centrul s$u (care este un punct laticial ) nu mai con ine n interiorul s$u alte puncte laticiale, atunci aria sa este < 4 ( ca unitate de arie se consider $ aria unui p$trat al re elei). Nu vom prezenta aici demonstra#ia teoremei lui Minkowski deoarece ea este destul de laborioas , dar n esen# este asem n toare cu cea a teoremei lui Pick. (indic m cititorului lucrarea [13]). Pentru un num r natural n fie t(n)=num rul de reprezent ri ale lui n ca sum de dou p trate de numere naturale (dou reprezent ri fiind considerate diferite dac difer ordinea termenilor )- vezi Teorema 1.7. de la Capitolul 11. De exemplu : t(1)=4, t(2)=4, t(3)=0, t(5)=8, t(6)=0, t(7)=0, t(8)=4, t(9)=4, t(10)=8. Dup cum am v zut mai nainte orice num r prim de forma 4k+1are o unic reprezentare ca sum de dou p trate de numere naturale (dac nu #inem cont de ordinea termenilor ; vezi Propozi#ia 1.5. de la Capitolul 11). De aici deducem c dac p este prim de forma 4k+1, atunci t(p)=8 (c ci dac (a, b) este o solu#ie, atunci sunt solu#ii $i (b, a) ca $i (a, b), (b, a) ). Observ m c dac n=x2+y2 atunci |x|, |y| n , deducem imediat c t(n)4 n . Pentru n
*

, fie T(n)=t(1)+t(2)++t(n). Atunci T(n) este num rul de


*

solu#ii din 3 ale inegalit #ilor: 0< x2+y2 n. LEMA 1.6. Pentru orice n , T(n)=4
2

[ n] k =0

[ n - k 2 ].
|y| n , deci num rul

Demonstra#ie Dac x=0, atunci y n numerelor y pentru care 0 < x +y n este 2[ n ]. Dac
2 2

x=k0, atunci k2 n, deci |k| n iar y2 n-k2, adic

|y| n - k 2 (deci num rul y-cilor este 1+2[ n - k 2 ]; am adunat $i pe 1 deoarece y=0 trebuie considerat). Deoarece k{1, 2,, [ n ]} iar semnele nu influen#eaz valoarea lui k2, ob#inem c : T(n)=2[ n ]+2
[ n] k =1

[1 + 2 n - k 2 ] =4[ n ]+4 [ n - k 2 ] =4 [ n - k 2 ] .
k =1 k =0

[ n]

[ n]

Astfel, de exemplu pentru n=100, avem: 212

T(100)= = 4([ 100 ]) + [ 99 ] + [ 96 ] + [ 91] + [ 84 ] + [ 75 ] + [ 64 ] + [ 51] + [ 36 ] + + [ 19 ]) = 4(10 + 9 + 9 + 9 + 9 + 8 + 8 + 7 + 6 + 4) = 316

Interpretare geometric pentru T(n) Pentru n , 1+T(n) reprezint num rul de perechi din 32 ce satisface inegalitatea x2+y2n. Astfel 1+T(n) reprezint num rul punctelor laticiale din interiorul cercului Cn de centru (0, 0) $i raz n (eventual de pe circumferin# ). n continuare, la fiecare punct laticial vom asocia un p trat ce are centrul n punctul respectiv, laturile paralele cu axele de coordonate $i aria 1 (vezi Fig.7). y
.

0 Fig. 7

Dac not m cu P aria acoperit de p tratele asociate punctelor laticiale care nu sunt n afara cercului Cn este egal cu num rul acestora, adic P=1+T(n). 1 Cercul C1n de centru (0, 0) $i raz n+ con#ine n interior sau pe 2 circumferin# toate punctele acoperite de p tratele asociate punctelor laticiale din 1 Cn ( aceasta deoarece n mod evident este cea mai mare distan# posibil a 2 unui punct din interiorul p tratului de arie 1 la centrul p tratului). 1 2 Atunci P aria (C1n) P 4 ( n + ). 2

213

Pe de alt parte, dac not m cu C2n cercul de centru (0, 0) $i raz 1 2 , atunci din aria (C2n) P deducem c 4 ( n ) P. n2 2 1 nlocuind P=1+T(n) deducem c 4 ( n Cum 4 2 <5 $i 0< 1 2 )2-1< T(n) < 4 ( n -

1 2

)2-1.

$i

1 4-1<1 n deducem c : 2 1 2 1 ) -1= 4 n + 4 2 n + 4-1< 4 n + 6 n 4( n + 2 2 1 2 1 ) -1= 4 n - 4 2 n + 4-1> 4 n - 6 n 4( n 2 2 T (n) 6 -p < iar de aici deducem : n n T ( n) =p . n

de unde 4 n-6 n <T(n)<4 n+6 n

PROPOZI'IA 1.7.

lim n

Dup cum am v zut T(100)=316, deci T(100)/100=3,16. Analog T(400)=1256, deci T(400)/400=3,14 iar T(1000)/1000=3,148. Avem astfel posibilitatea de a aproxima pe 4 considernd valori din ce n ce mai mari pentru n. 2. Puncte laticiale n spa iu Consider m spa#iul 93 raportat la un sistem ortogonal de axe 0xyz. DEFINI'IA 2.1. Un punct M(x, y, z)93 se zice punct laticial, dac$ (x, y, z)33. Multe rezultate legate de puncte laticiale din plan au extinderi aproape imediate la puncte laticiale din spa #iu. LEMA 2.2. Dac$ p, q, r7 p=q=r=0. 214 !i p 2 + q 3 + r 5 7, atunci

Demonstra#ie p 2 +q 3 =k -r 5 , Deducem c
2 2

Fie de
2

p 2 + q 3 + r 5 = k 7.
2 2 2

Atunci

unde
2

2 p + 2 pq 6 + 3q = k - 2kr 5 + 5r 2 . de unde

2 pq 6 + 2kr 5 = k 2 + 5r 2 - 2 p 2 - 3q 2 7,

2 pq = 2kr = k + 5r - 2 p - 3q = 0 iar de aici p = q = r = 0. & TEOREMA 2.3. Pentru orice num$r natural n * exist$ n spa iu o sfer$ ce con ine n interiorul s$u exact n puncte laticiale . Demonstra#ie S ar t m la nceput c sfera de centru ( 2 , 3 , 5 ) are cel mult un punct laticial pe suprafa #a ei. ntr-adev r, s presupunem c pe suprafa#a sferei cu centrul n punctul de coordonate ( 2 , 3 , 5 ) exist dou puncte laticiale de coordonate (a, b, c) respectiv (d, e, f). Scriind c

(a - 2 ) + (b - 3 ) + (c - 5 ) = (d - 2 ) + (e - 3 ) + ( f - 5 )
2 2 2 2 2

ob#inem 2 2 (d - a ) + 2 3 (e - b ) + 2 5 ( f - c ) = d 2 + e 2 + f 2 - a 2 - b 2 - c 2 7 $i atunci conform Lemei 2.2., d-a = e-b = f-c = 0 +a = d, b = e, c = f. Analog ca n cazul plan (teorema 1.2.) putem ordona punctele laticiale din spa#iu ntr-un $ir cresc tor M1, M2, n func#ie de distan#ele d1, d2, ale acestora la punctul de coordonate ( 2 , 3 , 5 ). Astfel, sfera cu centrul n punctul de coordonate ( 2 , 3 , 5 ) $i raz dn+1 con#ine n interiorul s u exact n puncte laticiale din spa#iu $i anume pe M1, M2, , Mn . & TEOREMA 2.4. (T. Kulikowski) Pentru orice num$r natural n * exist$ n spa iu o sfer$ ce con ine pe suprafa a sa exact n puncte laticiale. Demonstra#ie Conform Teoremei 1.3. exist un cerc n planul 0xy de

ecua#ie (x - a )2 + ( y - b )2 = c (cu a, b, c7, c>0) ce trece prin exact n puncte laticiale de coordonate (x, y). Identificnd punctele laticiale de coordonate (x, y) din planul 0xy cu punctele de coordonate (x, y, 0) din spa #iul 0xyz putem trage

concluzia c cercul (x - a )2 + ( y - b )2 = c con#ine exact n puncte laticiale (x, y, 0) din spa#iu. S consider m acum sfera cu centrul n punctul de coordonate (a, b, 2 ) $i de raz c + 2 a c rei ecua#ie n sistemul de axe 0xyz este : 215

(1) (2)

(x - a )2 + ( y - b )2 + (z -

=c+2

(x - a)2 + ( y - b)2 + z 2 - 2z

2 =c

Conform Teoremei lui Schinzel a, b, c7 ( putem avea de exemplu 1 1 a= sau $i b=0 iar c p tratul unui num r ntreg). 2 3 atunci ob#inem (x - a )2 + ( y - b )2 = c ce are numai n solu#ii. Cele n puncte laticiale de pe sfera de ecua#ie (1) sunt cele ce se ob#in intersectnd suprafa#a sferic cu planul de ecua#ie z=0 (ob#innd astfel cercul de ecua#ie (2) ce trece prin exact n puncte laticiale). n concluzie, sfera de centru (a, b, 2 ) $i raz puncte laticiale din spa#iu de forma (x, y, 0). & c + 2 trece prin exact n Astfel, dac (x, y, z)33 verific ecua#ia (2), atunci cu necesitate z=0 $i

216

S-ar putea să vă placă și